You are on page 1of 178

2

Published by
Vedantu Innovations Private Limited
D. No. 1081, 3rd Floor, Vistar Arcade,
14th Main Rd, Sector 3, HSR Layout
Bangalore, Karnataka, India 560 102
www.vedantu.com
Vedantu Innovations Private Ltd.
All rights reserved. No part of this book may be reproduced or utilized in any form or
by any means, electronic or mechanical including photocopying, recording or by
any information storage and retrieval system, without permission in writing from the
publishers.
Notice : Vedantu is committed to serving students with the best of the resources and
knowledge. Bearing that in mind, we have obtained all the information in this book
from sources regarded as reliable, and taken utmost care in editing and printing this
book. However, we as authors and publishers are not to be held responsible for unin-
tentional mistakes that might have crept in. Having stated that, errors (if any) brought
to our notice shall be gratefully acknowledged and rectified in upcoming editions.

Printed by
Colours Imprint
475C, Adugodi Main Road,
8th Block, Koramangala, Bengaluru 560 095
www.coloursimprint.com
3
4

Founder’s Message
Dear Student,

It gives me immense pleasure to present to you a Ready Reckoner and Practice booklet by
Vedantu - ‘TATVA’. Tatva in Sanskrit, means a collection of “Core” content/truth and isn’t that
what Vedantu brings to your table - the Core? Vedantu, at all times, ensures that you have
easy accessibility to a collection of essential concepts, theory, derivations, definitions, solved
examples, concept videos, and practice questions, important questions from competitive
examinations and I am pleased to say that all questions in ‘Tatva’ come with detailed solutions.

Tatva is the result of the constant endeavour and research done by our highly experienced
team of teachers and subject experts who compiled relevant content for you so that you come
out with flying colours in IIT JEE/NEET and Olympiads. We strongly believe and vouch for the
effectiveness and relevance of this booklet to grab the desired rank in IIT JEE/ NEET.

Key Points on ‘How to obtain optimal benefits from ‘Tatva’ ?


* We suggest starting with the Practice of ‘Tatva questions’ at the end of a chapter after
revision and Practice of basic questions.
* For daily practice of questions, we strongly recommend attempting session assignments
along with Vedantu live sessions
* Tatva equips you with Theory, Concept Videos, and Solved examples to help you revise
concepts, mark your notes, walk you through the entire summary and eventually clear
all your conceptual doubts all by yourself.
* Attempt Tatva questions post revision of 11th and 12th Grade topics. Take one topic at a
time to solve. Attempt any revision test series for NEET/JEE once you have solved Tatva
questions. Following this will not only help you during revision but also give a major
boost to your confidence.
* First solve basic level questions and gradually progress to the advanced level.
* Practice basic level questions regularly all through your journey of preparation.
* Your success is our dream-come-true

I wish you all the best.

Anand Prakash
Founder, Academic Head
Vedantu

Anand Prakash Sir has been a pioneer in producing


Top Ranks in JEE/NEET and Olympiads. He has
personally taught and mentored AIR 1,6,7 (JEE
ADVANCED) and AIR-1, 7, 9 (AIIMS), and thousands of
more students who have successfully cleared these
competitive exams in the last few years.
5

Credits
“Happiness lies in the joy of achievement
and the thrill of creative effort.”
—Theodore Roosevelt

Tatva is the brainchild of the creative Vedans who The managers who understood every aspect of what
strived tirelessly to weave success stories for you. Our the leadership were trying to accomplish and brought
heartfelt thanks to the Super Vedans who give wings so much of their own to the table and managed the
to the vision of Vedantu. execution of ‘Tatva’ immaculately.

Our leaders who have been our guiding light and Kiran Kumari Sushmitha M D
encouragement in every step: Harish Rao Akshatha J
Vamsi Sir, Anand Prakash Sir and Pulkit Sir Bhavya Bangera J Charubak Chakrabarti
Shraddha R Shetty
Our gratitude to the insightful leadership and
guidance of our leaders who dreamt about ‘Tatva’,
steered the project in the right direction and were
instrumental in making this dream into a reality:
Sahil Bhatia Sudhanshu Jain
Shubam Gupta Ajay Mittal
Arshad Shahid Jaideep Sontakke

Mathematics Team
Our true appreciation for all the Master Teachers of Assistant Teachers
Vedantu whose relentless efforts helped us in Mohammed Siddiqui Priyadharshini R
accomplishing this vision into reality. Our heartfelt Jyoti Garg Pravin S Rathore
gratitude to our creative content developers, and the Anjulika Gupta Sukamal Chatterjee
DTP team who have put in their hard work, insights, Pankaj Gupta
eagerness to execute and nurture Tatva into ‘your SME
ready handbook’ and to bring positive learning Nikhil Goyal(Team Lead) Aman Bhartiya
experience to you. Vipul Sachdeva
DTP
Raman Kumar
A loud shout out for our media team - a bunch of creative minds with contagious energy. We cannot thank
them enough.

Suraj Bhan Singh


Gadde Ranjith
Ranjeeth Ramesh
Sreyoshi Biswas
Devika Ramachandran

Special thanks and appreciation for the enthusiastic support provided by Arunima Kar, Savin Khandelwal,
Dipshi Shetty and Mohamed Anzar.

The journey of the imagination to the reality of ‘Tatva’ would not have been possible without our enthusiastic
Operations Team, our amazing Academic Team, our dedicated team of Teachers and our talented Tech
Team.
Table of Contents

QUADRATIC EQUATION
 Theory .............................................................................................................................................. 10

 Solved examples .............................................................................................................................. 14

 Exercise - 1 : Basic Objective Questions ......................................................................................... 20

 Exercise - 2 : Previous Year JEE Mains Questions .......................................................................... 26

 Exercise - 3 : Advanced Objective Questions ................................................................................ 29

 Exercise - 4 : Previous Year JEE Advanced Questions ................................................................... 36

 Answer Key ....................................................................................................................................... 41

COMPLEX NUMBER

 Theory ............................................................................................................................................... 43

 Solved examples ............................................................................................................................... 49

 Exercise - 1 : Basic Objective Questions ......................................................................................... 58

 Exercise - 2 : Previous Year JEE Mains Questions .......................................................................... 62

 Exercise - 3 : Advanced Objective Questions ................................................................................. 65

 Exercise - 4 : Previous Year JEE Advanced Questions .................................................................... 72

 Answer Key ....................................................................................................................................... 79


SEQUENCE AND SERIES

 Theory ............................................................................................................................................... 81

 Solved examples ............................................................................................................................... 84

 Exercise - 1 : Basic Objective Questions ..........................................................................................91

 Exercise - 2 : Previous Year JEE Mains Questions ........................................................................... 96

 Exercise - 3 : Advanced Objective Questions ................................................................................. 101

 Exercise - 4 : Previous Year JEE Advanced Questions .................................................................... 107

 Answer Key ....................................................................................................................................... 113

BINOMIAL THEOREM

 Theory ............................................................................................................................................... 115

 Solved examples ............................................................................................................................... 118

 Exercise - 1 : Basic Objective Questions ..........................................................................................126

 Exercise - 2 : Previous Year JEE Mains Questions ........................................................................... 130

 Exercise - 3 : Advanced Objective Questions ................................................................................. 133

 Exercise - 4 : Previous Year JEE Advanced Questions .................................................................... 137

 Answer Key ....................................................................................................................................... 139


PERMUTATION AND COMBINATION

 Theory ............................................................................................................................................... 141

 Solved examples .............................................................................................................................. 145

 Exercise - 1 : Basic Objective Questions ......................................................................................... 153

 Exercise - 2 : Previous Year JEE Mains Questions ........................................................................... 159

 Exercise - 3 : Advanced Objective Questions ................................................................................. 162

 Exercise - 4 : Previous Year JEE Advanced Questions .................................................................... 171

 Answer Key ....................................................................................................................................... 174


01
QUADRATIC EQUATION
QUADRATIC EQUATION 10

QUADRATIC EQUATION

1. QUADRATIC EXPRESSION 3. NATURE OF ROOTS

The general form of a quadratic expression in x is,


(a) Consider the quadratic equation ax2 + bx + c = 0 where
2
f (x) = ax + bx + c, where a, b, c  R & a  0. a, b, c  R & a  0 then;
and general form of a quadratic equation in x is,
(i) D > 0  roots are real & distinct (unequal).
2
ax + bx + c = 0, where a, b, c  R & a  0.
(ii) D = 0  roots are real & coincident (equal).
2. ROOTS OF QUADRATIC EQUATION
(iii) D < 0  roots are imaginary..
(a) The solution of the quadratic equation,
(iv) If p + i q is one root of a quadratic equation,
then the other must be the conjugate p – i q &
2 b ± b2  4ac
ax + bx + c = 0 is given by x =
2a 
vice versa. p, q  R & i = -1 . 
The expression D = b 2 – 4ac is called the discriminant
of the quadratic equation. (b) Consider the quadratic equation ax2 + bx + c = 0 where
a, b, c  Q & a  0 then;
(b) If  &  are the roots of the quadratic equation
(i) If D > 0 & is a perfect square, then roots are
ax 2 + bx + c = 0, then ;
rational & unequal.
(i)  +  = – b/a (ii) α β = c/a
(ii) If α = p + q is one root in this case, (where p is
D
(iii) | α  β |= .
|a|
rational & q is a surd) then the other root must
(c) A quadratic equation whose roots are  &  is
(x –  ) (x –  ) = 0 i.e. be the conjugate of it i.e. β = p - q & vice versa.

x 2 – ( +  ) x +  = 0 i.e.

x 2 – (sum of roots) x + product of roots = 0.

Remember that a quadratic equation cannot have three


different roots & if it has, it becomes an identity.
y  (ax 2  bx  c)  a(x  ) (x  )

2
 b  D
 a x   
 2a  4a
QUADRATIC EQUATION 11

4. GRAPH OF QUADRATIC EXPRESSION 6. MAX. & MIN. VALUE OF QUADRATIC EXPRESSION

Consider the quadratic expression, y = ax 2 + bx + c, Maximum & Minimum Value of y = ax2 + bx + c occurs
at x = –(b/2a) according as :
a  0 & a, b, c  R then ;
For a > 0, we have :
(i) The graph between x, y is always a parabola.
If a > 0 then the shape of the parabola is
concave upwards & if a < 0 then the shape of
the parabola is concave downwards.

(ii) y > 0  x  R, only if a>0&D<0


 4ac - b 2 
(iii) y < 0  x  R, only if a<0&D<0 y ,  
 4a 

5. SOLUTION OF QUADRATIC INEQUALITIES

ax 2 + bx + c > 0  a  0  .

(i) If D > 0, then the equation ax 2 + bx + c = 0 has D b


y min  at x  , and y max  
4a 2a
two different roots (x 1 < x2).

Then a > 0  x (–, x1) (x2 , )


For a < 0, we have :
a<0  x (x1, x2)

a>0

x1 x2
x x
x1 x2
a<0  4ac  b 2 
y   , 
 4a 

P x
(ii) Inequalities of the form 0 can be
Qx
D b
y max  at x  , and y min   
4a 2a
quickly solved using the method of intervals
(wavy curve).
12 QUADRATIC EQUATION

7. THEORY OF EQUATIONS

If 1, 2, 3, ....., n are the roots of the nth degree
polynomial equation : Remainder Theorem : If f (x) is a polynomial, then
f (h) is the remainder when f (x) is divided by x – h.
f (x) = a0xn + a1xn–1 + a2xn–2 + ...... + an–1x + an = 0
Factor theorem : If x = h is a root of equation
f (x) = 0, then x–h is a factor of f (x) and conversely.
where a 0, a 1, ....... a n are all real & a0  0,

Then,

a 9. MAX. & MIN. VALUES OF RATIONAL EXPRESSION


 α1 = – a 1 ;
0
Here we shall find the values attained by a rational
a
 α1 α 2 = + a 2 ; a1x 2  b1x  c1
0
expresion of the form for real values
a 2 x 2  b2 x  c2

a
 α1 α 2 α3 = – a 3 ; of x.
0
Example No. 4 will make the method clear.
............
10. COMMON ROOTS
a n
α1 α 2 α3 .....α n = (–1) n
a0 (a) Only One Common Root

Let  be the common root of ax 2 + bx + c = 0 &


8. LOCATION OF ROOTS
a’x 2 + b’x + c’ = 0, such that a, a’  0 and a b’  a’b.
Let f (x) = ax2 + bx + c, where a > 0 & a, b, c  R.
Then, the condition for one common root is :
(i) Conditions for both the roots of f(x) = 0 to be
greater than a specified number ‘k’ are : (ca  ca) 2 = (ab  a b) (bc  bc).
D 0 & f (k) > 0 & (–b/2a) > k.
(ii) Conditions for both roots of f (x) = 0 to lie on (b) Two Common Roots
either side of the number ‘k’ (in other words the Let  ,  be the two common roots of
numb e r ‘k’ l ies b etween the r o o ts o f
f (x) = 0 is: ax 2 + bx + c = 0 & a’x 2 + b’x + c’ = 0,
a f (k) < 0. such that a, a’  0.
(iii) Conditions for exactly one root of f (x) = 0 to lie
Then, the condition for two common roots is :
in the interval (k1, k2) i.e. k1 < x < k2 are :
D>0 & f (k1) . f (k2) < 0. a b c
 
a ' b ' c'
(iv) Conditions that both roots of f(x) = 0 to be
confined between the numbers k 1 & k 2 are
(k1 < k2) :
D  0 & f (k1) > 0 & f (k2) > 0 & k1 < (–b/2a) < k2.
QUADRATIC
13 EQUATION 13
QUADRATIC EQUATION

(iii) If there be any two real numbers ‘a’ & ‘b’ such
11. RESOLUTION INTO TWO LINEAR FACTORS
that f (a) & f (b) are of opposite signs, then
The condition that a quadratic function f (x) = 0 must have atleast one real root between
‘a’ and ‘b’.
f (x, y) = ax 2 + 2 hxy + by2 + 2 gx + 2 fy + c
(iv) Every equation f(x) = 0 of degree odd has
may be resolved into two linear factors is that ;
atleast one real root of a sign opposite to that
abc + 2fgh – af2 – bg2 – ch2 = 0 of its last term.
a h g 13. TRANSFORMATION OF EQUATIONS
OR h b f =0
g f c (i) To obtain an equation whose roots are reciprocals
of the roots of a given equation, it is obtained by
replacing x by 1/x in the given equation
12. FORMATION OF A POLYNOMIAL EQUATION (ii) Transformation of an equation to another
equation whose roots are negative of the roots
If 1, 2, 3, ....., n are the roots of the nth degree of a given equation–replace x by – x.
polynomial equation, then the equation is (iii) Transformation of an equation to another
n
x – S1x n–1
+ S2x n–2
+ S 3x n–3 n
+ ...... + (–1) Sn = 0 equation whose roots are square of the roots of a

where Sk denotes the sum of the products of roots given equation–replace x by x.


taken k at a time. (iv) Transformation of an equation to another
equation whose roots are cubes of the roots of a
Particular Cases
given equation–replace x by x1/3.
(a) Quadratic Equation if ,  be the roots the
quadratic equation, then the equation is :

x2 – S1x + S2 = 0 i.e. x2 – ( + ) x +  = 0

(b) Cubic Equation if , ,  be the roots the cubic


equation, then the equation is :

x3 – S1x2 + S2x – S3 = 0 i.e.

x3 – ( + +) x2 + ( +  + ) x –  = 0

(i) If  is a root of equation f(x) = 0, the polynomial


f (x) is exactly divisible by (x – ). In other words,
(x – ) is a factor of f(x) and conversely.

(ii) Every equation of nth degree (n  1) has exactly


n roots & if the equation has more than n roots,
it is an identity.
QUADRATIC
14 EQUATION 14
QUADRATIC EQUATION

SOLVED EXAMPLES

Example – 1 Example – 2

If the remainder on dividing x3 + 2x2 + kx + 3 by x – 3 is 21, Find all the zeros of the polynomial x4 + x3 – 9x2 –3x + 18 if
find the quotient and the value of k. Hence find the zeros
it is given that two of its zeros are  3 and 3.
of the cubic polynomial x3 + 2x2 + kx – 18.
4 3 2
3 2 Sol. Given polynomial f(x) = x + x – 9x – 3x + 18 has two of its
Sol. Let p (x) = x + 2x + kx + 3.
We are given that when p (x) is divided by the linear zeros  3 and 3.
polynomial x – 3, the remainder is 21.
 p (3) = 21 (Remainder Theorem)  (x  3) (x  3) is a factor of f (x),
3 2
 3 + 2 × 3 + k × 3 + 3 = 21 2
i.e., x – 3 is a factor of f (x).
 27 + 18 + 3k + 3 = 21 Now, we apply the division algorithm to the given polynomial
 3k = 21 – 27 – 18 – 3 2
with x – 3.
 3k = –27
 k = –9
3 2 2
Hence, p (x) = x + 2x – 9x + 3. x +x–6
To find the quotient obtained on dividing p(x) by x–3, we x 2  3 x 4  x 3  9x 2  3x  18
perform the following division :
x4  3x 2
 
x 2  5x  6
x  3 x  2x 2  9x  3
3 x 3  6x 2  3x  18

x 3  3x 2 x3  3x
   

5x 2  9x  3 6x 2  18

5x 2  15x 6x 2  18
   
6x  3 0  Remainder
6x  18
  4 3 2
Thus, x + x – 9x – 3x + 18
21
2 2
= (x – 3) (x + x – 6)
2
Hence, p (x) = (x + 5x + 6) (x–3) + 21 2 2
= (x – 3) × {x + 3x – 2x – 6}
(Divisor × Quotient + Remainder) 2
3 2 2
= (x – 3) × {x (x + 3) – 2 (x + 3)}
 x + 2x – 9x + 3 – 21 = (x + 5x + 6) (x – 3) 2
3 2 2 = (x – 3) × (x + 3) (x – 2)
 x + 2x – 9x – 18 = (x + 3x + 2x + 6) (x –3)
3 2 Putting x + 3 = 0 and x – 2 = 0
 x + 2x – 9x – 18 = (x + 3) (x + 2) (x – 3)
Hence, the zeros of x + 2x – 5x – 18 are given by
3 2 we get x = –3 and x = 2, i.e., –3 and 2 are the other two zeros
x + 3 = 0, x + 2 = 0, x – 3 = 0 of the given polynomial.
 x = –3, – 2, 3 Hence  3, 3, –3, 2 are the four zeros of the given
3 2
 The zeros of x + 2x – 9x – 18 are – 3, –2, 3.
polynomial.
QUADRATIC EQUATION 15

Example – 3 Example – 4

If the polynomial x4 – 6x3 + 16x2 – 25x + 10 is divided by


3x 2  7x  8
another polynomial x2 – 2x + k, the remainder comes out to Solve the inequality, 2
x2 1
be x + a, find k and a.

Sol. By division algorithm Sol. Domain : x  R


4 3 2 2 Given inequality is equivalent to
x – 6x + 16x – 25x + 10 = (x – 2x + k) q (x) + (x + a)

where q (x) is the quotient. 3x 2  7x  8


2 0
x2 1
As the degree on L.H.S. is 4; therefore, q (x) must be of
degree 2. 3x 2  7x  8  2x 2  2
 0
2 x2 1
Let q (x) = lx + mx + n, l 0.
4 3 2 2 2
Then x – 6x + 16x – 25x + 10 = (x – 2x + k) (lx + mx + n) + x + a 3x 2  7x  6  x  1 x  6 
 2
0  0
4 3 2 4 3 2
x 1 x2 1
 x – 6x + 16x – 25x + 10 = lx + (m–2l) x + (n–2m + kl) x + (mk
– 2n + 1) x + nk + a

Equating coefficients of like powers of x on the two sides,

we obtain
 x  [1, 6]
l=1 ... (1)
Example – 5
m – 2l = –6 ... (2)
Solve the equation 25x2 – 30x + 11 = 0 by using the general
n – 2m + kl = 16 ... (3) expression for the roots of a quadratic equation.
mk – 2 n + l = – 25 ... (4)
Sol. Comparing the given equation with the general form of a
and nk + a = 10 ... (5) 2
quadratic equation ax + bx + c = 0, we get
From (2), m = –6 + 2l = –6 + 2 × 1 = –4 and a = 25, b = –30 and c = 11.
Substituting these values in
then from (3), n = 16 + 2m – kl = 16 + 2 × (–4) – k × 1

 n=8–k ... (6)  b  b 2  4ac  b  b 2  4ac


 and  
2a 2a
From (4) and (6), we get
30  900  1100 30  900  1100
(–4) k – 2 (8 – k) + 1 = –25  and  
50 50
 – 4k – 16 + 2k = –25
30  200 30  200
 – 2k = –25 + 16 – 1   and  
50 50
 – 2k = –10 k = 5
30  10i 2 30  10i 2
Substituting this value of k in (6), we have   and  
50 50
n = 8 – 5 = 3 and then from (5),
3 2 3 2
   i and    i
we get 5 5 5 5
a = 10 – nk = 10 – 3 × 5 = –5. 3 2
Hence, the roots of the given equation are  i
5 5
16 QUADRATIC EQUATION

Example – 6

Solve the following quadratic equation by factorization


method :
x2 – 5ix – 6 = 0 An equation of the form
f (x) f (x)
a +b =c
Sol. The given equation is where a, b, c  R
2
x – 5ix – 6 = 0 and a, b, c satisfies the condition a + b = c
2 2

2 2
 x – 5ix + 6i = 0 then solution of the equation is f (x) = 2 and no other

2
x – 3ix – 2ix + 6i = 0
2 solution of this equation.
2 2
(ii) Here, 3 + 5 = 34, then given equation has a solution
 x (x – 3i) – 2i (x – 3i) = 0
x–4=2
 (x – 3i) (x – 2i) = 0
 x = 6 is a root of the original equation
 x – 3i = 0, x – 2i = 0
 x = 3i, x = 2i
Hence, the roots of the given equation are 3i and 2i.
g(x)
An equation of the form {f (x)} is equivalent to the
Example – 7 equation
g(x) g (x) log f (x)
Solve the equation {f (x)} = 10 where f (x) > 0
x+1 2–x
(i) 15.2 + 15.2 = 135.
x–4 x–4
(iii) We have 5x x
8x 1  500
(ii) 3 +5 = 34
x 3 2
(iii) 5x x
8x 1  500  5x 8x 1 = 5 . 2

 x 1 
 
Sol. (i) The equation rewrite in the form  5x .8 x 
 53.22

3x  3
60
30.2 x   135  5x .2 x
 53.22
2x
 x 3 
x  
Let t = 2  5x 3.2 x 
1
2
then 30t – 135 t + 60 = 0  (5.2 )
1/x (x – 3)
=1
2
6t – 27t + 12 = 0 is equivalent to the equation
2
 6t – 24t – 3t + 12 = 0  x 3 log  5.21/ x 
10 1
 (t – 4) (6t – 3) = 0 1/x
 (x – 3) log (5.2 ) = 0
1 Thus original equation is equivalent to the collection of
then t1 = 4 and t2 =
2 equations
1/x
x – 3 = 0, log (5.2 ) = 0
thus given equation is equivalent to 1/x
 x = 3, 5.2 = 1
1/x
x x 1  2 = (1/5)
2 = 4 and 2 =
2  x = –log52
then x = 2 and x = – 1 Hence roots of the original equation are

Hence roots of the original equation are x1 = 2 and x2 = – 1 x = 3 and x = –log52


QUADRATIC EQUATION 17

Example – 8 Example – 10

Form an equation whose roots are cubes of the roots of If P (x) = ax2 + bx + c and Q (x) = – ax2 + bx + c,
3 2
equation ax + bx + cx + d = 0
where ac  0, show that the equation

Sol. Replacing x by x1/3 in the given equation, we get P (x) · Q (x) = 0 has at least two real roots.

a (x1/3)3 + b (x1/3)2 + c (x1/3) + d =0 Sol. Roots of the equation P (x) Q (x) = 0


 ax + d = – (bx2/3 + cx1/3) ....... (i) i.e., (ax2 + bx + c) (–ax2 + bx + c) = 0 will be
 (ax + d)3 = – (bx2/3 + cx1/3)3 roots of the equations

 a3x3 + 3a2dx2 + 3ad2x + d3 ax2 + bx + c = 0 ....... (i)

= – [b3x2 + c3x + 3bcx (bx2/3 + cx1/3)] and – ax2 + bx + c = 0 ........ (ii)

 a3x3 + 3a2dx2 + 3ad2x + d3 If D1 and D2 be discriminants of (i) and (ii) then

[ – b3x2 – c3x + 3bcx (ax + d)] [From Eq. (i)] D1 = b2 – 4ac and D2 = b2 + 4ac

 a3x3 + x2 (3a2d – 3abc + b3) Now D1 + D2 = 2b2 0

+ x (3ad2 – 3bcd + c3) + d3 = 0 (since, b may be zero)

This is the requied equation. i.e., D 1 + D2  0

Example – 9 Hence, at least one of D1 and D2  0

i.e., at least one of the equations (i) and (ii) has real roots and
If , ,  be the roots of the equation
therefore, equation P (x) Q (x) = 0 has at least two real roots.
2 2
x (1 + x ) + x (6 + x) + 2 = 0,
Alternative Sol.
–1 –1 –1
then the value of  +  +  is
Since, ac  0

1  ac < 0 or ac > 0
(a) –3 (b)
2
Case I :
1
(c)  (d) None of these If ac < 0  – ac > 0
2
then D1 = b2 – 4ac > 0
Sol. 2x3 + 6x2 + x + 2 = 0 has roots , , .
Case II :
So, 2x3 + x2 + 6x + 2 = 0 has roots –1, –1, –1
(writing coefficients in revers order, since roots are reciprocal) If ac > 0

then D2 = b2 + 4ac > 0


Hence, Sum of the roots = –
 Coefficient of x  2

 Coefficient of x  3
So, at least one of D1 and D2 > 0.

1 Hence, at least one of the equations (i) and (ii) has real roots.
  1  1   1  
2
Hence, equation P (x) · Q (x) = 0 has at least two real roots.
Hence, (c) is the correct answer.
18 QUADRATIC EQUATION

Example – 11 Example – 13

Find all roots of the equation x4 + 2x3 – 16x2 – 22x + 7 = 0 if one


If x2 – ax + b = 0 and x2 – px + q = 0 have a root in common
and the second equation has equal roots. root is 2  3 .
ap
show that b + q = .
2 Sol. All coefficients are real, irrational roots will occur in
conjugate pairs.
Sol. Given equations are x2 – ax + b = 0 ...(i)
Hence another roots is 2  3 .
and x2 – px + q = 0 ...(ii)
 Product of these roots = (x – 2 – 3 ) (x – 2 + 3 )
Let  be the common root. Then roots of Eq. (ii) will be  2
and . Let  be the other root of Eq. (i). Thus roots of Eq. = (x – 2) – 3
2
(i) are ,  and those of Eq. (ii). are ,  = x – 4x + 1
= a ...(iii) 4 3 2 2
b ...(iv) Dividing x + 2x – 16x – 22x + 7 by x – 4x + 1 then the other
2
2p ...(v) quadratic factor is x + 6 x + 7
 q ...(vi) then the given equation reduce in the form
LHS = b + q = + 2 =  (+ ) ...(vii) 2 2
(x – 4x + 1) (x + 6x + 7) = 0
ap      2
and RHS =       ...(viii) 
2
x + 6x + 7 = 0
2 2
From Eqs. (vii) and (viii), LHS = RHS
6  36  28
Example – 12 then x 
2

The diagram shows the graph of = 3  2

y = ax2 + bx + c, Then,
Hence roots 2  3,  3  2
(0, c)
Example – 14

x1 x2 x 2  3x  4
If x is real, then prove that the values of lies
x 2  3x  4
(a) a > 0 (b) b < 0 1
(c) c > 0 (d) b2 – 4ac = 0 between and 7.
7

Sol. As it is clear from the figure that it is a parabola opens


downwards i.e. a < 0.
x 2  3x  4
 It is y = ax2 + bx + c i.e. degree two polynomial Sol. Let y = . Then,
x 2  3x  4
Now, if ax2 + bx + c = 0
 it has two roots x1 and x2 as it cuts the axis at two distinct x2 (y – 1) + 3x (y + 1) + 4 (y – 1) = 0
point x1 and x2.  for x  real, D  0
Now from the figure it is also clear that x1 + x2 < 0  9 (y + 1)2 – 16 (y – 1)2 0
(i.e. sum of roots are negative)
 – 7y2 + 50y – 7  0
b b  (7y – 1) (y – 7)  0
 0  0
a a
 b < 0 ( a < 0) (b) is correct. 1
Hence, the given expression lies between and 7.
As the graph of y = f (x) cuts the + y-axis at (0, c) 7

where c > 0  (c) is correct.


QUADRATIC
19 EQUATION 19
QUADRATIC EQUATION

Example – 15 As in Case I, [1, 3] should be the subset of

a, b, c  R, a  0 and the quadratic equation (a + 3, 3a)


ax2 + bx + c = 0 has no real roots, then, i.e., a+3<1 and 3a > 3 ...(iv)
Combining (iii) and (iv), we get :
(a) a + b + c > 0 (b) a (a + b + c) > 0
(c) b (a + b + c) > 0 (d) c (a + b + c) >0 a   i.e. No solution ...(v)

Combining both cases, we get : a  (0, 1/3)


Sol. Let f (x) = ax2 + bx + c. It is given that f (x) = 0 has no real
Alternate Sol.
roots. So, either f (x) > 0 for all x  R
Let f (x) = (x – 3a) (x – a – 3)
or f (x) < 0 for all x  R i.e. f (x) has same sign for all values
of x. for given equality to be true for all values of
x  [1, 3], 1 and 3 should lie between the roots of
 f (0) f (1) > 0
f (x) = 0.
 c (a + b + c) > 0
 f (1) < 0 and f (3) < 0
Also, af (1) > 0
Consider f (1) < 0 :
 a (a + b + c) > 0.
 (1 – 3a) (1 – a – 3) < 0
Example – 16  (3a – 1) (a + 2) < 0
Find the values of a for which the inequality  a (–2, 1/3) ...(i)
(x – 3a) (x – a – 3) < 0 is satisfied for all x such that Consider f (3) < 0 :
1  x  3.
 (3 – 3a) (3 – a – 3) < 0
Sol. (x – 3a) (x – a – 3) < 0  (a – 1) (a) < 0
 a  (0, 1) ...(ii)
Case I :
Combining (i) and (ii), we get :
Let 3a < a + 3 a < 3/2 ...(i)
a  (0, 1/3)
Solution set of given inequality is x (3a, a + 3)
Now for given inequality to be true for all Example – 17
x  [1, 3], set [1, 3] should be the subset of
(3a, a + 3) i.e. 1 and 3 lie inside 3a and a + 3 on number line. If ax2 – bx + 5 = 0 does not have two distinct real roots,
then find the minimum value of 5a + b.
So we can take, 3a < 1 and a + 3 > 3 ...(ii)
Combining (i) and (ii), we get : Sol. Let f(x) = ax2 – bx + 5
 a (0, 1/3) Since, f (x) = 0 does not have two distinct real roots, we
Case II : have either
Let 3a > a + 3  a > 3/2 ...(iii) f  x   0  x R or f  x   0  x R
Solution set of given inequality is
But f (0) = 5 > 0, so f (x)  0  x R
x (a + 3, 3a)
In particular f (–5) > 0  5a + b > – 1
Hence, the least value of 5a + b is –1.
QUADRATIC
20 EQUATION 20
QUADRATIC EQUATION

EXERCISE - 1 : BASIC OBJECTIVE QUESTIONS


Basics, Sum & Product of Roots
10. If 9x 2  6x  1   2  x  , then
1. Roots of the equation x2 + x (2 – p2) – 2p2 = 0 are
(a) – p2 and – 2 (b) p2 and – 2  3 1  3 1
(a) x    ,  (b) x    , 
(c) – p2 and 2 (d) p2 and 2  2 4  2 4

x  1  x  1  1, then x is equal to  3 1 1
2. If (c) x    ,  (d) x 
 2 4 4
(a) 2/3 (b) 3/5
(c) 5/4 (d) 4/5 11. If  and  are the roots of ax 2 + bx + c = 0 then the value

of (aα + b)-2 + (aβ + b)-2 is equal to


x y 10
3. If   and x + y = 10, then the value of xy will
y x 3
b 2 - 2ac c2 - 2ab
(a) (b)
be a 2 c2 a 2 b2
(a) 36 (b) 24
(c) 16 (d) 9 a 2 - 2bc
(c) (d) None
b2c2
8x 2 16x  51
4.  3 if x is such that
 2x  3 x  4  12. If  and  are the roots of 4x 2 + 3x + 7 = 0, then the

(a) x < – 4 (b) –3 < x < 3/2 1 1


value of 3
+ is
(c) x > 5/2 (d) all these true α β3

5. The sum of all real roots of the equation


27 63
|x – 2|2 + |x – 2| – 2 = 0, is (a)  (b)
64 16
(a) 0 (b) 8
(c) 4 (d) none of these 225
(c) (d) None of these
6. 2
The equation x –3 |x| + 2 = 0 has 343

(a) no real roots (b) one real root 13. If  and  are the roots of the equation

(c) two real roots (d) four real roots x 2  px  p2  q  0, then the value of

7. The sum of the real roots of the equation x2 + |x| – 6 = 0 is  2    2  q 


(a) 4 (b) 0
(a) 0 (b) 1
(c) – 1 (d) none of these
(c) q (d) 2q
8. The product of the real roots of the equation
|2x + 3|2 – 3 |2x + 3| + 2 = 0, is 14. If  ,  are the roots of the equation 8x 2  3x  27  0,

(a) 5/4 (b) 5/2 1/3 1/3


 2   2 
then the value of   
  is
(c) 5 (d) 2     

9. The roots of the equation |x2 – x – 6| = x + 2 are (a) 1/4 (b) 1/3

(a) –2, 1, 4 (b) 0, 2, 4 (c) 7/2 (d) 4

(c) 0, 1, 4 (d) –2, 2, 4


QUADRATIC EQUATION 21

24. If , are the roots of the equation x2 – p (x + 1) –c =0, then


15. If p, q are the roots of the equation x 2  px  q  0 where
(+ 1) (+ 1) =
both p and q are non-zero, then (p, q) =
(a) c (b) c–1
(a) (1, 2) (b) (1, –2)
(c) 1–c (d) none of these
(c) (–1, 2) (d) (–1, –2)
16. The product of the roots of the equation 25. If  and  are the roots of x 2  p (x  1)  c  0, then the

mx 2  6x  (2m  1)  0 is –1. Then m is equal to  2  2  1 2  2  1


value of  is
 2  2  c 2  2  c
(a) 1 (b) 1/3
(c) –1 (d) –1/3 (a) 2 (b) 1
17. If one root of the equation ax2 + bx + c = 0, a  0, is reciprocal (c) –1 (d) 0
of the other, then
26. If one root of x 2  x  k  0 be square of the other, then k
(a) b = c (b) a = c
is equal to
(c) a = 0 (d) b = 0
(a) 2  3 (b) 3  2
18. If the sum of the roots of the equation ax2 + 2x + 3a = 0 is
equal to their product, then value of a is (c) 2  5 (d) 5  2
2
(a)  (b) –3 27. The number of values of a for which
3
1 (a2 – 3a + 2) x2 + (a2 – 5a + 6) x + a2 – 4 = 0
(c) 4 (d) 
2 is an identity in x, is
19. If the product of the roots of the quadratic equation
mx2 – 2x + (2m – 1) = 0 is 3 then the value of m (a) 0 (b) 2

(a) 1 (b) 2 (c) 1 (d) 3

(c) –1 (d) 3 28. If p (x  1)2  q (x 2  3x  2)  x  1  0 be an identity in


20. If the roots of px2 + qx + 2 = 0 are reciprocals of each other, x, then p, q are
then (a) 2, –2 (b) 1, –1
(a) p = 0 (b) p = –2 (c) 0, 0 (d) none
(c) p = ± 2 (d) p = 2 29. If the difference between the roots of x2 + ax + b = 0 and
2
21. If the equation (k – 2) x – (k – 4) x – 2 = 0 has difference of x2 + bx + a = 0 is same and a  b, then
roots as 3 then the value of k is (a) a + b + 4 = 0 (b) a + b –4 = 0
(a) 1, 3 (b) 3, 3/2 (c) a – b –4 = 0 (d) a – b + 4 = 0
(c) 2, 3/2 (d) 3/2, 1 30. 2
If roots of the equation x + ax + 25 = 0 are in the ratio of
22. If sin  and cos are the roots of the equation 2 : 3 then the value of a is

lx 2  mx  n  0, then 5  25
(a) (b)
6 6
(a) l 2 – m 2  2ln  0 (b) l 2  m 2  2ln  0
5
(c) l 2 – m 2 – 2ln  0 (d) l 2  m 2 – 2ln  0 (c) (d) none of these
6
23. The roots of the equation x 2  px  q  0 are 31. If the roots of the equations x2 + 3x + 2 = 0 & x2 – x +  = 0
tan 22º and tan 23º then are in the same ratio then the value of  is given by
(a) p + q = 1 (b) p + q = –1 (a) 2/7 (b) 2/9
(c) p – q = 1 (d) p – q = –1 (c) 9/2 (d) 7/2
22 QUADRATIC EQUATION

32. If  are roots of the equation x2 – 5x + 6 = 0 then the 40. If , ,  are the roots of the equation
equation whose roots are  + 3 and  + 3 is
(a) x2 – 11x + 30 = 0 (b) (x – 3)2 – 5 (x – 3) + 6 = 0 x3  px 2  qx  r  0, then
(c) Both (a) and (b) (d) None of these
(1  2 ) (1  2 ) (1   2 ) is equal to
33. If  are roots of Ax + Bx + C = 0 and  are roots of
2

x2 + px + q = 0, then p is equal to (a) (1  q) 2  (p  r) 2 (b) (1  q) 2  (p  r)2


2 2 2 2
(a) (B – 2AC)/A (b) (2AC – B )/A
2
(c) (B – 4AC)/A 2
(d) (4AC – B2)A2 (c) (1  q) 2  (p  r)2 (d) none of these

34. If ,  are roots of the equation 41. If (x2 – 3x + 2) is a factor of x4 – px2 + q = 0, then the values
of p and q are
 2 2 (a) –5, 4 (b) 5, 4
ax 2  3x  2  0 (a  0), then  is greater than
  (c) 5, –4 (d) –5, –4
(a) 0 (b) 1 42. The least integral value of k which makes the roots of the
equation x2 + 5x + k = 0 imaginary is
(c) 2 (d) none of these
(a) 4 (b) 5
35. In a quadratic equation with leading coefficient 1, a student
reads the coefficient 16 of x wrongly as 19 and obtain the (c) 6 (d) 7
roots as –15 and –4. The correct roots are 43. The roots of the quadratic equation 7x2 – 9x + 2 = 0 are
(a) 6, 10 (b) –6, –10 (a) Rational and different (b) Rational and equal
(c) –7, –9 (d) none of these (c) Irrational and different (d) Imaginary and different
Cubic, Biquadratic, Nature of Roots
44. The roots of the equation x 2  2 2 x 1  0 are
36. If   and  are the roots of the cubic equation
(x – 1)(x2 + x + 3)=0, then the value of  +  +  is equal to (a) Real and different (b) Imaginary and different
(a) –1 (b) 0 (c) Real and equal (d) Rational and different
(c) 2 (d) 3 45. If a, b, c,  Q and b + c  0 then the roots of the equation

37. If one root of equation x2 + ax + 12 = 0 is 4 while the equation (b + c) x2 – (a + b + c) x + a = 0 are


x2 + ax + b = 0 has equal roots, then the value of b is (a) Real (b) Real and Rational
4 49 (c) Non real and different (d) Real and equal
(a) (b)
49 4 46. If l, m, n are real, l  m, then the roots of the equation
(l – m) x2 – 5 (l + m) x – 2 (l – m) = 0 are
7 4
(c) (d) (a) real and equal (b) Non real
4 7
(c) real and unequal (d) none of these
38. If , ,  are the roots of the equation 2x3 – 3x2 + 6x + 1 = 0, then
47. If a,b,c are distinct real numbers then the equation
2 + 2 + 2 is equal to
(b  c) x 2  (c  a) x  (a  b)  0 has
15 15
(a)  (b)
4 4 (a) equal roots (b) irrational roots

9 (c) rational roots (d) none of these


(c) (d) 4
4 48. If a,b,c are distinct rational numbers then roots of equation
39. The value of m for which the equation (b  c  2a) x 2  (c  a  2b)x  (a  b  2c)  0 are

x 3  mx 2  3x  2  0 has two roots equal in (a) rational (b) irrational


magnitude but opposite in sign, is (c) non-real (d) equal
(a) 1/2 (b) 2/3
(c) 3/4 (d) 4/5
QUADRATIC
23 EQUATION 23
QUADRATIC EQUATION

49. If a,b,c are distinct rational numbers and a + b + c = 0, then Common Root
the roots of the equation
57. If the equations x 2  2x  3  0 and
2 2 2 2
(c  ab) x  2 (a  bc) x  (b  ac)  0 are
2x 2  3x  5  0 have a non-zero common root, then
(a) imaginary (b) real and equal

(c) real and unequal (d) none of these
(a) 1 (b) –1
50. If a,b,c are distinct rational numbers and a + b + c = 0, then
(c) 3 (d) None
the roots of the equation
58. The value of a so that the equations
(b  c  a) x 2  (c  a  b) x  (a  b  c)  0 are
(2a  5) x 2  4x  15  0 and
(a) imaginary (b) real and equal
(c) real and unequal (d) none of these (3a  8) x 2  5x  21  0 have a common root, is
51. If a  Z and the equation (x – a) (x – 10) + 1 = 0 has integral (a) 4, 8 (b) 3, 6
roots, then the values of a are (c) 1, 2 (d) None
(a) 8, 10 (b) 10, 12
59. If a,b,c  R, the equation ax2  bx  c  0 (a,c  0) and
(c) 12, 8 (d) none
52. The quadratic equation with rational coefficients whose x 2  2x  3  0 have a common root, then a : b : c =
one root is 2 + 3 is (a) 1 : 2 : 3 (b) 1 : 3 : 4
(a) x2 – 4x + 1 = 0 (b) x2 + 4x + 1 = 0 (c) 2 : 4 : 5 (d) None
(c) x2 + 4x – 1 = 0 (d) x2 + 2x + 1 = 0
60. If the equations k(6x 2  3)  rx  2x 2  1  0 and
53. The quadratic equation with real coefficients whose one
root is 2 – i 3 is 6k (2x 2  1)  px  4x 2  2  0 have both roots common,
(a) x2 – 4x + 7 = 0 (b) x2 + 4x – 7 = 0 then the value of (2r – p) is

(c) x2 – 4x – 7 = 0 (d) none of these (a) 0 (b) 1/2

54. The equation of the smallest degree with real coefficients (c) 1 (d) None of these
having 1 + i as one of the roots is Range of Rational Expression
(a) x2 + x + 1 = 0 (b) x2 – 2x + 2 = 0
x 2  2x  4
61. If x is real, then takes values in the interval
x 2  2x  4
(c) x2 + 2x + 2 = 0 (d) x2 + 2x –2 = 0

55. If  are the roots of a quadratic equation x2 – 3x + 5 =0 then 1  1 


(a)  ,3 (b)  ,3 
the equation whose roots are ( 2 – 3 + 7) and 3  3 
(2 – 3 + 7) is
(a) x2 + 4x + 1 = 0 (b) x2 – 4x + 4 = 0  1 
(c) (3, 3) (d)   ,3 
 3 
(c) x2 – 4x – 1 = 0 (d) x2 + 2x + 3 = 0
56. If the roots of a1 x2 + b1 x + c1 = 0 are 1, 1 and those of x 2  14x  9
a2x2 + b2 x + c2 = 0 are 2 2 such that 1 2 = 1 2 = 1, then 62. If x is real then the value of the expression
x 2  2x  3
a1 b1 c1 a1 b1 c1 lies between
(a)   (b)  
a 2 b2 c2 c2 b 2 a 2 (a) –3 and 3 (b) –4 and 5
(c) a1a2 = b1b2 = c1c2 (d) none of these (c) –4 and 4 (d) –5 and 4
24 QUADRATIC EQUATION

Graph of Quadratic Expression 66. If a, b, c  R, for which of the following graphs of


63. 2
If a, b  R & the quadratic equation ax – bx + 1 = 0 has y = ax2 + bx + c, the product a b c is negative.
imaginary roots then a + b + 1 is
(a) positive
(b) negative
(c) zero
(d) depends on the sign of b (a) (b)
64. If a, b, c  R, the graph of the quadratic polynomial;
y = ax2 + bx + c is as shown in the figure. Then

(c) (d)
x

(a) b2 – 4ac > 0 (b) b < 0 67. The integer k for which the inequality
(c) a > 0 (d) c < 0 x2 –2 (4k–1) x + 15k2 – 2k – 7 > 0 is valid for any x, is
65. If a, b, c  R, Which of the following graph represents, (a) 2 (b) 3
f (x) = ax2 + bx + c when a > 0, b < 0 and c < 0 ?
(c) 4 (d) none of these

68. If a  R and x2 + 2ax + 10 –3a > 0 for all x R, then


(a) –5 < a < 2 (b) a < –5
(a) (b)
(c) a > 5 (d) 2 < a < 5
69. The real values of ‘a’ for which (a2 –1) x2 + 2 (a –1) x + 2 is
positive for any x, are
(a) a 1 (b) a 1
(c) a > –3 (d) a < –3 or a  1
70. The real value of a for which the sum of the squares of the
roots of the equation x2 – (a – 2) x – a – 1 = 0 assumes the
(c) (d) least value, is
(a) 0 (b) 1
(c) 2 (d) 3
71. If a, b, c R. Both roots of the equation
(x – b) (x – c) + (x – c) (x – a) + (x – a) (x – b) = 0 are
(a) positive (b) negative
(c) real (d) imaginary
QUADRATIC EQUATION 25

72. If a < b < c < d, then roots of 76. The value of ‘a’ for which the equation
(x – a) (x – c) + 2 (x – b) (x – d) = 0 are
x 2  2 (a  1) x  (2a  1)  0 has both roots
(a) real and equal (b) real and unequal
positive is
(c) imaginary (d) rational
(a) a > 0 (b) 0 < a < 4
Location of Roots
(c) a  4 (d) None of these
73. The value of k for which the equation
3x2 + 2x (k2 + 1) + k2 – 3k + 2 = 0 77. If the equation x 2  2 (a  1) x  9a  5  0 has only
has roots of opposite signs, lies in the interval negative roots, then
(a) (–, 0) (b) (–, –1) 5 
(a) a  (, 6) (b) a   , 1  (6, )
(c)(1, 2) (d) (3/2, 2) 9 
74. The value of a for which the equation
(c) a  (0,6) (d) a  0
2
2x  2 (2a  1)x  a (a  1)  0 has roots,  and  such 78. The value of k for which both the roots of the
that   a   is equation 4x 2  20kx  (25k 2  15k  66)  0 are less
(a) a  0 (b) a < 0 then 2, lies in

(c) 3  a  0 (d) None of these (a) (4/ 5, 2) (b) (2, 0)

75. The value of for which (c) (–1, –4/5) (d) (, 1)

2x 2  2 (2  1) x   (  1)  0 may have one root less 79. If the roots of x 2  x  a  0 exceed a, then
than and other root greater than are given by (a) 2 < a < 3 (b) a > 3
(a) 1    0 (b) –1    0 (c) –3 < a < 3 (d) a < – 2

(c)   0 (d)   0 or   1 80. The range of values of m for which the equation

(m  5) x 2  2 (m  10) x  m  10  0 has real roots of


the same sign, is given by
(a) m > 10 (b) –5 < m < 5
(c) m < –10, 5 < m  6 (d) None of these
QUADRATIC
26 EQUATION 26
QUADRATIC EQUATION

EXERCISE - 2 : PREVIOUS YEAR JEE MAINS QUESTIONS

1. If    but a2 = 5 – 3 and 2 = 5 – 3 then the equation 9. If (1 – p) is a root of quadratic equation x2 + px + (1 – p) = 0, then


its roots are (2004)
whose roots are / and / is (2002)
2 2 (a) 0, –1 (b) –1, 1
(a) 3x – 25x + 3 = 0 (b) x + 5x – 3 = 0
(c) 0, 1 (d) –1, 2
(c) x2 – 5x + 3 = 0 (d) 3x2 – 19x + 3 = 0
2
10. If one root of the equation x + px + 12 = 0 is 4, while the
2. Difference between the corresponding roots of
equation x2 + px + q = 0 has equal roots,
x2 + ax + b = 0 and x2 + bx + a = 0 is same and a  b , then
then the value of q is (2004)
(2002)
(a) 3 (b) 12
(a) a + b + 4 = 0 (b) a + b – 4 = 0
(c) 49/4 (d) 4
(c) a – b – 4 = 0 (d) a – b + 4 = 0
11. The value of a for which the sum of the squares of the
3. Product of real roots of the equation
roots of the equation x2 – (a – 2)x – a – 1 = 0 assume the
x2 + |x| + 9 = 0 (2002) least value, is (2005)
(a) is always positive (b) is always negative (a) 0 (b) 1
(c) does not exist (d) none of the above (c) 2 (d) 3
2
4. If p and q are the roots of the equation x + px + q = 0, then 12. If the roots of the equation x2 – bx + c = 0 be two
(2002) consecutive integers, then b2 – 4c equals (2005)
(a) p = 1, q = – 2 (b) p = 0, q = 1 (a) 3 (b) –2
(c) p = –2, q = 0 (d) p = –2, q = 1 (c) 1 (d) 2
5. If a, b, c are distinct real numbers and a2 + b2 + c2 = 1, 13. If both the roots of the quadratic equation
then ab + bc + ca is (2002) x2 – 2kx + k2 + k – 5 = 0 are less than 5,
(a) less than 1 (b) equal to 1 then k lies in the interval (2005)
(c) greater than 1 (d) any real no
(a)  6,  (b) (5, 6]
6. The value of a for which one root of the quadratic equation
(a2 – 5a + 3) x2 + (3a – 1) x + 2 = 0 is twice as large as the (c) [4, 5] (d)  ,4 
other, is (2003)
14. If the roots of the quadratic equations x2 + px + q = 0 are
(a) –2/3 (b) 1/3
tan 30o and tan 15o respectively, then the value of 2 + q – p is
1 2 (2006)
(c)  (d)
3 3 (a) 2 (b) 3
7. The number of real solution of the equations x2 – 3|x| + 2 = 0 is (c) 0 (d) 1
(2003) 15. All the values of m for which both roots of the equation
(a) 4 (b) 1 x2 – 2mx + m2 –1 = 0 are greater than –2 but less than 4,
lie in the interval (2006)
(c) 3 (d) 2
(a) –2 < m < 0 (b) m > 3
8. Let two numbers have arithmetic mean 9 and geometric mean
4. Then these numbers are the roots of the quadratic equation (c) –1 < m < 3 (d) 1 < m < 4
(2004) 16. If the difference between the roots of the equation
(a) x2 + 18x – 16 = 0 (b) x2 – 18x + 16 = 0 x2 + ax + 1 = 0 is less than 5 , then the set of possible
(c) x2 + 18x + 16 = 0 (d) x2 – 18x – 16 = 0. values of a is (2007)
(a)  3,  (b)  , 3

(c)  3, 3 (d)  3,  


QUADRATIC
27 EQUATION 27
QUADRATIC EQUATION

17. The quadratic equations 23. If a R and the equation –3 (x – [x])2 + 2 (x – [x]) + a2 = 0
x2 – 6x + a = 0 (where [x] denotes the greatest integer x) has no integral
and 2
x – cx + 6 = 0 solution, then all possible values of a lie in the interval:
(2014)
have one root in common. The other roots of the first and
second equations are integers in the ratio 4 : 3. Then the (a) (–, –2) (2,  (b) (–1, 0) (0, 1)
common root is (2008) (c) (1, 2) (d) (–2, –1)
(a) 2 (b) 1 24. If equations ax2 + bx + c = 0, (a, b  R, a  0) and
(c) 4 (d) 3 2x2 + 3x + 4 = 0 have a common root then a : b : c equals:

18. If the roots of the equation bx2 + cx + a = 0 be imaginary, (2014/Online Set–1)


then for all real values of x, the expression (a) 1 : 2 : 3 (b) 2 : 3 : 4
3b2x2 + 6bcx +2c2 is (2009)
(c) 4 : 3 : 2 (d) 3 : 2 : 1
(a) greater than 4ab (b) less than 4ab
(c) greater than –4ab (d) less than –4ab 1 1
25. If and are the roots of the equation,
 
19. If  and are the roots of the equation x2 – x +1 = 0, then
2009 + 2009 is equal to (2010) ax2 + bx + 1 = 0
(a) –2 (b) –1 (a  0, a, b  R), then the equation,
(c) 1 (d) 2 x (x + b3) + (a3 - 3abx) = 0 has roots:

20. Sachin and Rahul attempted to solve a quadratic equation. (2014/Online Set–1)
Sachin made a mistake in writing down the constant term
and ended up in roots (4, 3). Rahul made a mistake in (a) 3/2 and 3/2 (b) 1/2 and 1/2
writing down coefficient of x to get roots (3, 2). The correct
roots of equation are (2011) (c)  and  (d) 3/2 and 3/2

(a) –4, –3 (b) 6, 1 26. If  and  are roots of the equation,


(c) 4, 3 (d) –6, –1
x2 -4 2 kx + 2e4 ln k - 1 = 0 for some k, and  2  2  66
sin x – sin x
21. The equation e –e – 4 = 0 has (2012)
(a) infinite number of real roots then 3  3 is equal to: (2014/Online Set–2)

(b) no real roots (a) 248 2 (b) 280 2


(c) exactly one real root
(c) 32 2 (d) 280 2
(d) exactly four real roots
22. Let  and  be the roots of equation 27. The sum of the roots of the equation, x2 + |2x - 3| - 4 = 0, is
(2014/Online Set–3)
1 1
px2 + qx + r = 0, p 0. If p, q, r are in A.P. and   4, (a) 2 (b) -2
 
then the value of |–| is : (2014) (c) 2 (d)  2

2 13 61 28. The equation 3x 2  x  5  x  3, where x is real, has:


(a) (b)
9 9 (2014/Online Set–4)
(a) no solution
2 17 34
(c) (d) (b) exactly one solution
9 9
(c) exactly two solutions
(d) exactly four solutions
28 QUADRATIC EQUATION

29. Let  and  be the roots of equation x2 – 6 x –2 = 0. 36. Let p(x) be a quadratic polynomial such that p(0) = 1. If
a10  2a 8 p(x) leaves remainder 4 when divided by x “ 1 and it leaves
If an = n –n, for n1, then the value of is remainder 6 when divided by x + 1; then :
2a 9
(2017/Online Set–1)
equal to: (2015)
(a) p(2) = 11 (b) p(2) = 19
(a) 3 (b) –3
(c) p(–2) = 19 (d) p (–2) = 11
(c) 6 (d) –6
37. The number of real values of  for which the system of
30. If 2 + 3i is one of the roots of the equation linear equations
2x3 – 9x2 + kx – 13 = 0, k  R, then the real root of this
equation (2015/Online Set–1) 2x + 4y – z = 0
(a) exists and is equal to 1 4x + y + 2z = 0
x + 2y + 2z = 0
1
(b) exists and is equal to – has infinitely many solutions, is (2017/Online Set–1)
2
(a) 0 (b) 1
1 (c) 2 (d) 3
(c) exists and is equal to
2 38. The sum of all the real values of x satisfying the equation
2  5x  50)
(d) does not exist 2(x 1) ( x  1 is : (2017/Online Set–2)
31. A value of b for which the equations (a) 16 (b) 14
x2 + bx - 1 = 0, x2 + x + b = 0, have one root in common is (c) –4 (d) –5
(2015/Online Set–2)
39. If   R is such that the sum of the cubes of the roots of
(a)  2 (b) i 3 the equation, x2 +  2 -   x +  10 -   = 0 is minimum, then
(c) i 5 (d) 2 the magnitude of the difference of the roots of this
equation is : (2018/Online Set–1)
32. The sum of all real values of x satisfying the equation
x 2  4 x  60 (a) 4 2 (b) 2 5
x 2
 5x  5  = 1 is : (2016)

(a) –4 (b) 6 (c) 2 7 (d) 20

(c) 5 (d) 3 40. If f(x) is a quadratic expression such that


f(1) + f(2) = 0, and -1 is a root of f(x) = 0, then the other root
33. If b  C and the equations x 2 + bx –1 = 0 and of f(x) = 0 is : (2018/Online Set–2)
x2 + x + b = 0 have a common root different from –1, then
|b| is equal to : (2016/Online Set–1) 5 8
(a) - (b) -
(a) 2 (b) 2 8 5
(c) 3 (d) 3 5 8
(c) (d)
34. If x is a solution of the equation, 2x  1 8 5
 1 41. Let p, q and r be real numbers  p  q,r  0  , such that the
 2x  1  1,  x   , then
 2 4x 2  1 is equal to :
(2016/Online Set–2) 1 1 1
roots of the equation x + p + x + q = r are equal in
3 1
(a) (b) magnitude but opposite in sign, then the sum of squares
4 2
of these roots is equal to : (2018/Online Set–3)
(c) 2 (d) 2 2
35. If, for a positive integer n, the quadratic equation, p2 + q2
(a) (b) p2+ q2
2
x(x + 1) + (x + 1) (x + 2)+….+(x+ n  1 ) (x+n) = 10n
(c) 2(p2+ q2) (d) p2+q2+r2
has two consecutive integral solutions, then n is equal
to: (2017)
(a) 12 (b) 9
(c) 10 (d) 11
QUADRATIC EQUATION 29

EXERCISE - 3 : ADVANCED OBJECTIVE QUESTIONS


1. The number of real values of the triplet (a, b, c) for which
8x 2 16x  51
a cos 2x + b sin2 x + c = 0 is satisfied by all real x, is 7.  3 if x is such that
 2x  3 x  4 
(a) 0 (b) 2
(c) 3 (d) infinite (a) x < – 4 (b) –3 < x < 3/2

2. cos  is a root of the equation 25x2 + 5x – 12 = 0, –1 < x < 0, (c) x > 5/2 (d) all these true

then the value of sin 2 is 8. Let  be the roots of ax2 + bx + c = 0 and ,  be the roots
of px2 + qx + r = 0 ; and D1, D2 the respective Discriminants
(a) 24/25 (b) –12/25
of these equations. If  are in A.P., then D1 : D2
(c) –24/25 (d) 20/25
3. Set of all values of x satisfying the inequality a2 a2
(a) (b)
p2 b2
x 2 – 7x  6 > x + 2 is

b2 c2
(c) (d)
 2 2  q2 r2
(a) x   – ,  (b) x   ,  
 11   11 
9. If 0  x  , then the solution of the equation
(c) x  (–, 1] [6, ) (d) x  [6, ) 2x 2x
16sin + 16cos = 10 is given by x equal to
4. If the product of the roots of the quadratic equation
mx2 – 2x + (2m – 1) = 0 is 3 then the value of m    
(a) , (b) ,
(a) 1 (b) 2 6 3 3 2

(c) –1 (d) 3
 
(c) , (d) none of these
6 2
 x  1 A B
5. If   , then 16A + 9B is
 2x  1 3x  1  2x  1  3x  1 10. Two real numbers  and  are such that +  = 3 and
equal to | – | = 4, then  and  are the roots of the quadratic
(a) 4 (b) 5 equation

(c) 6 (d) 8 (a) 4x2 – 12x – 7 = 0 (b) 4x2 – 12x + 7 = 0


(c) 4x2 – 12x + 25 = 0 (d) none of these
1 1 1
6. If a, b, c R. For the equation   , 11. 2
If (x + 1) is greater then 5x – 1 and less than 7x – 3 then the
xa xb xc
integral value of x is equal to
if the product of roots is zero, then the sum of roots if
(a) 1 (b) 2
b  c  0 is
(c) 3 (d) 4

2ab 12. The value of m for which one of the roots of


(a) 0 (b) x2 – 3x + 2m = 0 is double of one of the roots of
bc
x2 – x + m = 0 is
2bc 2bc (a) 0, 2 (b) 0, –2
(c) (d)
bc bc (c) 2, –2 (d) none of these
30 QUADRATIC EQUATION

13. If and are the roots of the equation, x3 – x – 1 = 0 21. If  are the roots of the equation ax2 + bx + c = 0 and
1  1  1  Sn = n +n, then a Sn + 1 + c Sn – 1 =
then,   has the value equal to
1  1  1  (a) b Sn (b) b2Sn
(c) 2bSn (d) – bSn
(a) zero (b) –1
(c) –7 (d) 1 22. If the roots of equation x2 + bx + ac = 0 are  and roots
of the equation x2 + ax + bc = 0 are  then the value of
14. If the quadratic equations, 3x 2 + ax + 1 = 0 and
 respectively
2x2 + bx + 1 = 0 have a common root, then the value of the
expression 5ab – 2a2 – 3b2 is (a) a, b, c (b) b, c, a

(a) 0 (b) 1 (c) c, a, b (d) none of these


(c) –1 (d) none 23. If the roots of the equation, x3 + Px2 + Qx – 19 = 0 are each
one more than the roots of the equation,
15. If a, b, p, q are non-zero real numbers, the two equations,
2 a2x2 – 2 abx + b2 = 0 and p2x2 + 2 pqx + q2 = 0 have x3–Ax2+Bx–C=0 where A, B, C, P and Q are constants then
the value of A + B + C =
(a) no common root
(a) 18 (b) 19
(b) one common root if 2 a2 + b2 = p2 + q2
(c) 20 (d) none
(c) two common roots if 3 pq = 2 ab
24. If  are the roots of the equation ax2 + bx + c = 0, then
(d) two common roots if 3 qb = 2 ap
the equation ax2 – bx (x – 1) + c (x – 1)2 = 0 has roots
6x 2  5x  3
16. If  4, the least and the highest values of  
x 2  2x  6 (a) , (b)  – 1,  – 1
1–  1– 
4 x2 are
(a) 0 and 81 (b) 9 and 81   1–  1– 
(c) , (d) ,
(c) 36 and 81 (d) none of these  1 1  

17. If p & q are roots of the equation x2 – 2x + A = 0 and r & s 25. x2 + x + 1 is a factor of ax3 + bx2 + cx + d = 0, then the real
be roots of the equation x2 – 18 x + B = 0 if p < q < r < s be
root of above equation is (a, b, c, d  R)
in A.P., then A and B are respectively
(a) –d/a (b) d/a
(a) – 3, 77 (b) 3, 77
(c) (b – a)/a (d) (a – b)/a
(c) 3, – 77 (d) none of these
18. If  are roots of the equation ax2 + 3x + 2 = 0 (a < 0), then 26. If a, b R, a  b . The roots of the quadratic equation,
 is greater than x2 – 2 (a + b) x + 2 (a2 + b2) = 0 are
(a) 0 (b) 1 (a) Rational and different (b) Rational and equal
(c) 2 (d) none of these
(c) Irrational and different (d) Imaginary and different
19. If  be the roots x2 + px – q = 0 and  be the roots of
27. If the quadratic equations ax 2 + 2cx + b = 0 and
         ax2 + 2bx + c = 0 (b  c) have a common root, then
x2 + px + r = 0 then
         a + 4b + 4c is equal to
(a) –2 (b) –1
(a) 1 (b) q
(c) 0 (d) 1
(c) r (d) q + r
28. If the expression x2 – 11x + a and x2 – 14x + 2a must have a
20. If  be roots of x2 + px + 1 = 0 and  are the roots of
common factor and a  0, then, the common factor is
x2 + qx + 1 = 0 then () () () () =
(a) (x – 3) (b) (x – 6)
(a) p2 + q2 (b) p2 – q2
(c) (x – 8) (d) none of these
(c) q2 – p2 (d) none of these
QUADRATIC EQUATION 31

29. Let a > 0, b > 0 and c > 0. Then both the roots of the 37. The equations x 3 + 5x 2 + px + q = 0 and
equation ax2 + bx + c = 0 x3 + 7x2 + px + r = 0 have two roots in common. If the third
(a) are real and negative (b) have negative real parts root of each equation is represented by x 1 and x 2
respectively, then the ordered pair (x1, x2) is
(c) are rational numbers (d) none of these
(a) (–5, –7) (b) (1, –1)
30. If r and s are positive, then roots of the equation
x2 – rx – s = 0 are (c) (–1, 1) (d) (5, 7)
(a) imaginary 38. If (2x – 3x + 1) (2x + 5x + 1) = 9x2, then equation has
2 2

(b) real and both positive (a) four real roots


(c) real and of opposite signs (b) two real and two imaginary roots
(d) real and both negative (c) all imaginary
31. If a, b, c R and roots of the equation ax2 + 2bx + c = 0 are (d) none of the above
real and different, then roots of the equation 39. If (x + 2) (x + 3) (x + 8) (x + 12) = 4x2, then equation has
(a2 + 2b2 – ac) x2 + 2b (a + c) x + (2b2 + c2 – ac) = 0 are (a) no real roots (b) all real roots
(a) real and equal (b) real and unequal (c) can’t be discussed (d) none of these
(c) imaginary (d) none of these
a
32. If p,q,r,s, R and , are roots of the equation 40. If ‘x’ is real and satisfying the inequality, x   a R  ,
x
x2 + px + q = 0 and 4 and 4 are roots of x2 – rx + s = 0, then
the roots of x2 – 4qx + 2q2 – r = 0 are then

(a) both real (b) both positive (a) x  0,  


a for a  0
(c) both negative (d) none of these
33. If the roots of the quadratic equation x2 – 4x – log3a = 0 are  
(b) x   a , 0 for a  0
real, then the least value of a is
(a) 81 (b) 1/81
 
(c) x   a , 0 for a  0
(c) 1/64 (d) none of these
If  are the roots of the equation, x4 – Kx3 + Kx2 +
34.
Lx + M = 0 where K, L and M are real numbers then the

(d) x   a , 
a for a  0

minimum value of 2 +2 +2 +2 is 41. The set of real ‘x’ satisfying, ||x – 1| –1| < 1 is
(a) 0 (b) –1 (a) [0, 2] (b) [–1, 3]
(c) 1 (d) 2 (c) [–1, 1] (d) [1, 3]
35. The value of ‘a’ for which the sum of the squares of the 42. If one root of the equation 4x2 + 2x – 1 = 0 is , then other
roots of the equation x2 – (a – 2) x – a – 1 = 0 assume the root is
least value is
(a) 2 (b) 4 – 3
(a) 0 (b) 1
(c) 4 + 3 (d) none of these
(c) 2 (d) 3
43. If   are the roots of x2 – p (x + 1) – c = 0 then
36. If the two roots of the equation, x3 – px2 + qx – r = 0 are
equal in magnitude but opposite in sign then  2  2  1 2  2  1
 is equal to
 2  2  c 2  2  c
(a) pr = q (b) qr = p
(c) pq = r (d) none (a) 0 (b) 1
(c) 2 (d) none of these
32 QUADRATIC EQUATION

44. If the equation (k – 2) x2 – (k – 4) x – 2 = 0 has difference of 53. If b < 0, then the roots x 1 and x 2 of the equation
roots as 3 then the value of k is
x  x 
(a) 1, 3 (b) 3, 3/2 2x2 + 6x + b = 0, satisfy the condition  1    2  < k
 x 2   x1 
(c) 2, 3/2 (d) 3/2, 1
where k is equal to
45. If the equation sin x – (k + 2) sin2 x – (k + 3) = 0 has a
4

solution then k must lie in the interval (a) – 3 (b) – 5

(a) (–4, –2) (b) [–3, 2) (c) – 6 (d) – 2

(c) (–4, –3) (d) [–3, –2] 2x


54. Consider y  , then the range of expression,
46. x 2
The equation,  = –2x + 6x – 9 has 1 x 2
(a) no solution (b) one solution y2 + y – 2 is
(c) two solutions (d) infinite solutions (a) [–1, 1] (b) [0, 1]
2
47. If both roots of the quadratic equation x + x + p = 0 exceed (c) [–9/4, 0] (d) [–9/4, 1]
p where p  R then p must lie in the interval
55. The least value of expression, x2 + 2xy + 2y2 + 4y + 7 is
(a) (–, 1) (b) (–, –2)
(a) –1 (b) 1
(c) (–, –2) (0, 1/4) (d) (–2, 1)
(c) 3 (d) 7
48. If both roots of the quadratic equation (2 – x) (x + 1) = p are 2
56. If the graph of |y| = f (x), where f (x) = ax + bx + c, b, c  R,
distinct and positive then p must lie in the interval
a 0, has the maximum vertical height 4, then
(a) p > 2 (b) 2 < p < 9/4
(a) a > 0 (b) a < 0
(c) p < –2 (d) –  < p <  2
(c) (b – 4ac) is negative (d) Nothing can be said
49. The quadratic equation, ax2 + bx + c = 0 will always have
57. Set of all possible real values of a such that the inequality
imaginary root if 2
(x – (a – 1)) (x – (a + 2)) < 0 holds for all x  (–1, 3) is
(a) a < –1, 0 < c < 1, b > 0
(a) (1, ) (b) (–, –1]
(b) a < –1, –1 < c < 0, 0 < b < 1
(c) (–, –1) (d) (0, 1)
(c) a < –1, c < 0, b > 1
58. If a(p + q) + 2bpq + c = 0 and a(p + r)2 + 2bpr + c = 0,
2

(d) a < –1, c < –1, 1 < b < 2 (a  0) then


50. If b > a and the equation (x – a) (x – b) + 1 = 0 has real roots
2 c
then (a) qr  p  (b) qr = p2
a
(a) both roots in (a, b)
(c) qr = –p2 (d) None of these
(b) both roots in (–, a)
59. If p(x) = ax + bx and q(x) = lx 2 + mx + n with
2
(c) both roots in (b, )
p(1) = q(1); p(2) – q(2) = 1 and p(3) – q(3) = 4, then
(d) one root in (–, a) and other in (b, ) p(4) – q(4) is
51. If  are the roots of the equation, x2 – 2mx + m2 – 1 = 0 (a) 0 (b) 5
then the range of values of m for which (–2, 4) is
(c) 6 (d) 9
(a) (–1, 3) (b) (1, 3)
60. If x  R, then the maximum value of
(c)  ,  1   3,   (d) none

52. If   are the roots of the quadratic equation,



2 2
y = 2(a – x) x + x  b is 
x2 – 2p (x – 4) – 15 = 0 then the set of values of p for which (a) a2 + b2 (b) a2 – b2
one root is less than 1 & the other root is greater than 2 is
(c) a2 + 2b2 (d) none of these
(a) (7/3, ) (b) (–, 7/3)
(c) x  R (d) none
QUADRATIC EQUATION 33

61. If a, b, c R, a > 0 and c 0 Let  and be the real and 68. Let a, b, c  R. If ax2 + bx + c = 0 has two real roots A and
distinct roots of the equation ax2 + bx + c = |c| and p, q be B where A < –1 and B > 1, then
the real and distinct roots of the equation
ax2 + bx + c = 0. Then (a) 1  b  c  0 (b) 1 – b  c  0
a a a a
(a) p and q lie between  and 
(b) p and q do not lie between  and  (c) |c| < |a| (d) |c| < |a| – |b|
(c) Only p lies between  and  2x

(d) Only q lies between  and 


69. 
5x  2 3  – 169  0 is true in the interval.

62. Let f (x) = ax2 + bx + c; a, b, c  R. If f (x) takes real values (a) (–, 2) (b) (0, 2)
for real values of x and non-real values for non-real values (c) (2, ) (d) (0, 4)
of x, then a satisifes.
70. If a < b < c < d, then for any positive , the quadratic
(a) a > 0 (b) a = 0 equation (x – a) (x – c) + (x – b) (x – d) = 0 has
(c) a < 0 (d) a  R (a) non-real roots
3
63. The value of a for which the equations x + ax + 1 = 0 and (b) one real root between a and c
x4 + ax2 + 1 = 0 have a common root is
(c) one real root between b and d
(a) –2 (b) –1
(d) irrational roots
(c) 1 (d) 2
e e   ee
 log x 2 –  9 / 2  log x  5
 3  3
71. Equation    0 has
64. The equation x  
 3 3 has x –e x – x––e

(a) at least one real solution (a) one real root in (e,) and other in ( – e,e)
(b) exactly three real solutions (b) one real root in (e,) and other in (, + e)
(c) exactly one irrational solution (c) two real roots in (– e, + e)
(d) non real roots (d) No real root
65. For a > 0, the roots of the equation 72. If a < 0, then root of the equation x2 – 2a |x – a| – 3a2 = 0 is
logax a + logx a2 + loga2x a3 = 0, are given by

(a) a – 1 – 6  
(b) a 1 – 2 
(a) a–4/3 (b) a–3/4
(c) a–1/2 (d) a–1
2 2 2

(c) a – 1  6  
(d) a 1  2 
66. The roots of the equation, (x + 1) = x(3x + 4x + 3), are
given by 73. If a, b, c  R and  is a real root of the equation
ax2 + bx + c = 0, and  is the real root of the equation
(a) 2 – 3  
(b) – 1  i 3 / 2 , i  –1
a 2
–ax2 + bx + c = 0, then the equation x + bx + c = 0 has
2
(c) 2  3  
(d) – 1 – i 3 / 2 , i  –1
(a) real roots
67. If 0 < a < b < c, and the roots   of the equation (b) none- real roots
ax2 + bx + c = 0 are non real complex roots, then
(c) has a root lying between  and 
(a) || = || (b) || > 1
(d) None of these
(c) || < 1 (d) none of these
34 QUADRATIC EQUATION

Assertion Reason 78. Assertion : If a > b > c and a3 + b3 + c3 = 3abc, then


(A) If both ASSERTION and REASON are true and the equation ax2 + bx + c = 0 has one
reason is the correct explanation of the assertion. positive and one negative real roots.
(B) If both ASSERTION and REASON are true but reason
Reason : If roots of opposite nature, then
is not the correct explanation of the assertion.
product of roots < 0 and |sums of
(C) If ASSERTION is true but REASON is false.
roots| > 0.
(D) If both ASSERTION and REASON are false.
(E) If ASSERTION is false but REASON is true. (a) A (b) B (c) C (d) D (e) E

74. Assertion : If roots of the equation x2 – bx + c = 0 Using the following passage, solve Q.79 to Q.81
are two consecutive integers, then
b2 – 4c = 1. Passage –1

Reason : If a, b, c are odd integer then the roots In the given figure vertices of  ABC lie on y = f (x)
of the equation 4abc x2 + (b2 – 4ac) = ax2 + bx + c. The ABC is right angled isosceles triangle
x – b = 0 are real and distinct.
whose hypotenuse AC = 4 2 units, then
(a) A (b) B
(c) C (d) D
(e) E
Y
75. Assertion : If one roots is 5 – 2 is then the y = f (x)
= ax2 + bx + c
equation of lowest degree with rational
coefficient is x4 – 14x2 + 9 = 0.
A O C
Reason : For a polynomial equation with X
rational co-efficient irrational roots
B
occurs in pairs.
(a) A (b) B
(c) C (d) D
(e) E 79. y = f (x) is given by
76. Assertion : The set of all real numbers a such that
a2 + 2a, 2a + 3 and a2 + 3a + 8 are the x2 x2
sides of a traiangle is (5, ). (a) y  –2 2 (b) y  –2
2 2 2
Reason : Since in a triangle sum of two sides is
greater than the other and also sides (c) y = x2 – 8
2
(d) y  x – 2 2
are always positive.
(a) A (b) B 80. Minimum value of y = f (x) is
(c) C (d) D
(a) 2 2 (b) – 2 2
(e) E
77. Assertion : The number of roots of the equation (c) 2 (d) – 2
1 x –x
sin (2x) cos (2x) = (2 + 2 ) is 2. k
4 81. Number of integral value of k for which lies between
2
Reason : AM  GM.
the roots of f (x) = 0, is
(a) A (b) B
(c) C (d) D (a) 9 (b) 10
(e) E (c) 11 (d) 12
QUADRATIC EQUATION 35

Using the following passage, solve Q.82 to Q.84 86. Column–I Column–II
Passage –2 (A) Number of real solution of (P) 2
If roots of the equation x4 – 12x3 + bx2 + cx + 81 = 0 are |x + 1| = ex is
positive then (B) The number of non-negative (Q) 3
82. Value of b is real roots of 2x–x–1 = 0 equal to
(a) – 54 (b) 54 (C) If p and q be the roots of the (R) 6
(c) 27 (d) –27
quadratic equation
83. Value of c is
x2 – ( – 2) x –  – 1 = 0, then
(a) 108 (b) –108
minimum value of p2 + q2 is
(c) 54 (d) – 54
equal to
84. Root of equation 2bx + c = 0 is
(D) If  and  are the roots of (S) 5
1 1
(a) – (b) 7
2 2 2x2 + 7x + c = 0 & |2 – 2| = ,
4
(c) 1 (d) –1
then c is equal to
Match the column
Subjective
85. The value of k for which the equation
87. When x100 is divided by x2 – 3x + 2, the remainder is
x3 – 3x + k = 0 has
(2k+1 –1) x – 2(2k – 1) where k is a numerical quantity, then
Column–I Column–II k must be.
(A) three distinct real roots (P) |k| > 2
x
(B) two equal roots (q) k = –2, 2 88. If roots x1 and x2 of x2 + 1 = satisfy
a
(C) exactly one real root (R) |k| < 2
(D) three equal roots (S) no value of k 1 1  1 
x12 – x 22  , then a   – , 0    0 , 
a  2   k

the numerical quantity k must be equal to


89. The integral part of positive value of a for which, the least
value of 4x2 – 4ax + a2 – 2a + 2 on [0, 2] is 3, is
90. If x, y, z are unequal and positive and if x + y + z = 1, the

expression
1 x  1 y  1 z  is greater than
1 – x  1 – y  1 – z 

(The best possible number)


36 QUADRATIC EQUATION

EXERCISE - 4 : PREVIOUS YEAR JEE ADVANCED QUESTIONS


Single Answer Question
2 2
1. Let a > 0, b > 0 and c > 0. Then, both the roots of the 8. The equation x   1 has (1984)
x 1 x 1
equation ax2 + bx + c = 0 (1979)
(a) no root (b) one root
(a) are real and negative
(c) two equal roots (d) infinitely many roots
(b) have negative real parts
9. If log0.3 (x–1) < log0.09 (x–1), then x lies in the interval
(c) have positive real parts
(1985)
(d) None of the above
(a) (2, ) (b) 1, 2)
2. Both the roots of the equation
(c) (–2, –1) (d) None of these
(x–b) (x–c) + (x–a) (x–c) + (x–a) (x–b) = 0
10. If a, b and c are distinct positive numbers, then the
are always (1980) expression (b + c – a) (c + a – b) (a + b – c) – abc is
(a) positive (b) negative (1986)
(c) real (d) None of these (a) positive (b) negative
3. The least value of the expression 2 log10 x–logx (0.01), (c) non-positive (d) non-negative
for x > 1, is (1980)
11. If and are the roots of x + px + q = 0 and 4, 4 are the
2
(a) 10 (b) 2 roots of x 2 –rx + s = 0, then the equation
(c) –0.01 (d) None of these x2 – 4qx + 2q2 – r = 0 has always, if  and are real numbers.
4. The number of real solutions of the equation (1989)
|x|2 –3|x| + 2 = 0 is (1982) (a) two real roots
(a) 4 (b) 1 (b) two positive roots
(c) 3 (d) 2 (c) two negative roots
5. If x1, x2,....., xn are any real numbers and n is any positive (d) one positive and one negative root
integer, then (1982)
3 5
(log x )2  log x
2 2
2 n n 2 12. The equation x 4 4
 2 has (1989)
n
 n   2 
(a) n  x    x i 2
i (b)  x    x i 
i (a) at least one real solution
i 1  i 1  i 1  i 1 
(b) exactly three real solutions
2
n
2  n
 (c) exactly one irrational solution
(c) x
i 1
i  n   xi 
 i 1 
(d) None of these
(d) complex roots
13. Let a,b,c be real numbers, a  0. If  is a root of
6. The largest interval for which x12 – x9 + x4 – x + 1 > 0 is
a2x2 + bx + c = 0, is the root of a2x2 – bx – c = 0 and
(1982) 0 <  < , then the equation a2x2 + 2bx + 2c = 0 has a root 
(a) – 4 < x 0 (b) 0 < x < 1 that always satisfies (1989)
(c) – 100 < x < 100 (d) –< x < 
  
7. If a + b + c = 0, then the quadratic equation (a)   (b)    
2 2
3ax2 + 2bx + c = 0 has (1983)
(c) =  (d) < < 
(a) at least one root in (0, 1)
14. Let f (x) be a quadratic expression which is positive for all real
(b) one root in (2, 3) and the other in (–2, –1) values of x. If g (x) = f(x) + f’(x) + f’’(x), then for any real x
(c) imaginary roots (1990)
(d) None of the above (a) g (x) < 0 (b) g (x) > 0
(c) g (x) = 0 (d) g (x)  0
QUADRATIC EQUATION 37

15. The number log2 7 is (1990) 24. The number of values of k for which the system of
(a) an integer (b) a rational number equations
(c) an irrational number (d) a prime number (k + 1) x + 8y = 4k
16. Let  be the roots of the equation kx + (k + 3) y = 3k – 1
(x – a) (x – b) = c, c  0 has infinitely many solution, is (2002)
Then the roots of the equation (x – ) (x – ) + c = 0 are (a) 0 (b) 1
(1992) (c) 2 (d) infinite
(a) a, c (b) b, c 25. The set of all real numbers x for which
(c) a, b (d) a + c, b + c x2 – |x + 2| + x > 0 is (2002)

17. The equation x 1  x 1  4x 1 has (1997) (a) (–, –2) (2, ) 


(b)   ,  2    2,  
(a) no solution
(b) one solution (c) (–, –1) (1, ) (d)  2,  
(c) two solutions 26. For all ‘x’, x2 + 2ax + (10 – 3a) > 0, then the interval in which
(d) more than two solutions ‘a’ lies is (2004)
(a) a < –5 (b) –5 < a < 2
 P Q
18. In a triangle PQR, R  , if tan   and tan   are (c) a > 5 (d) 2 < a < 5
2 2 2
27. If one root is square of the other root of the equation
the roots of the equation ax2 + bx + c = 0 (a  0), then
x2 + px + q = 0, then the relation between p and q is
(1999)
(2004)
(a) a + b = c (b) b + c = a 3 2 3 2
(a) p – (3p – 1) q + q = 0 (b) p – q (3p + 1) + q = 0
(c) a + c = b (d) b = c
(c) p3 + q (3p – 1) + q2 = 0 (d) p3 + q (3p + 1) + q2 = 0
19. If the roots of the equation x2 – 2ax + a2 + a – 3 = 0 are real
28. If a, b, c are the sides of a triangle ABC such that
and less than 3, then (1999)
x2 – 2 (a + b + c) x + 3 (ab + bc + ca) = 0 has real roots, then
(a) a < 2 (b) 2 < a < 3
(2006)
(c) 3 < a < 4 (d) a > 4
4 5
20. If  and  ( < ) are the roots of the equation (a)   (b)  
x2 + bx + c = 0, where c < 0 < b, then (2000) 3 3
(a) 0 <  <  (b) < 0 <  < ||  4 5 1 5
(c)    ,  (d)    , 
(c)  < < 0 (d)  < 0 ||<   3 3 3 3
21. If b > a, then the equation (x – a) (x – b) – 1 = 0 has
29. Let  be the roots of the equation x2 – px + r = 0 and
(2000) /2, 2 be the roots of the equation x2 – qx + r = 0. Then the
(a) both roots in (a, b) value of r is (2007)
(b) both roots in (–, a) (a) 2/9 (p – q) (2q – p) (b) 2/9 (q – p) (2p – q)
(c) both roots in (b, + ) (c) 2/9 (q – 2p) (2q – p) (d) 2/9 (2p – q) (2q – p)
(d) one root in (–, a) and the other in (b, + ) 30. Let p and q be the real numbers such that p 0, p3  q and
p3  – q. If  and  are non-zero complex numbers
22. For the equation 3x2 + px + 3 = 0, p > 0, if one of the root is
satisfying + = – p and 3 + 3 = q, then a quadratic
square of the other, then p is equal to (2000)
(a) 1/3 (b) 1  
equation having and as its roots is (2010)
(c) 3 (d) 2/3  

23. The number of solutions of log4 (x–1) = log2 (x – 3) is (a) (p3 + q) x2 – (p3 + 2q) x + (p3 + q) = 0
(2001) (b) (p3 + q) x2 – (p3 – 2q) x + (p3 + q) = 0
(a) 3 (b) 1 (c) (p3 – q) x2 – (5p3 – 2q) x + (p3 – q) = 0
(c) 2 (d) 0 (d) (p3 – q) x2 – (5p3 + 2q) x + (p3 – q) = 0
38 QUADRATIC EQUATION

31. Let and be the roots of x2 – 6x – 2 = 0, with > . If 36. Let a, b, c, p, q be the real numbers. Suppose , are the
a10  2a 8 1
an = n – n for n 1, then the value of is roots of the equation x2 + 2px + q = 0 and , are the
2a 9 
(2011) roots of the equation ax 2 + 2bx + c = 0, where
2 {–1,0,1}. (2008)
(a) 1 (b) 2
Assertion : (p2 – q) (b2 – ac)  0
(c) 3 (d) 4
Reason : b pa or c qa.
32. A value of b for which the equations x2 + bx – 1 = 0,
x2 + x + b = 0 have one root in common is (2011) (a) (b) (c) (d)
Passage Q. 37–39
(a)  2 (b) i 3 If a continuous f defined on the real line R, assumes positive
and negative values in R, then the equation
(c) i 5 (d) 2 f(x) = 0 has a root in R. For example, if it is known that a
33. The quadratic equation p(x) = 0 with real coefficients has continuous function f on R is positive at some point and
purely imaginary roots. its minimum values is negative, then the equation f(x) = 0
has a root in R.
Then the equation x
Consider f(x) = ke – x for all real x where k is real constant.
p(p(x)) = 0
(2007)
has (2014) x
37. The line y = x meets y = ke for k  0 at
(a) only purely imaginary roots (a) no point (b) one point
(b) all real roots (c) two points (d) more than two points
(c) two real and two purely imaginary roots 38.
x
The positive value of k for which ke – x = 0 has only one
(d) neither real nor purely imaginary roots root is
34. Let S be the set of all non-zero real numbers  such that 1
the quadratic equation x2 – x +  = 0 has two distinct real (a) (b) 1
e
roots x1 and x2 satisfying the inequality |x1 – x2| < 1. Which
of the following intervals is(are) a subset(s) of S ? (c) e (d) loge 2
x
(2015) 39. For k > 0, the set of all values of k for which ke – x = 0 has
two distinct roots, is
 1 1   1 
(a)   ,   (b)   , 0   1 1 
 2 5  5  (a)  0,  (b)  , 1
 e e 
 1   1 1
(c)  0,  (d)  ,  1 
(c)  ,   (d) (0, 1)
 5  5 2 e 
  Passage Q. 40 to 42
35. Let   . Suppose  and  are the roots of the
6  2 3
2
equation x – 2x sec  + 1 = 0 and  and  are the roots Consider the polynomial f (x) = 1 + 2x + 3x + 4x . Let s be
of the equation x2 + 2x tan  1 = 0. If  and , the sum of all distinct real roots of f (x) and let t = |s|
then   equals. (2016) (2010)
(a) 2 (sec  – tan ) (b) 2 sec  40. The real numbers s lies in the interval
(c) 2 tan  (d) 0 3
 1  
Assertion & Reason (a)   ,0  (b)  11,  
 4   4
(A) If ASSERTION is true, REASON is true, REASON is a
correct explanation for ASSERTION.  3 1  1
(c)   ,   (d)  0, 
(B) If ASSERTION is true, REASON is true, REASON is  4 2  4
not a correct explanation for ASSERTION.
(C) If ASSERTION is true, REASON is false
(D) If ASSERTION is false, REASON is true
QUADRATIC EQUATION 39

41. The area bounded by the curve y = f(x) and the lines True/False
x = 0, y = 0 and x = t, lies in the interval 51. If x – r is a factor of the polynomial
n n–1
3   21 11  f(x) = a n x + a n–1 x + ... + a 0, repeated m times
(a)  ,3  (b)  ,  (1 < m n), then r is a root of f’(x) = 0 repeated m times.
4   64 16 
(1983)
2
 21  52. The equation 2x + 3x + 1 = 0 has an irrational root.
(c) (9, 10) (d)  0, 
 64  (1983)
42. The function f’ (x) is 53. If a < b < c < d, then the roots of the equation
(x–a) (x–c) + 2 (x–b) (x–d) = 0 are real and distinct.
 1  1 
(a) increasing in   t,   and decreasing in   , t  (1984)
 4  4  2 2
54. If P (x) = ax + bx + c and Q(x) = –ax + bx + c, where
ac 0, then P (x) Q (x) has at least two real roots.
 1  1 
(b) decreasing in   t,   and increasing in  ,t (1985)
 4  4 
Subjective Questions
(c) increasing in (–t, t)
(d) decreasing in (–t, t) 26  15 3
55. Show that the square of is a rational
Passage Q. 43 and 44
5 2  (38  5 3)
Let p, q be integers and let ,  be the roots of the
number. (1978)
equation, x2 – x – 1 = 0, where   . For n = 0, 1, 2,……., 2
56. If and are the roots of the equation x + px + 1 = 0;
let an = pn + qn. 2
,  are the roots of x + qx + 1 = 0, then
2 2
FACT : If a and b are rational numbers and a  b 5  0, q – p = (– ) (– ) (+ ) (+ ) (1978)
then a = 0 = b. (2017) 57. Solve 2 logx a + logax a + 3 logb a = 0,
2
43. If a4 = 28, then p + 2q = where a > 0, b = a x (1978)
2
(a) 12 (b) 21 58. If and are the roots of x + px + q = 0 and , are the
2
roots of x + rx + s = 0, then evaluate
(c) 14 (d) 7
(– ) (– ) (– ) (– ) in terms of p, q, r and s.
44. a12 =
(1979)
(a) a11 + 2a10 (b) a11 + a10
59. Show that for any triangle with sides a,b,c;
2
(c) a11 – a10 (d) 2a11 + a10 3(ab + bc + ca) (a + b + c) 4 (ab + bc + ca) (1979)
Fill in the Blanks 60. Find the integral solutions of the following systems of
2
inequalities
45. If 2  i 3 is a root of the equation x + px + q = 0, where p 2
(a) 5x – 1 < (x + 1) < 7x – 3
and q are real, then (p, q) = (.......). (1982)
46. If the products of the roots of the equation x 1 6x 1
2 2 log k (b)  ,  (1979)
x –3kx + 2e – 1 = 0 is 7, then the roots are real for 2x  1 4 4x  1 2
k = ... . (1984)
2
61. For what values of m, does the system of equations
47. If the quadratic equations x + ax + b = 0 and
2 3x + my = m
x + bx + a = 0 (a  b) have a common root, then the numerical
value of a + b is ... . (1986) and 2x – 5y = 20
has solution satisfying the conditions x > 0, y > 0 ?
48. The solution of the equation log7 log5 ( x  5  x )  0
(1980)
is .... (1986) 2
62. If one root of the quadratic equation ax + bx + c = 0 is
49. If , , are the cube roots of p, p < 0, then for any x, y and equal to the nth power of the other, then show that
x  y  z
z, then  ... (1990) 1 1
x  y  z (acn ) n 1  (a n c) n 1  b  0 (1983)

50. The sum of all the real roots of the equation


2
|x–2| + |x–2| – 2 = 0 is .... . (1997)
40 QUADRATIC EQUATION
QUADRATIC EQUATION
2 2
63. Find all real values of x which satisfy x – 3x + 2 > 0 and x 75. Let –1 < p < 1. Show that the equation 4x3 – 3x – p = 0 has
– 2x – 4  0. (1983) a unique root in the interval [1/2, 1] and identify it.
64. If a > 0, b > 0 and c > 0 prove that (2001)
76. Let a, b, c be real numbers with a  0 and let  be the
1 1 1 roots of the equation ax2 + bx + c = 0. Express the roots of
(a  b  c)      9 (1984)
a b c a3x2 + abcx + c3 = 0 in terms of , . (2001)
77. If x2 + (a – b) x + (1 – a – b) = 0 where a, b  R, then find the
x2 3 x 2 3
65. Solve for x (5  2 6)  (5  2 6)  10 (1985) values of a for which equation has unequal real roots for
all values of b. (2003)
66. For a  0, determine all real roots of the equation
2 2 78. If  are the roots of ax2 + bx + c = 0, (a 0) and + ,
x – 2a |x–a| – 3a = 0 (1986)
2
+ , are the roots of Ax2 + Bx + C = 0, (A 0) for some
67. Solve |x + 4x + 3| + 2x + 5 = 0 (1987)
b2  4ac B2  4AC
68. Find the set of all x for which constants , then prove that  (2004)
a2 A2
2x 1
2
 (1987) 79. If x 2 – 10ax – 11b = 0 have roots c & d,
2x  5x  2 x  1 x2 – 10cx – 11d = 0 have roots a and b. (a  c) Find
69. Solve the x the following equation a + b + c + d. (2006)
2
log(2x + 3) (6x + 23x + 21) Integer Answer Type Questions
2
= 4 – log(3x + 7) (4x + 12x + 9) (1987) 80. The smallest value of k, for which both the roots of the
2 2
2 equation x – 8kx + 16 (k – k + 1) = 0 are real, distinct and
70. Let a, b, c be real. If ax + bx + c = 0 has two real roots and
have values at least 4, is.... (2009)
, where < – 1 and > 1, then show that
81. Let (x, y, z) be points with integer coordinates satisfying
c b the system of homogeneous equations
1  0 (1995)
a a 3x – y – z = 0, –3x + z = 0, –3x + 2y + z = 0. Then the number
2 2 2
of such points for which x + y + z  100 is...
71. Let S be a square of unit area. Consider any quadrilateral
which has one vertex on each side of S. If a, b, c and d (2009)
denote the length of the sides of the quadrilateral, prove 2 2
that 2 < a2 + b2 + c2 + d2 < 4 (1997) 82. Let be the complex number cos
 i sin . Then the
3 3
72. Find the set of all solutions of the equation number of distinct complex number z satisfying
2|y| – |2y – 1 – 1| = 2y – 1 + 1 (1997)
z 1  2
73. Let f (x) = Ax2 + Bx + C where, A, B, C are real numbers.
Prove that if f (x) is an integer whenever x is an integer,  z  2 1  0 is equal to..... (2010)
then the numbers 2A, A + B and C are all integers. 2 1 z
Conversely, prove that if the numbers 2A, A + B and C are
all integers, then f (x) is an integer whenever x is an integer. 83. Let a, b, c be three non-zero real numbers such that the
(1998)
  
74.
2
If ,  are the roots of ax + bx + c = 0, (a  0) and equation 3acosx  2bsin x  c, x    ,  , has two
2  2 2
+ , + are the roots of Ax + Bx + C = 0, (A 0) for some
constant , then prove that 
distinct real roots  and  with   . . Then, the value
3
b 2  4ac B2  4AC
 (2000)
a2 A2 b
of is _________. (2018)
a
QUADRATIC EQUATION 41

ANSWER KEY
EXERCISE - 1 : BASIC OBJECTIVE QUESTIONS
1. (b) 2. (c) 3. (d) 4. (d) 5. (c) 6. (d) 7. (b) 8. (b) 9. (d) 10. (a)
11. (a) 12. (c) 13. (a) 14. (a) 15. (b) 16. (b) 17. (b) 18. (a) 19. (c) 20. (d)
21. (b) 22. (a) 23. (d) 24. (c) 25. (b) 26. (c) 27. (c) 28. (d) 29. (a) 30. (b)
31. (b) 32. (c) 33. (b) 34. (d) 35. (b) 36. (b) 37. (b) 38. (a) 39. (b) 40. (a)
41. (b) 42. (d) 43. (a) 44. (a) 45. (a,b) 46. (c) 47. (c) 48. (a) 49. (b) 50. (b,c)
51. (c) 52. (a) 53. (a) 54. (b) 55. (b) 56. (b) 57. (b) 58. (a) 59. (a) 60. (a)
61. (a) 62. (d) 63. (a) 64. (a,b,c,d) 65. (b) 66. (a,b,c,d) 67. (b) 68. (a) 69. (d) 70. (b)
71. (c) 72. (b) 73. (c) 74. (d) 75. (d) 76. (c) 77. (b) 78. (d) 79. (d) 80. (c)

EXERCISE - 2 : PREVIOUS YEAR JEE MAINS QUESTIONS


1. (d) 2. (a) 3. (c) 4. (a) 5. (a) 6. (d) 7. (a) 8. (b) 9. (a) 10. (c)
11. (b) 12. (c) 13. (d) 14. (b) 15. (c) 16. (c) 17. (a) 18. (c) 19. (c) 20. (b)
21. (b) 22. (a) 23. (b) 24. (b) 25. (a) 26. (b) 27. (c) 28. (a) 29. (a) 30. (c)
31. (c) 32. (d) 33. (d) 34. (a) 35. (d) 36. (c) 37. (b) 38. (c) 39. (b) 40. (d)
41. (b)

EXERCISE - 3 : ADVANCED OBJECTIVE QUESTIONS


1. (d) 2. (a,c) 3. (a) 4. (c) 5. (c) 6. (d) 7. (d) 8. (a) 9. (a) 10. (a)
11. (c) 12. (b) 13. (c) 14. (b) 15. (a) 16. (a) 17. (a) 18. (d) 19. (a) 20. (c)
21. (d) 22. (c) 23. (a) 24. (c) 25. (a,d) 26. (d) 27. (c) 28. (c) 29. (b) 30. (c)
31. (c) 32. (d) 33. (b) 34. (b) 35. (b) 36. (c) 37. (a) 38. (a) 39. (b) 40. (a,c)
41. (b) 42. (b) 43. (b) 44. (b) 45. (d) 46. (a) 47. (b) 48. (b) 49. (d) 50. (a)
51. (a) 52. (b) 53. (d) 54. (c) 55. (c) 56. (b) 57. (b) 58. (a) 59. (d) 60. (a)
61. (a) 62. (b) 63. (a) 64. (a,b,c) 65. (a,c) 66. (a,b,c,d) 67. (a,b) 68. (a,b) 69. (a,b) 70. (b,c)
71. (b,c) 72. (b,c) 73. (a,c) 74. (b) 75. (a) 76. (a) 77. (e) 78. (a) 79. (a) 80. (b)
81. (c) 82. (b) 83. (b) 84. (c) 85. A–R; B–Q; C–P; D–S 86. A–Q; B–P; C–S; D–R
87. 0099 88. 0005 89. 0008 90. 0008

EXERCISE - 4 : PREVIOUS YEAR JEE ADVANCED QUESTIONS


1. (b) 2. (c) 3. (d) 4. (a) 5. (d) 6. (d) 7. (a) 8. (a) 9. (a) 10. (b)
11. (a,d) 12. (a,b,c) 13. (d) 14. (b) 15. (c) 16. (c) 17. (a) 18. (a) 19. (a) 20. (b)
21. (d) 22. (c) 23. (b) 24. (b) 25. (b) 26. (b) 27. (a) 28. (a) 29. (d) 30. (b)
31. (c) 32. (b) 33. (d) 34. (a, d) 35. (c) 36. (b) 37. (b) 38. (a) 39. (a) 40. (c)
41. (a) 42. (b) 43. (a) 44. (b) 45. (–4, 7) 46. k = 2 47. –1 48. x = 4 49. w2 50. 4
51. False 52. False 53. True 54. True 57. x = a–1/2 or a–4/3 58. (q – s)2 – rqp – rsp + sp2 + qr2
 15 
60. (a) x = 3 (b) x =  61. m   ,    (30, ) 63. x  [1  5, 1)  [1  5, 2) 65. x   2,  2
 2

66. x  {a (1  2), a ( 6  1)} 67. x  4, (1  3)


 2 1
68. x  (2, 1)    ,  
 3 2
1 1   
69. x   72. y {–1} [1, ) 75. x  cos  cos 1 p  76. x =  
4  3 
77. a > 1 79. 1210 80. k = 2 81. 7 82. 1 83. (0.5)
02
COMPLEX NUMBER
COMPLEX NUMBER 43

COMPLEX NUMBER

4. Division :
1. DEFINITION
z1 a  bi a  bi c  di
  .
z 2 c  di c  di c  di
A number of the form a + ib, where a, b  R and i  1 ,
is called a complex number and is denoted by ‘Z’.  ac  bd   bc  ad 
 2 2 
 2 2 
i
 c d   c d 
z  a  ib
 
Re  z  Im  z 
1. a + ib = c + id
1.1 Conjugate of a Complex Number a=c&b=d
For a given complex number z = a + ib,
 1; r0
its conjugate ‘ z ’ is defined as z = a – ib  i;
 r 1
i 4k  r 
2. ALGEBRA OF COMPLEX NUMBERS 2. 1; r2
i; r3

Let z1 = a + ib and z2 = c + id be two complex numbers 3. a b = a b only if atleast one of either a or b


where a, b, c, d  R and i  1 . is non-negative.
1. Addition :
3. ARGAND PLANE
z1 + z 2 = (a + bi) + (c + di)
= (a + c) + (b + d) i A complex number z = a + ib can be represented by a
unique point P (a, b) in the argand plane.
2. Subtraction :
z1 – z 2 = (a + bi) – (c + di)
= (a – c) + (b – d) i Im(z)

3. Multiplication :
P (a, b)
z1 . z 2 = (a + bi) (c + di) b

= a (c + di) + bi (c + di)
2 a
= ac + adi + bci + bdi Re(z)

= ac – bd + (ad + bc) i
2
( i = –1)
Z = a + ib is represented by a point P (a, b)
44 COMPLEX NUMBER

3.1 Modulus and Argument of Complex Number 3.2 Principal Argument

If z = a + ib is a complex number The argument ‘’ of complex number z = a + ib is called


principal argument of z if – <   .

b
Let tan   , and  be the arg (z).
a
Im(z)
(a, b) P

O (0, 0) Re(z)

(i) Distance of Z from origin is called as modulus of complex

number Z.

It is denoted by r  z  a2  b2

(ii) Here, i.e. angle made by OP with positive direction of 



O 
real axis is called argument of z. 

P (a, b) P (a, b)
arg (z) =  +  arg (z) = 2– 
(iii) (iv)

z1 > z2 or z1 < z2 has no meaning but |z1| > |z2| or |z1| < |z2|

holds meaning. In (iii) and (iv) principal argument is given by – +  and


– respectively.
COMPLEX NUMBER 45

14. amp (z1 . z2) = amp z1 + amp z2 + 2 k; k  I


4. POLAR FORM
 z1 
15. amp   = amp z1  amp z2 + 2 k; k  I
 z2 
P (a, b)
16. amp(zn) = n amp(z) + 2k ; k  I
r r sin 
 6. DE-MOIVRE’S THEOREM
r cos 
Statement : cos n  + i sin n  is the value or one of the
values of (cos + i sin )n according as if ‘n’ is integer or
a rational number. The theorem is very useful in
a = r cos  & b = r sin ; determining the roots of any complex quantity
where r = |z| and  = arg(z)
7. CUBE ROOT OF UNITY
 z = a + ib
3
= r (cos  + isin ) Roots of the equation x = 1 are called cube roots of unity.
3
x – 1=0
2
(x – 1) (x + x + 1) = 0
2
x=1 or x +x+1=0
i
Z = re is known as Euler’s form; where
r =|Z| &  = arg(Z) 1  3i 1  3i
i.e x or x 
 2
   2 

w w2
5. SOME IMPORTANT PROPERTIES

1  i 3 1  i 3
1. (z)  z (i) The cube roots of unity are 1 , , .
2 2
2. z  z  2 Re  z  (ii) W3 = 1

3. z  z  2i Im(z) (iii) If w is one of the imaginary cube roots of unity then


1 + w + w² = 0.
4. z1  z 2  z1  z2 (iv) In general 1 + wr + w2r = 0 ; where r  I but is not the
multiple of 3.
5. z1z 2  z1 z2 (v) In polar form the cube roots of unity are :
6. | z | = 0 z = 0
2 2 4 4
cos 0 + i sin 0 ; cos + i sin , cos + i sin
7. zz | z | 2 3 3 3 3
(vi) The three cube roots of unity when plotted on the argand
z z1 plane constitute the verties of an equilateral triangle.
8. | z1 z 2 |  | z1 | | z 2 | ; 1 
z2 z2 (vii) The following factorisation should be remembered :

9. | z |  | z |  | z | a3  b3 = (a  b) (a  b) (a  ²b) ;
x2 + x + 1 = (x  ) (x  2) ;
10. | z1  z 2 |2  | z1 |2  | z 2 |2  2 Re (z1 z2 )
a3 + b3 = (a + b) (a + b) (a + 2b);
11. | z1 + z2 |  | z1 | + | z2 | (Triangle Inequality)
12. | z1 – z2 | | |z1 | – | z2 || a3 + b3 + c3  3abc = (a + b + c) (a + b + ²c) (a + ²b + c)
2 2 2 2 2 2
13. | az1 – bz2 | + | bz1 + az2 | = (a + b ) (| z1 | + | z2 | )
46 COMPLEX NUMBER

th
8. ‘n’ n ROOTS OF UNITY 9. SQUARE ROOT OF COMPLEX NUMBER
n
Solution of equation x = 1 is given by
Let x + iy = a  ib , Squaring both sides, we get
2k 2k 2
(x + iy) = a + ib
x  cos  i sin ; k = 0, 1, 2, ..., n – 1
n n 2 2
i.e. x – y = a, 2xy = b
 2k 
i
Solving these equations, we get square roots of z.
e n 
 ; k = 0, 1, ....., n – 1
10. LOCI IN COMPLEX PLANE

(i) |z – zo| = a represents circumference of circle, centred


at zo, radius a.
1. We may take any n consecutive integral values of k (ii) |z – zo| < a represents interior of circle
th
to get ‘n’ n roots of unity.
th
(iii) |z – zo| > a represents exterior of this circle.
2. Sum of ‘n’ n roots of unity is zero, n  N
th
(iv) |z – z1| = |z – z2| represents  bisector of segment
3. The points represented by ‘n’ n roots of unity are located with end points z1 & z2.
at the vertices of regular polygon of n sides inscribed in
a unit circle, centred at origin & one vertex being one
z  z1 circle, k  1 
+ve real axis. (v)  k represents :  
z  z2   bisector, k  1
Properties :

If 1 , 1 , 2 , 3 ..... n  1 are the n , nth root of unity (vi) arg (z) =  is a ray starting from origin (excluded)
then : inclined at an with real axis.

(i) They are in G.P. with common ratio ei(2/n) (vii) Circle described on line segment joining z1 & z2 as
diameter is :
p p p 0, if p  k n
(ii) 1p +  1 +  2 + .... + n  1 =  where kZ
 n, if p = k n  z  z1  z  z2    z  z 2  z  z1   0.
(iii) (1  1) (1  2) ...... (1  n  1) = n (viii)Four pts. z1, z2, z3, z4 in anticlockwise order will be
concyclic, if & only if
 0, if n is even
(iv) (1 + 1) (1 + 2) ....... (1 + n  1) = 
1, if n is odd  z  z4   z 2  z3 
  arg.  2   arg  
 z1  z 4   z1  z 3 
-1, if n is even
(v) 1 . 1 . 2 . 3 ......... n  1 = 
 1, if n is odd
 z  z4   z 2  z3 
 arg  2   arg.    2n ;  n  I 
 z1  z 4   z1  z 3 

sin  n 2   n 1   z  z 4   z1  z3  
(i) cos  + cos 2  + cos 3  + ..... + cos n = cos     arg  2     2n
sin   2   2 
 z1  z 4   z 2  z 3  

sin  n 2   n 1 
(ii) sin  + sin 2  + sin 3  + ..... + sin n  = sin   
sin   2   2   z 2  z 4   z1  z 3 
    is real & positive.
 z1  z 4   z 2  z3 
COMPLEX NUMBER 47

11. VECTORIAL REPRESENTATION OF A COMPLEX 12. SOME IMPORTANT RESULTS

Every complex number can be considered as if


(i) If z1 and z2 are two complex numbers, then the distance
it is the position vector of that point. If the point
between z1 and z2 is |z2 – z1|.
P represents the complex number z then,
(ii) Segment Joining points A (z1) and B(z2) is divided by point
  P (z) in the ratio m1 : m2
OP = z &  OP  = z

m1z 2  m2 z1
then z  , m1 and m2 are real.
m1  m2

C (z3) (iii) The equation of the line joining z1 and z2 is given by


Y
(z3-z1) B(z2)

z z 1
(z2-z1)
Q (z’) z1 z1 1  0 (non parametric form)
A(z1)
r z2 z2 1
P(z)
 r

O X Or

z  z1 z  z 2

z  z1 z  z2

(iv) az  az  b  0 represents general form of line.

(v) The general eqn. of circle is :


 
(i) If OP = z = r ei  then OQ = z1 = r ei ( + ) = z . e i. zz  az  az  b  0 (where b is real no.).

 
If OP and OQ are of unequal magnitude then Centre : (–a) & radius | a |2  b  aa  b.
 
OQ  OP e i
(vi) Circle described on line segment joining z1 & z2 as diameter
(ii) If z1, z2, z3, are three vertices of a triangle ABC described
is :
in the counter-clock wise sense, then

z 3  z1 AC AC |z z |  z  z1  z  z2    z  z 2  z  z1   0.
  cos   i sin    .ei  3 1 .ei
z 2  z1 AB AB | z 2  z1 |
48 COMPLEX NUMBER

(vii) Four pts. z1, z2, z3, z4 in anticlockwise order will be Important Identities
concylic, if & only if
2 2
(i) x + x + 1 = (x–) (x– )
 z  z4   z 2  z3 
  arg.  2   arg  
 z1  z 4   z1  z 3  2 2
(ii) x – x + 1 = (x + ) (x +  )

 z  z4   z 2  z3  2 2
(iii) x + xy + y = (x–y) (x–y )
2
 arg  2   arg.    2n ;  n  I 
 z1  z 4   z1  z 3 
2 2 2
(iv) x – xy + y = (x + y) (x + y )
 z  z   z  z  
 arg  2 4   1 3    2n 2 2
 z1  z 4   z 2  z 3   (v) x + y = (x + iy) (x – iy)

3 3 2
(vi) x + y = (x + y) (x + y) (x + y )
 z2  z4   z1  z 3 
    is real & positive.
 z1  z 4   z 2  z3  3 3 2
(vii) x – y = (x – y) (x – y) (x – y )
(viii) If z1, z2, z3 are the vertices of an equilateral triangle
2 2 2 2 2
where z0 is its circumcentre then (viii) x + y + z – xy –yz – zx = (x + y+z ) (x+y + z)

1 1 1
(a)   0 or
2
(x+ y + z) (x + y+ z)
2
z 2  z3 z 3  z1 z1  z 2

2 2
(b) z 12 + z 22 + z 23  z1 z2  z2 z3  z3 z1 = 0 or (x+ y + z ) (x + y + z).

(c) z 12 + z 22 + z 23 = 3 z 20 3 3 3
(ix) x + y + z – 3xyz = (x + y + z) (x+y+ z)
2

2
(ix) If A, B, C & D are four points representing the complex (x + y+z)
numbers z1, z2 , z3 & z4 then

z 4  z3
AB  CD if is purely real ;
z 2  z1

z 4  z3
AB  CD if is purely imaginary ]
z 2  z1

(x) Two points P (z1) and Q(z2) lie on the same side or opposite
side of the line az  az  b accordingly as az1  az1  b and

az 2  az2  b have same sign or opposite sign.


COMPLEX NUMBER 49

SOLVED EXAMPLES

Example – 1
10  10i  5i  5i 2
=
Express the following in the form of a + ib, a, b  R, 25  25i 2

i = 1. State the values of a and b. 15  5i


=
50
i  4  3i  2  i
(i) (ii) 3  i 1  2i
1  i     3 1
=  i
10 10
4i8  3i 9  3
(iii) (iv) (1 + i)6 + (1–i)3 3 1
3i11  4i10  2 here, a  , b
10 10

i  4  3i  4i8  3i 9  3
Sol. (i) z = (iii) z =
1  i  3i11  4i10  2

4i  3i 2 1  i 4.i 4 .i 4  3i5 .i 4 i  3
=  =
1 i 1 i 3.i 4 .i 4 .i3  4i 4 .i4 .i 2  2

 4i  3 1  i  =
4  3i  3
= 2 3i  4  2
1 i

4i  4i 2  3  3i 7  3i 2  3i
= = 
11 2  3i 2  3i

7  i 14  21i  6i  9i 2
 =
2 4  9i 2

7 1 23 15
=   i =  i
2 2 13 13

7 1 23 15
here, a   ,b here, a  , b
2 2 13 13
6 2 3 2 3 3 3
2  i (iv) (1 + i) = {(1 + i) } = (1 + i + 2i) = (1–1 + 2i) = 8i = –8i
(ii) z=
 3  i  1  2i  3 3 2
and (1–i) = 1 – i – 3i + 3i = 1 + i – 3i – 3 = –2 – 2i
6 3
2i Therefore, (1 + i) + (1 – i) = –8i – 2 – 2i = –2 – 10i
=
3  6i  i  2i 2 here, a = – 2, b = – 10

2  i 5  5i
= 
5  5i 5  5i
50 COMPLEX NUMBER

Example – 2 Example – 4

Find the modulus and amplitude of the following complex


a  ib 2 a 2  b2
If x  iy  , prove that  x 2  y2   2 2 numbers.
c  id c d
(i) 3  2i (ii) 1 + i

a  ib
Sol. x + iy = Sol. (i) z = 3  2i here a  3, b  2
c  id
 |z| = a 2  b2
a  ib
 x – iy =
c  id = 3 2
(Taking complex conjugate)
= 5
a  ib a  ib
 (x + iy) (x – iy) =  1  b 
c  id c  id amp (z) = tan  
a

2 2 2 a 2  i2 b2 1
 2
x –i y = = tan  
c2  i2 d 2
 3

2 2 a 2  b2 1
 2
 x +y = = tan  3 
c2  d 2  
(ii) z = 1 + i here a = 1, b = 1
2 2 2 a 2  b2
 (x + y ) = 2
c  d2 |z| = a 2  b2
Example – 3 = 11
Find the value of x3 + x2 – x + 22 if x = 1 + 2i = 2

Sol. x = 1 + 2i 1  b 
amp (z) = tan  
(x – 1)
2
= (2i)
2 a
2
x – 2x + 5 = 0 1  1 
= tan  
3 2 2
Now, x + x – x + 22 = (x – 2x + 5) (x + 3) + 7 1
–1
So, Putting x = 1 + 2i, we get :- = tan 1
3 2
x + x – x + 22 
= (a, b > 0 I quadrant)
=0+7=7 4
COMPLEX NUMBER 51

Example – 5
3 3
y 2  ,i.e., y  
For z = 2 + 3i verify the following : 4 2

2 Hence, the solutions of the given equations are


(i) z = (ii) zz  z
1 3 1 3
(ii)  z  z  is real (iv) z  z  6i 0  i0,1  i0,   i ,  i
2 2 2 2

Sol. z = 2 + 3i Example – 7

z = 2 – 3i Express the following complex numbers in the polar form :


(i) = 2 + 3i
1 i 2  6 3i
Hence, (i) (ii)
zz 1 i 5  3i

(ii) z.  z  = (2 + 3i) (2 – 3i)


1 i
2
Sol. Let z = , and, let r (cos  + i sin ) be the polar form
=4–9i 1 i
of z. Then, r = |z| and  = arg(z).
|z| = 4  9  13

Hence, z. z = |z|2 1  i 1  i  1  i 
Now, z = 1 i  1 i 1 i
  
(iii) z + z = 2 + 3i + 2 – 3i
=4
1  2i  i 2 1  2i  1
=   i  0 1 i
Hence, z + z is a real number.. 1  i2 11
(iv) z – z = (2 + 3i) – (2 – 3i)
 r = |z| = 0 1  1 .
= 2 + 3i – 2 + 3i
Since the point (0, 1) representing z = 0 + i lies on positive
=6i direction of imaginary axis. Therefore,
Hence, z – z is an imaginary number.. arg(z) = /2.
Example – 6
    
Hence, z = 1 cos  i sin   cos  i sin
 2 2  2 2
Solve the equation z 2  z, where z = x + iy

2  6 3i
2 2 (ii) Let z = , and let r (cos  + i sin ) be the polar
Sol. z 2  z x – y + i2xy = x – iy 5  3i
2 2
Therefore, x – y = x ... (1) form of z. Then, r = |z| and  = arg (z)
and 2xy = –y ... (2)
2  6 3i
Now, z 
1 5  3i
From (2), we have y = 0 or x  
2
2
When y = 0, from (1), we get x – x = 0, i.e., x = 0 or x = 1. 2  6 3i 5  3i 
 z = 5  3i .
1 2 1 1
When x   , from (1), we get y   or
5  3i 
2 4 2
52 COMPLEX NUMBER

Example – 9
28  28 3i
=  1 i 3
28 Find the square root of i.

r = |z| = 1  3 = 2.
Sol. Let i  x  iy . Then,
Let  be the smallest positive angle given by
i  x  iy
 Im  z  
tan –1    tan
1
 
3 /1  tan 1  
3 .
 i = (x + iy)
2
  
Re z
2 2
 (x – y ) + 2i xy = 0 + i
2 2
  x –y =0 . ..(i)
Then,  
3
 
. Since the point 1, 3 lies in first quadrant.
and, 2xy = 1 ...(ii)
2 2 2 2 2 2 2 2
Now, (x + y ) = (x – y ) + 4x y

  . 
2 2 2
(x + y ) = 0 + 1 = 1
3
2 2 2 2
 x + y = 1 [ x + y > 0] ...(iii)
   Solving (i) and (iii), we get
Hence, the polar form of z is z = 2  cos  i sin 
 3 3 2 2
x = 1/2 and y = 1/2
Example – 8  x=±1 2 and y = ± 1/ 2

Find the square roots of –15 – 8i. From (ii), we find that 2xy is positive. So, x and y are of
same sign.

Sol. Let 15  8i  x  iy . Then,  1 1   1 1 


 x  and y   or  x   and y   
 2 2  2 2
15  8i  x  iy
 –15 – 8i = (x + iy)
2  1 1  1
Hence, i   1  i 
2 2  2 2i  2
 –15 – 8i = (x – y ) + 2i xy ...(i)
2 2
 –15 = x – y ...(ii) Example – 10
and, 2xy = –8
2 2 2 2 2 2 2 2 Find the value of :
Now, (x + y ) = (x – y ) + 4x y
2 2 2 2 r 6
 (x + y ) = (– 15) + 64 = 289  2r 2r 
2 2
 sin 7
 i cos
7 
 x + y = 17 ...(iii) r 1

On solving (i) and (iii), we get


2 2
x = 1 and y = 16  x = ± 1 and y = ± 4 r 6
 2r 2r 
From (ii), 2xy is negative. So, x and y are of opposite signs.
Sol. Let S =  1.sin
r 1 7
 i cos
7 
 (x = 1 and y = –4) or, (x = –1 and y = 4)

Hence, 15  8i   1  4i   r  6 i 24 r 


=   e  eio  e  io 
 r 1 
r 6
 2r 2r 
   i 2 sin  i cos [Add and substract]
r 1  7 7 
COMPLEX NUMBER 53

Take (–i) common to get : Example – 12

 r 6 i 2  r 
 i   e 7  1 z 1
 r 0  (i) If | z | = 1, prove that  z  1 is a purely
z 1
r 6 imaginary number.
 2r 2r 
  cos  isin
r 1  7 7 
z 1
= – i (sum of 7th roots of unity – 1) (ii) If the number is purely imaginary, then prove
z 1
= – i (0 – 1) = i that | z | = 1.
r 6 2r
i
 i  e 7

r 1
z 1
Sol. (i) Let w
Example – 11 z 1

If  and  are roots of x2 – 2x + 4 = 0 then find n + n.  z 1   z 1 


  
w  w  z 1  z 1
Re  w   
Sol. ,   1  i 3 2 2

  1  i 3,   1  i 3  1 
1
1  z  1 z  1 1  z  1 z 
     
n 2  z  1 z  1  2  z  1 1 
n  1  i 3   
1
z 

n
1 i 3 
n
 2  
 2  1  z  1 1  2  1  z 1 z 1
=    0
2  z  1 1  2  2  z  1 z  1 
m
1 i   Re (w) = 0
   1,
1 i   w is a pure imaginary number.

n
z 1
n  i  n
(ii) w
| 3  2 e  . z 1
 3
As w is purely imaginary.
Re (w) = 0
n
 n  n  2n .2 cos z 1 z 1
3 
ww z 1 z 1  0
 0 
2 2
 n 
 2n 1 cos  
 3  z 1 z 1 z 1 1 z
   
z 1 z 1 z 1 1 z
Apply C and D to get :
1
z  zz  1  | z 2 | 1  | z | 1.
z
54 COMPLEX NUMBER

Example – 13 Example – 15

If iz3 + z2 – z + i = 0, then show |z| = 1. ABCD is a rhombus. Its diagonals AC and BD intersect
at the point M and satisfy BD = 2AC. Its points D and M
Sol. iz3 + z2 – z + i = 0 represent the complex numbers l + i and 2 – i respectively.
Find the complex number represented by A.
By substituting z = i in the equaiton, we get 0 = 0

Hence z – i is a factor of iz3 + z2 – z + i Sol. Let A be (x, y)

 iz2 (z – i) – 1 (z – i) = 0  (iz2 – 1) (z – i) = 0 It is given that BD = 2AC  MD = 2AM

Either iz2 – 1 = 0 or z – i = 0 Also DM is perpendicular to AM


2 2
When z – i = 0, z = i  | z || 0  i.1 | 0  1  1.  (1 – 2)2 + (1 + 1)2 = 4 [(x – 2)2 + (y + 1)2] ...(1)

When iz2 – 1 = 0, z2 1/i = – i


2
| z 2 || 0  1.i | 02   1  1 | z 2 | 1 or | z | 1

In any case we have |z| = 1

Example – 14

It is given that n is an odd integer greater than 3, but not


a multiple of 3. Prove that x3 + x2 + x is a factor of
(x + 1)n – xn – 1.
y 1 11
and .  1  2  y  1  x  2
Sol. We have x3 + x2 +x = x (x2 + x + 1) = x (x – ) (x – 2). x  2 1 2
where , 2 are cube roots of unity but not equal to 1.
With x – 2 = 2(y + 1), (1) gives (y + 1)2 = 1/4
Moreover, 3 = 1.
y = –1/2, –3/2 x = 3, 1
x3 + x2 + x is a factor of (x + 1)n – xn –1. It means that
(x + 1)n – xn – 1 should be zero at x = 0, = , x = 2. A represent z = 3 –i/2, or 1–3i/2
At x = 0, Alternative Solution :
(x + 1)n – xn – 1 = 1n –1 = 1 – 1 = 0 MD = 2AM and AM  DM i.e. AMD = /2
At x = 
z   2  i AM  i2
n n n n 2 n
(x + 1) –x – 1 = (1 + ) –  – 1 = (– ) –  – 1 n   .e   i / 2
1  i    2  i  MD
n 2n n 2n n
= (–1)  –  – 1 = –[ +  + 1] = 0 as n is not a
multiple of 3.
i
 z  2  i    1  2i   z  3  i / 2 or 1  3i / 2
At x = 2, 2

(x + 1)n – xn – 1 = (1 + 2)n – 2n – 1

= (–)n – 2n – 1 = – [n + 2n + 1] = 0

 x3 + x2 + x is a factor of (x + 1)n – xn – 1.
COMPLEX NUMBER 55

Example – 16 Example – 18

Show that the triangle whose vertices are the points Find the locus of point z satisfying the conditioin
represented by the complex numbers z1, z2, z3 on the
zi
argand diagram is equilateral if and only if  2.
z i
z12  z 22  z 32  z1z 2  z 2 z 3  z 3 z1 .

2
Sol. Applyiung rotation about B, Sol. z i
 4  | z  i |2  4| z  i |2
zi
z1  z 2
 e i / 3 ...(1)
z3  z 2
  z  i  z  i   4  z  i  z  i 
Applying rotation about C,
 z z  i  z  z   1  4z z  4i  z  z   4
z 2  z3
 e i / 3 ...(2)
z1  z 3
 3 | z |2 5i  z  z   3  0

 3x 2  3y 2  10y  3  0.
Which represents the interior and boundary of the circle.

Example – 19

Show that the representive points of the complex numbers


z1  z 2 z 2  z 3 i, –2 – 5i, 1 + 4i and 3 + 10i are collinear.
From (1) and (2), 
z 3  z 2 z1  z 3
Sol. Let Cartesian coordinates of these points be a (0, 1), B
Simplifying, we get the required Conditions
(–2, – 5), C (1, 4) and D (3, 10)
Example – 17
5  1
y 1   x  0
Let z1, z2, z3 be three distinct complex numbers satisfying 2  0
|z1 = 1| = |z2 –1| = |z3 – 1|. Let A, B, and C be the points
 y = 3x + 1 ...(1)
represented in the argand plane corresponding to z1, z2
and z3 respectively. Prove that z1 + z2 + z3 = 3 if and only Points C (1, 4) and D (3, 10) satisfy the equatioin (1).
of ABC is an equilateral triangle. Hence points A, B, C and D are collinear.

Example – 20
Sol. |z1 – 1| = |z2 – 1| = |z3 – 1|
 The point corresponding to 1 (say P) is equidistant from If A and B represent the complex number z1 and z2 such
the points A, B and C. that |z1 + z2| = |z1 – z2|, then find the circumcentre of
 P is the circumcentre of the ABC triangle OAB where O is the origin.
Now if z1 + z2 + z3 = 3 then the point corresponding to
z  z  z3 Sol. |z1 + z2|2 = |z1–z2|2
centroid of the ABC is 1 2 1
3 z  
arg  1   
 circumcentre and centroid coincide  ABC is equilateral z 2
 2
Conversely if  ABC is equilateral, then centroid is the
same as the circumcentre i.e. P. Hence centroid OAB is a right angled triangle right angle at O.

z1  z 2  z3 z1  z 2
 1  z1  z 2  z3  3 circumcentre is
3 2
56 COMPLEX NUMBER

Example – 21 (iv) z2 = – z or (x + iy)2 = – (x – iy)

Prove that : x4 + 4 = (x + 1 + i) (x + 1 – i) (x – 1 + i) or x2 – y2 + 2ixy = – x + iy


(x – 1 – i). Equating real and imaginary parts, we get
x2 – y2 = – x ...(i)
Sol. Consider R.H.S.
and 2xy = y or y (2x – 1) = 0 ...(ii)
= [(x + 1 + i) (x + 1 – i)] [(x – 1 + i) (x – 1 – i)]
From (ii) either = 0 or 2x – 1 = 0
= [(x2 + 2) + 2x] [(x2 + 2) – 2x]
i.e., x = 1/2.
= [(x + 1)2 – i2] [(x – 1)2 – i2)] = (x2 + 2)2 – (2x)2
When y = 0, (i) gives x2 = – x or x (x + 1) = 0 which gives
2 2
= (x + 2x + 1 + 1) (x – 2x + 1 + 1) x = 0, – 1.
4 2 2 4
= x + 4x + 4 – 4x = x = x + 4 = L.H.S. Thus we get two sets of solution x = 0, y = 0 and x = –1, y = 0.
Example – 22 When x = 1/2, (i) gives y2 = 3/4 which gives
y = + 3/2.
Find all complex numbers z which satisfy the following
equations : Thus we get two more sets of solutions
x = 1/2, y = 3/2, and x = 1/2, y = – 3/2.
(i) z  z (ii) z   z
Hence the given equation has in all the following four
2
(iii) z  4  z (iv) z   z solutions : z1 = 0 + i0, z2 = – 1 + 0i = – 1,
z3 = 1/2 + i (3/2), z4 = 1/2 – (3/2)
Sol. Let z = x + iy. Then z = – iy..
Example – 23
(i) The equation z  z becomes x + iy = x – iy..
or 2iy = 0 which gives y = 0. 
What is the locus of z, if amplitude of z – 2 – 3i is ?
Hence z = x i.e., all the real numbers constitute the solutions 4
of the given equation.

(ii) The equation z   z is equivalent to Sol. Let z = x + iy. Then z – 2 – 3i = (x – 2) + i (y – 3)

x + iy = – (x – iy) or 2x = 0 or x = 0. y3
Let be the amplitude of z – 2 – 3i. Then tan  
Hence z = iy i.e., the solutions of the given equation are all x2
pure imaginary numbers.
 y3 
(iii) z = 4 – z or x – iy = 4 – x – iy or x = 4 – x.  tan   sin ce   
4 x 2 4
This gives x = 2.
Hence z = 2 + iy. y3
 1 i.e. x – y + 1 = 0
 The given equation is satisfied by all complex numbers x2
whose real part is 2.
Hence, the locus of z is a straight line.
COMPLEX NUMBER 57

Example – 24 Example – 25

Find all circles which are orthogonal to | z | = 1 and Locate the complex number z = x + iy for which
|z –1|=4.
(i) z 2  z 2  2 | z |2  8i (z  z)

Sol. | z | = 1; |z – 1| = 4 (ii) log1/3 {log1/2 (|z|2 + 4|z| + 3)} < 0


 x2 + y2 = 1 and (x – 1)2 + y2 = 4
S1 : x2 + y2 = 1; S2 : x2 + y2 – 2x – 15 = 0 and Sol. (i) z 2  z 2  2 | z |2  8i (z  z)

S : x2 + y2 +2gx + 2fy + c = 0 Substitute z = x + iy


Using the condition of orthogonality of S and S1, we have x2 – y2 + i (2xy) + x2 – y2 – i (2xy) + 2 (x2 + y2) < 8i (–2iy)
2g(0) + 2f(0) = c – 1 c = 1  4x2 < 16y x2 < 4y

Similarly, using the condition of orthogonality of S and


S2, we have
2g (–1) + 2f (0) = 1 – 15 g = 7

radius = g 2  f 2  c  48  f 2

So, all the circles orthogonal to S1 and S2 are :

z  (7  if )  48  f 2 ; f R
x2
So, locus is the interior of the parabola y 
4

(ii) log1/3 {log1/2 (| z |2 + 4| z | + 3)} < 0


 log1/2 (| z |2 + 4| z | + 3) > 1

1
 | z |2  4 | z | 3 
2

5
 | z |2 4 | z |   0
2

 4  6 4  6 
 | z | ,  ; But | z | 0
 2 2 

 So, no such value of z exists.


58 COMPLEX NUMBER

EXERCISE - 1 : BASIC OBJECTIVE QUESTIONS


Algebra of Complex Numbers
1  b  ia
1. The roots of the equation x4 – 1 = 0, are : 8. If a2 + b2 = 1, then is equal to :
1  b  ia
(a) 1, 1, i, – i (b) 1, –1, i, – i
(a) 1 (b) 2
(c) 1, –1, , 2 (d) none of these
(c) b + ia (d) a + ib
2. The value of (1 + i) (1 + i2) (1 + i3) (1 + i4) is
1
(a) 2 (b) 0     
9. The real part of 1  cos    i sin    is
 5
   5 
(c) 1 (d) i
3. The value of 1 + i + i + i + ... + i2n is :
2 4 6
1
(a) positive (b) negative (a) 1 (b)
2
(c) zero (d) cannot be determined
1  1 
(c) cos   (d) cos  
4. ( 2) ( 3) is equal to 2  10  2 5

(a) 6 (b)  6
Representation, Modulus &Argument of Complex Numbers
(c) i 6 (d) none of these
1  2i
10. In which quadrant of the complex plane, the point
1 i
4
 1 lies ?
5. (1  i) 4   1   
 i (a) Fourth (b) First
(c) Second (d) Third
(a) 16 (b) 0
(c) 8 (d) 64 1  i 4i
11. The modulus of  is
3i 5
n
 2i 
6. The least positive integer n such that   is a positive 11
1 i  (a) 5 unit (b)
5
unit
integer, is
(a) 16 (b) 8 5 12
(c) unit (d) unit
(c) 4 (d) 2 5 5

7. Inequality a + ib > c + id can be explained only when :   


12. The amplitude of sin  i  1  cos  is
(a) b = 0, c = 0 (b) b = 0, d = 0 5  5
(c) a = 0, c = 0 (d) a = 0, d = 0
2 
(a) (b)
5 15

 
(c) (d)
10 5
COMPLEX NUMBER 59

Eulers form & De-Moivre’s Theorem


    
13. If z1 = 2  cos  i sin  and z2 = 3  cos  i sin  , then
 4 4  3 3 4(cos 75   i sin 75  )
19. The value of is :
|z1 z2| is 0.4 (cos 30   i sin 30  )

(a) 6 (b) 2 2 2
(a) (1  i) (b) (1  i)
10 10
(c) 6 (d) 3
10 10
25 3 (c) (1  i) (d) (1  i)
14. The polar form of (i ) is 2 2
  20. The principal amplitude of (sin 40o + i cos 40o)5 is
(a) cos  isin (b) cos  + i sin 
2 2
(a) 70o (b) –110o
  (c) 110o (d) –70o
(c) cos – i sin  (d) cos  isin
2 2
cos   i sin   
 1  3i  21. Let z  ,    . Then, arg z is
15. Argument of the complex number   cos   i sin  4 2
 2i 

(a) 45o (b) 135o (a) 2 (b) 2– 


o o
(c) 225 (d) 240 (c) + 2 (d) None of these
1  2i
16. If z  , then arg (z) equals
1  (1  i) 2 3i
22. If z  , then the fundamental amplitude of z is
3 i

(a) 0 (b)
2
 
(c)  (d) none of these (a)  (b)
3 3
17. Principal value of amplitude of (1 + i) is

  
(a) (b) (c) (d) None of these
4 12 6

3 23. If ei = cos  + i sin , then for the ABC, eiA . eiB . eiC is
(c) (d) 
4
(a) – i (b) 1
2
18. If z  , then the value of arg(z) is (c) – 1 (d) None of these
1 i 3
24. If z = cos  + i sin , then

(a)  (b)
3 1 n 1
(a) z n   2 cos n (b) z   2n cos n
zn zn
2 
(c) (d)
3 4
1 n 1
(c) z n   2 n i sin n (d) z   (2i) n sin n
zn zn
60 COMPLEX NUMBER

Properties
1
31. If z  , then | z | =
25. If |z 1 | = |z 2 | = ... = |z n| = 1, then the value of (2  3i) 2
|z1 + z2 + z3 + ... + zn | is:
(a) 1 (b) |z1| + |z2| + ... + |zn| 1 1
(a) (b)
13 5
1 1 1
(c) z  z  ...  z (d) none of these 1
1 2 n (c) (d) none of these
12

26. For any two complex numbers z1 and z2 and any real
1
numbers a and b; |(az1 – bz2)|2 + |(bz1 + az2)|2 is equal to : 32. If z  , then | z | =
(1  i) (2  3i)
(a) (a2 + b2) (|z1| + |z2|) (b) (a2 + b2) (|z1|2 + |z2|2)
(c) (a2 + b2) (|z1|2 – |z2|2) (d) none of the above (a) 1 (b) 1/ 26

(c) 5 / 26 (d) none of these


1
27. The conjugate of a complex number is . Then, that
i 1 33. If x + iy = (1 + i) (1 + 2i) (1 + 3i), then x2 + y2 =
complex number is (a) 0 (b) 1
(c) 100 (d) none of these
1 1
(a) (b) 
i 1 i 1 34. The value of amp (i) + amp (i2), where i  1 and

1 1   3 1 = non-real, is
(c) (d) 
i 1 i 1

(a) 0 (b)
2
1  2i
28. If (x + iy) = , then (x2 + y2)2 is equal to (c)  (d) None of these
3  4i
35. If z1, z2, z3 are complex numbers such that
(a) 5 (b) 1/5
(c) 2/5 (d) 5/2 1 1 1
|z1| = |z2| = |z3| =    1, then |z1 + z2 + z3| is
z1 z 2 z3
29. If a  ib = x + iy, then possible value of a  ib is
(a) equal to 1 (b) less than 1
(a) x2 + y2 (b) x 2  y2 (c) greater than 3 (d) equal to 3
(c) x + iy (d) x – iy
(1  i 3) (2  2i)
36. The magnitude and amplitude of are
| z |2 ( 3  i)
30. If z is a non-zero complex number, then is equal to
zz respectively

3 3
z (a) 2, (b) 2 2,
(a) (b) | z | 4 4
z
 
(c) | z | (d) none of these (c) 2 2, (d) 2 2,
4 2
COMPLEX NUMBER 61

37. If arg (z) = , then arg ( z ) is equal to 43. If is a non-real cube root of unity, then the expression
(1–) (1–2) (1+4) (1+8) is equal to
(a) –  (b) – 
(c)  (d) – (a) 0 (b) 3
(c) 1 (d) 2
38. Let z, w be complex numbers such that z  iw  0 and
44. If is a cube root of unity, then
arg (zw) = . Then, arg (z) equals
(3 + 5+ 32)2 + (3 + 3+ 52)2 is equal to
  (a) 4 (b) 0
(a) (b)
4 2
(c) –4 (d) None of these
3 5 5 5
(c) (d)  3 i  3 i
4 4 45. If z         , then
39. If z and w are two non-zero complex numbers such that  2 2  2 2

 (a) Re (z) = 0
|zw| = 1 and arg (z) – arg (w)  , then z w is equal to
2 (b) 1m (z) = 0
(a) 1 (b) –1 (c) Re (z) > 0, 1m (z) > 0
(c) i (d) –i (d) Re (z) > 0, 1m (z) < 0
Cube Root of Unity 1
46. If z is any complex number such that z   1, then the
40. If the cube roots of unity are 1, , 2, then the roots of the z
equation (x – 1)3 + 8 = 0, are:
99 1
(a) – 1, 1 + 2, 1 + 22 (b) –1, 1 – 2, 1 – 22 value of z  is
z 99
(c) –1, –1, –1 (d) –1, –1 + 2, –1 – 22
(a) 1 (b) –1
41. One root of (1)1/3 is
(c) 2 (d) –2
3i 1 3 i Square Root
(a) (b)
2 2 47. If square root of –7 + 24i is x + iy, then x is
(a) + 1 (b) + 2
1  3i 1  3i
(c) (d) (c) + 3 (d) + 4
4 2
48. If x  iy  ± (a + ib), then x  iy is equal to
42. If i  1, then
(a) ± (b + ia) (b) ± (a – ib)
334 365
 1 3 1 3 (c) (ai + b) (d) ± (b – ia)
4  5    i   3   i 
 2 2  2 2  49. Square root of 5 + 12i is
(a) ± (3 + 2i) (b) ± (3–2i)
is equal to
(c) 2 + 4i (d) –1 –2i
(a) 1  i 3 (b) 1  i 3
50. Square root of –15 –8i is
(c) 4 3i (d) i 3 (a) ± (1 + 4i) (b) ± (1–4i)
(c) ± (2 + i) (d) None
62 COMPLEX NUMBER

EXERCISE - 2 : PREVIOUS YEAR JEE MAINS QUESTIONS


1. z and  are two non zero complex number such that 8. If z1 and z2 are two non-zero complex numbers such that
|z| = || and Arg z + Arg  = , then z equals (2002) |z1 + z2| = |z1| + |z2|, then arg z1 – arg z2 is equal to
(2005)
(a)  (b) – 
(a) – (b) /2
(c)  (d) –
(c) –/2 (d) 0
2. If |z – 4| < |z – 2|, its solution is given by (2002)
(a) Re(z) > 0 (b) Re(z) < 0 10
 2k 2k 
9. The value of   sin  i cos  is (2006)
(c) Re(z) > 3 (d) Re(z) > 2 k 1  11 11 

x (a) i (b) 1
1 i 
3. If    1, then (2003) (c) –1 (d) –i
 1 i 
2
10. If z + z + 1 = 0, where z is a complex number, then the value
(a) x = 2n, where n is any positive integer
2 2 2 2
(b) x = 4n + 1, where n is any positive integer  1  2 1   3 1   6 1
of  z     z  2    z  3   ...   z  6  is
 z z  z  z  
(c) x = 2n + 1, where n is any positive integer
(d) x = 4n, where n is any positive (2006)
4. If z and  are two non-zero complex numbers such that |z| (a) 18 (b) 54

 (c) 6 (d) 12
= 1, and Arg (z) – Arg()  , then z is equal to 11. If |z + 4| < 3, then the maximum value of |z + 1| is
2
(2003) (2007)
(a) –1 (b) i (a) 6 (b) 0
(c) –i (d) 1 (c) 4 (d) 10
5. Let z1 and z2 be two roots of the equation z2 + az + b = 0, z 1
being complex further, assume that the origin, z1 and z2 12. The conjugate of a complex number is . Then that
i 1
from an equilateral triangle, then (2003)
complex number is (2008)
(a) a2 = 2b (b) a2 = 3b
(c) c2 = ab (d) a2 = b 1 1
(a) (b)
i 1 i 1
x y
   1 1
p q
6. If z = x – iy and z = p + iq, then  2
1/3
equal to (c)
i 1
(d)
i 1
 p  q2 
(2004) 4
13. If z   2, then the maximum value of | z | is equal to
(a) 2 (b) –1 z
(c) 1 (d) –2 (2009)
7. If |z2 – 1| = |z|2 + 1, then z lies on (2004) (a) 3 1 (b) 5 1
(a) a circle (b) the imaginary axis
(c) 2 (d) 2  2
(c) the real axis (d) an ellipse
14. The number of complex numbers z such that
| z – 1| = | z + 1 | = | z – i | equals (2010)
(a) 0 (b) 1
(c) 2 (d) 
COMPLEX NUMBER 63

15. Let ,  be real and z be a complex number. If


z i
z2 + z + =0 has two distinct roots on the line Re z = 1, 21. Let z  - i be any complex number such that is a
then it is necessary that (2011) zi

(a) (–1, 0) (b) | | = 1 1


purely imaginary number then z  is:
(c) (1, ) (d) (0, 1) z
16. If (1) is a cube root of unity and (1 + )7 = A + B. Then, (2014/Online Set–3)
(A, B) equals to (2011) (a) 0
(a) (1, 1) (b) (1, 0) (b) any non-zero real number other than 1.
(c) (–1, 1) (d) (0, 1) (c) any non-zero real number.
17. If z is a complex number of unit modulus and argument , (d) a purely imaginary number.
 1 z  22. For all complex numbers z of the form 1 + i,   R, if
then arg   is equal to (2013)
 1 z  z2 = x + iy, then: (2014/Online Set–4)
(a) y2 - 4x + 2 = 0 (b) y2 + 4x - 4 = 0

(a) – (b)   (c) y2 - 4x + 4 = 0 (d) y2 + 4x + 2 = 0
2
23. A complex number z is said to be unimodular if |z| = 1
(c)  (d) – 
Suppose z 1 and z 2 are complex numbers such that
18. If z is a complex number such that
z1  2z 2
1
|z|  2, then the minimum value of z  : (2014) 2  z1 z 2 is unimodular and z2 is not unimodular. Then
2
the point z1 lies on a: (2015)
3 5 (a) circle of radius 2
(a) is strictly greater than but less than
2 2 (b) circle of radius 2
5 (c) straight line parallel to x-axis
(b) is equal to
2 (d) straight line parallel to y-axis
(c) lies in the interval (1, 2) 24. The largest value of r for which the region represented by
the set {  C/|–4–i| r} is contained in the region
5 represented by the set {z  C/|z –1|  |z + i|}, is equal to:
(d) is strictly greater than
2 (2015/Online Set–1)
19. Let w (lm w  0) be a complex number. Then the set of all
3
complex numbers z satisfying the equation (a) 2 (b) 17
2
w  wz  k 1  z  for some real number k, is:
5
(2014/Online Set–1) (c) 2 2 (d) 2
2
(a) {z : |z| = 1} (b) {z : z = z }
25. For all complex numbers z1, z2 satisfying z1  12 and
(c) {z : z  1} (d) {z : |z| = 1, z  1}
20. If z1, z2 and z3, z4 are 2 pairs of complex onjugate numbers, z 2  3  4i  5, the minimum value of z1  z 2 is

 z1   z2  (2015/Online Set–2)
then arg  z   arg  z  equals: (2014/Online Set–2) (a) 0 (b) 2
 4  3
(c) 7 (d) 17

(a) 0 (b)
2

3
(c) (d) 
2
64 COMPLEX NUMBER

26. A value of  for which 2  3i sin  is purely imaginary,,  iz  2 


1  2i sin  30. The equation 1m    1  0, z  C, z  i represents
 zi 
is : (2016) a part of a circle having radius equal to :
 1  3 (2017/Online Set–2)
(a) (b) sin  4  (a) 2 (b) 1
6  

1  1   3 1
(c) sin  3  (d) (c) (d)
  3 4 2
27. The point represented by 2 + i in the Argand plane moves 31. If   C are the distinct roots, of the equation
1 unit eastwards, then 2 units northwards and finally from
x 2  x  1  0, then 101  107 is equal to : (2018)
there 2 2 units in the south-westwards direction. Then
its new position in the Argand plane is at the point (a) 2 (b) -1
represented by: (2016/Online Set–1) (c) 0 (d) 1
(a) 2 + 2i (b) 1 + i
1 +  1 - 8  z
(c) –1 –i (d) –2 –2i 32. The set of all R , for which w = is a
1-z
28. Let z = 1 + ai be a complex number, a > 0, such that z3 is a
real number. Then the sum 1 + z + z2 + ……. + z11 is equal purely imaginary number, for all z  C satisfying |z| = 1
to: (2016/Online Set–2) and Re z  1 , is : (2018/Online Set–1)
(a) an empty set (b) {0}
(a) 1250 3 i (b) 1250 3 i
 1 1
(c) 1365 3 i (d) 1365 3 i (c)  0, , -  (d) equal to R
 4 4
29. Let z  C, the set of complex numbers. Then the equation,
33. If |z – 3 + 2i|  4 then the difference between the greatest
2|z + 3i| – |z – i| = 0 represents : (2017/Online Set–1)
value and the least value of |z| is :
8 (2018/Online Set–2)
(a) a circle with radius
3
(a) 2 13 (b) 8
10
(b) a circle with diameter (c) 4 + 13 (d) 13
3
n
16  1+ i 3 
(c) an ellipse with length of major axis
 1 - i 3  = 1,
3 34. The least positive integer n for which  is:
 
16 (2018/Online Set–3)
(d) an ellipse with length of minor axis
9 (a) 2 (b) 3
(c) 5 (d) 6
65 COMPLEX NUMBER

EXERCISE - 3 : ADVANCED OBJECTIVE QUESTIONS

3  2i sin  z 2n  1
7. If z = cos  + i sin , then is equal to
1. 1  2i sin  will be purely imaginary, if  is equal to z 2n  1
(n is an integer)
  (a) i cot n (b) i tan n
(a) 2n  (b) n 
3 3
(c) tan n (d) cot n
i iz
 8. If z = re , then | i | is equal to
(c) n  (d) None of these
3 (a) e–r sin  (b) re–r sin 

2. The complex number z satisfies z + |z| = 2+ 8i. The value 



r cos 
of |z| is (c) e 2 (d) re–r cos 

(a) 10 (b) 13
 
(c) 17 (d) 23 9. If z n  cos  i sin for
n(n  1) (n  2) n(n  1) (n  2)
3. If (3  i) (z  z)  (2  i) (z  z)  14i  0, then z z is
n=1, 2, 3, ....., k, then the value of Lk 
im (z , z .....z ) is
1 2 k
equal to
(a) 5 (b) 8
1 i 1 3
(c) 10 (d) 40 (a)  (b)   i
2 2 2 2
4. The value of
1 3 1 i
         (c)   i (d) 
 cos  i sin  cos  i sin   cos  i sin  ... 2 2 2 2
 2 2  4 4  8 8
3
is 10. Number of complex numbers z satisfying z  z is

(a) –1 (b) 0 (a) 1 (b) 2

(c) – 3 (d) None of these (c) 4 (d) 5

4
 cos   i sin   4
5. is equal to 11. If z   2 , then the maximum value of |z| is equal to
5 z
 sin   i cos  
(a) cos  – i sin  (b) sin  – i cos  (a) (b)
3 1 5 1
(c) cos 9 – i sin 9 (d) sin 9 – i cos 9
(c) 2 (d) 2  2
6. If  (0,  the principal value of the arg (z) and |z| of
5 12. If and  are different complex numbers with || = l, then
1  cos   i sin  
the complex number z  3
is
 cos   i sin   
is
1  
 5   
(a)  ,32 cos (b) ,32 cos5 (a) 0 (b) 3/2
2 2 2 2
(c) 2 (d) 1
 4 
(c)  ,16 cos (d) None of these
2 2
COMPLEX
66 NUMBER 66
COMPLEX NUMBER

13. For a complex number z the minimum value of 20. If z is a complex number satisfying |z2 – 1|=4 |z|, then the
| z | + | z – 2 | is minimum value of |z| is
(a) 1 (b) 2
(a) 2 5  4 (b) 2 5  4
(c) 3 (d) None of these
14. If (a + ib) (c + id) (e + if) (g + ih) = A + i B, then (c) 5 2 (d) None of these
(a2 + b2) (c2 + d2) (e2 + f2) (g2 + h2) is equal to
(a) A2 – B2 (b) A2 + B2 1  iz
21. If z = x + iy and w  , then |w| = 1
zi
(c) A4 + B4 (d) A4 – B4
15. If 8iz3 + 12z2 – 18z + 27i = 0, then implies that in the complex plane
(a) z lies on the imaginary axis
3 2
(a) | z | (b) | z | (b) z lies on the real axis
2 3
(c) z lies on the unit circle
3
(c) | z | = 1 (d) | z | (d) None of the above
4

16. The minimum value of Z  1  2i  4i  3  Z is 22. The system of equations | z  1  i| 2  , (where i = 1 ) has

| z | 3
(a) 5 (b) 5

(c) 2 13 (d) 15 (a) no solution (b) one solution


(c) two solutions (d) none of these
17. If z1 and z2 are two complex numbers satisfying the equation
23. The area of the triangle on the complex plane formed by
z1  iz 2 z the complex numbers z, –iz and z + iz is
 1, then 1 is
z1  iz 2 z2
(a) |z|2 (b) | z |2
(a) purely real (b) of unit modulus
(c) purely imaginary (d) None of these | z |2
(c) (d) none of these
18. Let z1, z2 and z3 be three points on |z| = 1. If 1, 2 and 3 be 2
the arguments of z 1 , z 2 , z 3 respectively then 24. The equation |z + 1 – i| = |z – 1 + i| represents a
cos (1 – 2) + cos (2 – 3) + cos (3 – 1)
(a) straight line (b) circle
3 3 (c) parabola (d) hyperbola
(a)   (b)  
2 2
25. Number of solutions of the equation z2 + |z|2 = 0 is

3 (a) 1 (b) 2
(c)  (d) none of these
2 (c) 3 (d) infinitely many

19. If | z | = 1, then | 1 – z | + | 1 + z2 | is less than or equal to 26. The complex number z = 1 + i is rotated through an angle
3
(a) 4 (b) 3 in anticlockwise direction about the origin and
2
(c) 2 (d) 1
stretched by additional 2 units, then the new complex
number is

(a)  2  2i (b) 2  2i

(c) 2  2i (d) 2–2i


COMPLEX NUMBER 67

27. If | z | = max {| z – 1|, |z + 1|} then 33. |z1 + z2| = |z1| + |z2| is possible if

1 1
(a) | z  z | (b) z  z  1 (a) z 2  z1 (b) z 2 
2 z1

(c) | z  z | 1 (d) None of these (c) arg (z1) = arg (z2) (d) |z1| = |z2|
28. If z 1 lies in |z–3| < 4, z 2 on |z–1| + |z +1| = 3 and 34. Number of complex numbers z such that |z| = 1 and
A = |z1 –z2|, then
z z
  1 is
15 15 z z
(a) 0  A  (b) 0  A 
2 2
(a) 4 (b) 6
17 17 (c) 8 (d) more than 8
(c) 0  A  (d) 0  A 
2 2 35. Let P denotes a complex number z on the Argand’s plane,

2z  i  
29. If z C, then  m, m R represents a straight line and Q denotes a complex number 2 | z |2 CiS    
5z  1  4 
if where  = amp z. If ‘O’ is the origin, then the OPQ
(a) m = 2 (b) m = 5 is
(c) m = 2.5 (d) m = 0.4 (a) isosceles but not right angled

30. If arg (z1 ) = arg (z2), then (b) right angled but not isosceles
(c) right isosceles
(a) z2 = kz1–1 (k > 0) (b) z2 = kz1 (k > 0)
(d) equilateral
(c) | z 2 || z1 | (d) None of these
36. If A and B be two complex numbers satisfying

z1z  z 2 A B
31. If  k, k > 0 (z1, z2  0), then   1 . Then the two points represented by A and
z1z  z2 B A
B and the origin form the vertices of
(a) for k  1, locus z is a straight line
(a) an equilateral triangle
(b) for k {1, 0}, z lies on a circle
(b) an isosceles triangle which is not equilateral
(c) for k  0, z represents a point
(c) an isosceles triangle which is not right angled
(d) for k  1, z lies on the perpendicular bisector of the line
(d) a right angled triangle
z z
segment joining 2 and  2 . 37. If a point z1 is the reflection of a point z2 through the line
z1 z1
bz  bz  c, b  0, in the Argand plane, then bz2  bz1 is
32. Locus of z, if
equal to

 3 (a) 4c (b) 2c
 4 , when | z |  | z  2 | (c) c (d) None of these
arg[z  (1  i)]   is
  , when | z |  | z  2 | 38. If |z + 4|  3, then the maximum value of |z + 1| is
 4
(a) 4 (b) 10
(a) straight line passing through (2, 0) (c) 6 (d) 0
(b) straight lines passing through (2, 0), (1, 1)
(c) a line segment
(d) a set of two rays
68 COMPLEX NUMBER

39. If the complex numbers z 1 , z 2 , z 3 satisfying Passage Q. 45 to 48

z1  z3 1  i 3
 , then triangle is  2   2 
z 2  z3 2 Let w = cos    i sin   and  = w + w2 + w4 and
 7  7 
(a) an equilateral triangle
(b) a right angled triangle = w3 + w5 + w6.

(c) a acute angled triangle 45. equals :


(d) an obtuse angled isosceles triangle (a) –1 (b) 0

1 3 (c) 1 (d) 2
40. Which of the following is a fourth root of i ?
2 2 46. and are roots of the equations :

  (a) x2 + x + 1 = 0 (b) x2 + x + 2 = 0
(a) cis (b) cis
12 2 (c) x2 + 3x + 5 = 0 (d) None of these

  47. 2equals :
(c) cis (d) cis
6 3
(a) 1  7 i (b) 1  7 i
41. If z2 – z + 1 = 0, then zn – z–n, where n is a multiple of 3,
is (c) 1 + 7i (d) 1 – 7i
n
(a) 2 (–1) (b) 0
n+1 6
(c) (–1) (d) None of these k2
48. w equals :
42. The area of the triangle whose vertices are i, , , where k 0

i  1 and ,  are the non-real cube roots of unity,,


is (a) i (b) 7i

3 3 3 3
(a) (b) (c) –i (d)  7 i
2 4

3 49. The value of the expression


(c) 0 (d)
4
 1  1   1  1 
43. If 1,  2, ...... ,n–1 are n, nth roots of unity, the value 2  1  1  2   3  2   2  2 
         
of (9 – ) (9 – 2) ..... (9 – n – 1) will be

(a) n (b) 0  1  1   1  1 
4  3    3  2   .......  (n  1)  n    n  2  ,
        
9n  1 9n  1
(c) (d)
8 8 where  is an imaginary cube root of unity, is

44. If z and  are two non-zero complex numbers such that n (n 2  2) n (n 2  2)


(a) (b)
3 3

|z| = 1, and arg (z) – arg ()  , then z is equal to
2
n 2 (n  1) 2  4n
(c) (d) None of these
(a) 1 (b) –1 4

(c) i (d) –i
COMPLEX NUMBER 69

50. 1, 2, 3, .........., 100 are all the 100th roots of unity. The 56. If a complex number z has modulus 1 and argument
/3, then z2 + z
numerical value of 1  (  )5 is
i  j100 i j
(a) is purely imaginary
(a) 20 (b) 0
(b) has modulus 3
1/20
(c) (20) (d) None of these
(c) lies on the imaginary axis
6 6 (d) none of these
 1 i 3   1 i 3 
51. The value of  is
 1  i 3    1  i 3  57. If z0, z1 represents points P, Q on the locus |z–1|=1 and
   

(a) 2 (b) –2 the line segment PQ subtends an angle at the points
2
(c) 1 (d) 0 z = 1, then z1 is equal to
15
52. The number of roots of the equation z = 1 satisfying
i
|arg z| < /2 are (a) 1 + i (z0 –1) (b)
z0  1
(a) 6 (b) 7
(c) 8 (d) none of these (c) 1 – i (z0 –1) (d) i (z0 –1)
58. Let z1, z2 be two complex numbers represented by points
53. If z1 and z2 are two of the 8th roots of unity, such that
on the circle |z| = 1 and |z| = 2, respectively, then
z  (a) max |2z1+z2| = 4 (b) min |z1–z2| = 1
arg  1  is least positive, then z1/z2 is
 z2 
1
(c) z 2  3 (d) None of these
(a) 1 + i (b) 1 – i z1

1 i 1 i 59. If z1, z2, z3, z4 are roots of the equation


(c) (d)
2 2 a0z4 + a1z2 + a1z2 + a3z + a4 = 0,
54. If , , are the cube roots of a negative number p, then where a0, a1, a2, and a4 are real, then
for any three real numbers x, y, z the value of (a) z1 , z2 , z3 , z4 are also roots of the equation
x  y  z
is (b) z1 is equal to at least one of z1 , z2 , z3 , z4
x  y  z
(c)  z1 ,  z2 ,  z3 ,  z4 are also roots of the equation
1 i 3 1  i 3 (d) None of the above
(a) (b)
2 2
60. If is a complex constant such that z2 + z +  = 0 has
(c) (x + y + z) i (d)  a real roots, then
55. Suppose A(z1), B(z2) and C(z3) are vertices of a triangle (a)     1
lying on the unit circle |z| = 1. AD is altitude of the
ABC meeting the unit circle in E. (b)     0

(a) orthocentre of ABC is z1 + z2 + z3 (c)     1


(b) affix of E is –z2z3/z1 (d) the absolute value of the real root is 1

(c) if z12 = z2z3 and z22 = z3z1, then ABC is equilateral


(d) if z2 + z3 = 0, then ABC is a right angled.
70 COMPLEX NUMBER

61. If z1 = a + ib and z2 = c + id are complex numbers such 64. If A (z1), B (z2) and C (z3) be the vertices of a triangle ABC
that |z1| = |z2| = 1 and Re  z1 z2  = 0, then the pair of
 AB
complex numbers w1 = a + ic and w2 = b + id satisfies in which ABC  and  2, then the value of z2
4 BC
(a) |w1| =1 (b) |w2| =1
is equal to
(c) Re  w1w 2   0 (d) None of the above
(a) z3 + i (z1 + z3)
62 Let z1 and z2 be complex numbers such that z1  z 2 and (b) z3 – i (z1 – z3)
| z1 | = | z2 |. If z1 has positive real part and z2 has negative
(c) z3 + i (z1 – z3)
z z
imaginary part, then 1 2 may be (d) None of these
z1  z2
65. Match the following
(a) zero (b) real and positive
Column I Column II
(c) real and negative (d) purely imaginary
(A) Locus of the point z (p) A circle
63. Let P(x) and Q(x) be two polynomials. Suppose that
f(x) = P (x3) + xQ (x3) is divisible by x2 + x + 1, then satisfying the equation
(a) P(x) is divisible by (x–1) but Q(x) is not divisible by
Re (z2) = Re (z  z)
x–1
(b) Q(x) is divisible by (x–1) but P(x) is not divisible by (B) Locus of the point z (q) A straight line
x–1
satisfying the equation
(c) Both P(x) and Q(x) are divisible by x–1
|z–z1| + |z–z2|=, R+
(d) f (x) is divisible by x–1
and   |z1 – z2|

(C) Locus of the point z (r) An ellipse and mR+

satisfying the equation

2z  i
 m, where i  1
z 1

(D) If | z |  25, then the points (s) A rectangular

representing the complex hyperbola

number 1  75 z will be on
71 COMPLEX NUMBER

Passage Q.No. 66 to 68 67. If z 1 = 3 + 4i and z 2 = 4 + 3i, then the value of


Let z1 = a1 + ib1  (a1, b1) and z2 = a2 + ib2  (a2, b2); where  3 
sin        is equal to
i  1, be two complex numbers.  2 

1 7
(a)  (b) 
7 25

24 1
(c)  (d) 
25 25

68. If z1 = 5 + 12i and z2 = 3 + 4i, then (the projection of


z1 on z2 + projection of z2 on z1) is equal to

4131 3411
If POQ = , From Rotation theorem (a) (b)
65 65

z 2  0 | z 2 | i z z |z |
 e  2 1  2 e i 1134 1341
z1  0 | z1 | z1 z1 | z1 | (c) (d)
65 65

z 2 z1 | z 2 | i Assertion Reason
  e  z 2 z1 | z1 | | z 2 | ei
| z1 |2 | z1 | (A) If ASSERTION is true, REASON is true, REASON is a
correct explanation for ASSERTION.
 z 2 z1 | z1 | | z 2 | (cos   i sin )
(B) If ASSERTION is true, REASON is true, REASON is
 Re (z 2 z1 ) | z1 | | z 2 | cos  ... (i) not a correct explanation for ASSERTION.
(C) If ASSERTION is true, REASON is false
and Im (z 2 z1 ) | z1 | | z 2 | sin  ... (ii)
(D) If ASSERTION is false, REASON is true
The dot product of z1 and z2 is defined by z1oz2 = |z1| |z2| cos (E) If both ASSERTION and REASON are false.
= Re (z 2 z1 ) [from (i)] and cross product of z1 and z2 is 69. Assertion : If z is a complex number (z  1), then
defined z1 × z2 = |z1| |z2| sin = Im (z 2 z1 ) z
 1  | arg z |
[from Eq. (ii)] |z|

66. If z1 = 2 + 5i, z2 = 3 – i, then the value of (z1 .z 2  z 2  z1 ) Reason : In a unit radius circle chord (AP)  arc (AP)

is equal to (a) A (b) B


(c) C (d) D
(a) 2 (b) 3

(c) 2 3 (d) 3 2 1 3
70. Assertion : If |z| 2, then the least value of z  is .
2 2

Reason : |z1 + z2| |z1| + |z2|


(a) A (b) B
(c) C (d) D
COMPLEX NUMBER 72

EXERCISE - 4 : PREVIOUS YEAR JEE ADVANCED QUESTIONS


Single Answer Type Questions 6
 2k 2k 
8. The value of   sin  i cos  is (1987)
 1 i 
n k 1  7 7 
1. The smallest positive integer n for which    1, is
1 i  (a) –1 (b) 0
(1980) (c) –i (d) i
(a) 8 (b) 16 9. If z1 and z2 are two non-zero complex numbers such that |z1
+ z2| = |z1| + |z2|, then arg (z1) – arg (z2) is equal to
(c) 12 (d) None of these
(1987)
2. The complex numbers z = x + iy which satisfy the equation
z  5i 
 1, lie on (1981) (a) – (b) 
z  5i 2

(a) the x-axis 


(c) 0 (d)
(b) the straight line y = 5 2
(c) a circle passing through the origin 10. The complex numbers sin x + i cos 2x and cos x – i
(d) None of these sin 2x are conjugate to each other, for (1988)
(a) x = n (b) x = 0
5 5
 3 i  3 i (c) x = (n + 1/2) (d) no value of x
3. If z         , then (1982)
 2 2  2 2 11. If    1 is a cube root of unity and (1 + )7 = A + B, then
(a) Re (z) = 0 (b) Im (z) = 0 A and B are respectively (1995)
(c) Re (z) > 0, Im (z) > 0 (d) Re (z) > 0 , Im (z) < 0 (a) 0, 1 (b) 1, 1
4. The inequality |z – 4| < |z – 2| represents the region given (c) 1, 0 (d) – 1, 1
by (1982) 12. Let z and w be two non-zero complex numbers such that |z|
(a) Re (z) > 0 (b) Re (z) < 0 = |w| and arg (z) + arg (w) = , then z equals (1995)
(c) Re (z) > 0 (d) None of these (a) w (b) –w
5. If z = x + iy and w = (1 – iz) / (z – i), then |w| = 1 implies that, (c) w (d)  w
in the complex plane (1983)
13. Let z and w be two complex numbers such that
(a) z lies on the imaginary axis
|z| < 1, |w| < 1 and |z + iw| = | z  iw | = 2, then z equals
(b) z lies on the real axis
(1995)
(c) z lies on the unit circle
(a) 1 or i (b) i or –i
(d) None of these
(c) 1 or –1 (d) i – or – 1
6. The points z1, z2, z3, z4 in the complex plane are the vartices
14. For positive integers n 1, n2 the value of expression
of a parallelogram taken in oder, if and only if
n1 n1 n2 n2
(1983) 1  i   1  i 3   1  i5   1  i 7  , here i  1
(a) z1 + z4 = z2 + z3 (b) z1 + z3 = z2 + z4 is a real number, if and only if (1996)
(c) z1 + z2 = z3 + z4 (d) None of these (a) n1 = n2 + 1 (b) n1 = n2 – 1
7. If a, b, c and u, v, w are the complex numbers represeing (c) n1 = n2 (d) n1 > 0, n2 > 0
the vertices of two triangles such that c = (1 – r) a + rb and
w = (1 – r) u + rv, where r is a complex number, then the two 15. If  is an imaginary cube root of unity, then (1 + w – w2)7 is
triangles (1985) equal to (1998)

(a) have the same area (b) are similar (a) 128 (b) –128

(c) are congruent (d) None of these (c) 1282 (d) –1282
COMPLEX NUMBER 73

13
16. The value of sum   i n  i n 1  , where i  1 equals 23.
1
Let     i
3
, then value of the determinant
n 1
2 2
(1998)
1 1 1
(a) i (b) i – 1 2
1 1   2 is (2002)
(c) –i (d) 0
1 2 
6i 3i 1
(a) 3  (b) 3  ( – 1)
17. If 4 3i 1 = x + iy, then (1998) 2
(c) 3 (d) 3  (1 – )
20 3 i
24. For all complex numbers z1, z2 satisfying | z 1| = 12 and
(a) x = 3, y = 1 (b) x = 1, y = 1 | z2–3 – 4i | = 5, the minimum value of |z1 – z2| is
(c) x = 0, y = 3 (d) x = 0, y = 0 (2002)
334 365
(a) 0 (b) 2
 1 i 3  1 i 3 (c) 7 (d) 17
18. If i  1, then 4  5      3    
 2 2  2 
  2
z 1
equal to (1999) 25. If | z | = 1 and w   where z  1 , then Re (w) is
z 1
(a) 1  i 3 (b) 1  i 3 (2003)

(c) i 3 (d) i 3 1
(a) 0 (b)
19. If arg (z) < 0, then arg (–z) – arg (z) equals (2000) | z  1|2
(a)  (b) –
1 1 2
(c) –/2 (d) /2 (c) . (d)
z  1 | z  1|2 | z  1|2
20. If z 1 , z 2 and z 3 are complex numbers such that
n n
1 1 1 26. If    1 be a cube root of unity and 1  2   1  4  ,
| z1 |  | z 2 |  | z 3 |    1 , then |z1 + z2 + z3| is
z1 z 2 z 3 then the least positive value of n is (2004)
(a) 2 (b) 3
(2000)
(c) 5 (d) 6
(a) equal to 1 (b) less than 1
27. The minimum value of |a + b + c2|, where a, b and c are all
(c) greater than 3 (d) equal to 3
21. Let z1 and z2 be nth roots of unity which subtend a right not equal integers and    1 is a cube root of unity, is
angled at the origin, then n must be of the form (2005)
(where k is an integer) (2001)
1
(a) 4k + 1 (b) 4k + 2 (a) 3 (b)
2
(c) 4k + 3 (d) 4k
(c) 1 (d) 0
22. The complex numbers z 1 , z 2 and z 3 satisfying
z1  z3 1  i 3
 are the vertices of a triangle which is
z 2  z3 2
(2001)
(a) of area zero (b) right-angled isosceles
(c) equilateral (d) obtuse-angled isosceles
74 COMPLEX NUMBER

28. The shaded region, where P = (–1, 0), Q = (–1 + 2, 2 ) R 32. A particle P starts from the point z0 = 1 + 2i, where i  1.
It moves first horizontally away from origin by 5 units and
= (–1 + 2,  2 ), S = (1, 0) is represented by (2005)
then vertically away from origin by 3 units to reach a point
z1. From z1 the particle moves 2 units in the direction of

the vector ˆi  ˆj and then it moves through an angle in
2
anticlockwise direction on a circle with centre at origin, to
reach a point z2. The point z2 is given by
(2008)
(a) 6 + 7i (b) –7 + 6i
(c) 7 + 6i (d) –6 + 7i
15
2m 1
33. Let z = cos + i sin . Then, the value of  lm (z
m 1
) at
 = 2° is (2009)
(a) | z  1| 2,| arg | (z  1) |
4
1 1
 (a) (b)
(b) | z  1| 2,| arg | (z  1) | sin 2 3 sin 2
2

 1 1
(c) | z  1|  2,| arg | (z  1) | (c) (d)
4 2 sin 2 4 sin 2
34. Let z = x + iy be a complex number where x and y are

(d) | z  1| 2,| arg | (z  1) | integers. Then, the area of the rectangle whose vertices
2
are the roots of the equation zz3  zz3  350 is (2009)
29. If w =  + i, where   0 and z  1, satisfied the condition
(a) 48 (b) 32
 w  wz  (c) 40 (d) 80
that   is purely real, then the set of values of z is
 1 z  35. Let 1 be a cube root of unity and S be the set of all non-
(2006)
1 a b
(a) | z | = 1, z  2 (b) | z | = 1 and z  1
singular matrices of the form   1 c  , where each of

(c) z  z (d) None of the above 2  1 
30. A man walks a distance of 3 units from the origin towards
a, b and c is either or 2. Then the number of distinct
the north-east (N 45o E) direction. From there, he walks a
matrices in the set S is (2011)
distance of 4 units towards the north-west (N 45oW)
direction to reach a point P. Then, the position of P in the (a) 2 (b) 6
Argand plane is (2007) (c) 4 (d) 8
i/4 i/4
(a) 3e + 4i (b) (3 – 4i)e 36. Let z be a complex number such that the imaginary part of
(c) (4 – 3i)e i/4 i/4
(d) (3 + 4i)e z is non-zero and a = z2 + z + 1 is real. Then, a cannot take
the value (2012)
z
31. If | z | = 1 and z  1 , then all the values of lie on 1
1  z2
(a) –1 (b)
3
(2007)
(a) a line not passing through the origin 1 3
(c) (d)
2 4
(b) | z | 2
(c) the x-axis
(d) the y-axis
COMPLEX NUMBER 75

P Q R S
1
37. Let complex numbers  and lies on circles (a) 1 2 4 3

(b) 2 1 3 4
(x – x0)2 + (y – y0)2 = r2 and
(c) 1 2 3 4
(x – x0)2 + (y – y0)2 = 4r2 , respectively.
If z0 = x0 + iy0 satisfies the equation 2|z0|2 = r2 + 2, then || is (d) 2 1 4 3
equal to (2013) Multiple Type Question
41. If z1 = a + ib and z2 = c + id are complex numbers such that
1 1
(a) (b) |z1| = |z2| = 1 and Re (z1 z2 )  0 , then the pair of complex
2 2
numbers w1 = a + ic and w2 = b + id satisfies
1 1 (1985)
(c) (d)
7 3 (a) |w1| = 1 (b) |w2| = 1
Passage Type Questions (c) Re (w1 w 2 )  0 (d) None of these
Let S = S1 S2 S3, where 42. Let z 1 and z 2 be complex numbers such that
z1  z2 and |z1| = |z2|. If z1 has positive real part and z2 has
  z 1  3 i  
S1 = {z C : |z| < 4}, S2   z  C : Im    0 z1  z 2
negative imaginary part, then may be (1986)
  1 3 i   z1  z 2
and S3 : {z C : Re z > 0}
(a) zero (b) real and positive
38. Area of S is equal to (2013)
(c) real and negative (d) purely imaginary
10  20 43. Let z1 and z2 be two distinct complex numbers and let
(a) (b)
3 3 z = (1 – t) z1 + tz2 for some real number t with 0 < t < 1. If arg
(w) denotes the principal argument of a non-zero complex
16  32 number w, then (2010)
(c) (d)
3 3 (a) | z – z1 | + | z – z2 | = | z1 – z2 |
min |1–3i – z| is equal to (b) arg (z – z1) = arg (z – z2)
39. zs
(2013)
z  z1 z  z1
(c) 0 (d) arg (z – z1) = arg (z2 – z1)
2 3 2 3 z 2  z1 z2  z1
(a) (b)
2 2
3 i
3 3 3 3 44. Let w  and P = {wn : n = 1, 2, 3,....}. Further
(c) (d) 2
2 2
 1  1
H1   z  C : Re z   and H 2  z  C : Re z    ,
 2k   2k   2  2
40. Let z k  cos    i sin   ; k = 1,2, ...., 9. (2014)
 10   10  where C is the set of all complex numbers, if z1 P H1, z2
P H2 and O represents the origin, then z1 O z2 is equal
List I List II
to (2013)
P. For each zk there exists a zj such 1. True
that zk . zj = 1  
(a) (b)
Q. There exists a k  {1, 2, .... , 9} 2. False 2 6

such that z1 . z = zk has no solution 2 5


(c) (d)
z in the set of complex numbers. 3 6

| 1  z1 ||1  z 2 | ... | 1  z 9 | 45. Let  be a complex cube root of unity with 1 and
R. equals 3. 1 P = [pij] be a n × n matrix with pij = i + j. Then, p2 0, when
10
n is equal to (2013)
9  2k  (a) 57 (b) 55
S. 1  cos   equals 4. 2
k 1  10  (c) 58 (d) 56
76 COMPLEX NUMBER

46. Let a, b  R and a 2 + b 2  0. Suppose 49. Let s, t, r be non-zero complex numbers and L be the set of

S  z  C : z 
1 
, t  R, t  0  , where i  –1 If z  
solutions z  x  iy x, y  ,i  1 of the equation
 a  ibt 
sz  tz  r  0, where z  x  iy. Then, which of the
= x + iy and z  S, then (x, y) lies on (2016)
following statement(s) is (are) TRUE? (2018)
1  1  (a) If L has exactly one element, then s  t
(a) the circle with radius and centre  ,0 
2a  2a 
for a > 0, b ‘“ 0 (b) If s  t , then L has infinitely many elements
1  1 
(b) the circle with radius  and centre   ,0  (c) The number of elements in L  z : z  1  i  5 is at
2a  2a 
most 2
for a < 0, b ‘“ 0
(d) If L has more than one element, then L has infinitely
(c) the x-axis for a ‘“ 0, b = 0
many elements .
(d) the y-axis for a = 0, b ‘“ 0
Match the Column
47. Let a, b, x and y be real numbers such that
a – b = 1 and y  0. If the complex number z = x + iy 50. Match the conditions/expressions in Column I with
statement in Column II.
 az  b 
satisfies Im    y, then which of the following z  0 is a complex number
 z 1 
Column I Column II
is(are) possible value(s) of x ? (2017)
(A) Re(z) = 0 (p) Re (z2) = 0
(a) 1  1  y 2 (b) 1  1  y 2

(B) arg (z)  (q) Im (z2) = 0
4
(c) 1  1  y 2 (d) 1  1  y 2
(r) Re (z2) = Im (z2)
48. For a non-zero complex number z, let arg(z) denote the 51. Match the statement of Column I with these in Column II.
principal argument with -   arg  z    Then, which [Note: Here z takes values in the complex plane and Im (z)
of the following statement(s) is (are) FALSE? (2018) and Re (z) denotes respectively, the imaginary part and
real part of z]

(a) arg  -10 i   , where i  1 Column I Column II
4
(A) The set of points z (p)an ellipse with eccentricity
(b) The function f :  
  ,   defined by satisfying |z – i| z ||=|z+i|z|| is 4/5
f(t) = arg(-1 + it) for all t   , is continuous at all points contained in or equal to
(B) The set of points z (q) the set of points z
of , where i  1
satisfying |z+4|+|z–4|=10 is satisfying 1m (z) = 0
(c) For any two non-zero complex number z1 and z2,
contained in or equal to
z  (C) If |w|=2, then the set of (r) the set of points z
arg  1   arg  z1   arg  z 2  is an integer multiple of 2
 z2 
1
points z  w  is satisfying |1m z| 1
(d) For any three given distinct complex numbers z1, z2 w
and z3, the locus of the point z satisfying the condition
contained in or equal to
  z  z1   z 2  z3   (D) If |w| = 1, then the set of (s) the set of points
arg   , lies on a straight line .
  z  z   z  z  
 3 2 1 
1
points z  w  is satisfying |Re z|  2
w
contained in or equal to
(t) the set of points satisfying

|z|3 (2010)
COMPLEX NUMBER 77

Passage 60. If a and b are real numbers between 0 and 1 such that the
Read the following passage and answer the questions. points z1 = a + i, z2 = 1 + bi and z3 = 0 form an equilateral
triangle, then a = ... and b = ... . (1990)
Let A, B, C be three sets of complex number as defined
below 61. ABCD is a rhombus. Its diagonals AC and BD intersect at
the point M and satisfy BD = 2AC. If the points D and M
A = {z : Im z  1} represent the complex numbers 1 + i and 2 – i respectively,
B = {z : |z – 2 – i| = 3} then A represents the complex number...
or ... . (1993)
C = {z : Re ((1 – i)z) = 2} (2008)
62. Suppose z1, z2, z3 are the vertices of an equilateral triangle
52. The number of elements in the set A B C is
inscribed in the circle |z| = 2. If z1 = 1 + i 3, then
(a) 0 (b) 1
z2=..., z3 = ... . (1994)
(c) 2 (d) 
63. The value of the expression
53. Let z be any point in A  B  C. The 2 2 2
2 2
|z + 1 – i| + |z – 5 – i| lies between 1 (2–) (2– ) + 2 (3–) (3– ) +...+ (n–1) . (n–) (n– ),
(a) 25 and 29 (b) 30 and 34 where is an imaginary cube root of unity, is... (1996)
(c) 35 and 39 (d) 40 and 44 True/False
54. Let z be any point in A B C and let w be any point 64. For complex numbers z1 = x1 + iy1 and z2 = x2 + iy2, we write
satisfying |w – 2 – i| < 3. Then, |z| – |w| + 3 lies between z1 z2, if x1 x2 and y1 y2. Then, for all complex numbers
(a) –6 and 3 (b) –3 and 6 1 z
z with 1 z, we have  0. (1981)
(c) –6 and 6 (d) – 3 and 9 1 z
Integer Answer Type Questions 65. If the complex numbers z1, z2 and z3 represent the vertices
of an equilateral triangle such that |z1| = |z2| = |z3|, then
55. If z is any complex number satisfying |z – 3 –2i|  2, then
z1 + z2 + z3 = 0. (1984)
the minimum value of |2z – 6 + 5i| is... (2011)
i2/3 66. The cube roots of unity when represented on Argand
56. Let = e and a, b, c, x, y, z be non-zero complex numbers
2 diagram form the vertices of an equilateral triangle.
such that a + b + c = x, a + b + c = y,
(1988)
2 | x |2  | y |2  | z |2
a + b + c= z. Then, the value of is.... Subjective Questions
| a |2  | b |2  | c |2
(2011) a  ib
67. If x  iy  , prove that
 k   k 
c  id
57. For any integer k, let  k  cos    isin   , where
 7   7 
2 2 2 a 2  b2
12 (x + y ) = . (1978)
 k 1  k c2  d 2
k 1

i  1. The value of the expression 3


is 68. If x = a + b, y = a+ b, z = a+ b, where , are complex

k 1
4k 1   4k  2 3
cube roots of unity, show that xyz = a + b .
3

(1979)
(2015)
1
69. Express in the form A + iB. (1979)
Fill in the Blanks (1  cos )  2i sin 
70. It is given that n is an odd integer greater than 3, but n is
 x x  3 2
not a multiple of 3. Prove that x + x + x is a factor of
sin  2   cos  2   i tan (x)  n n
58. If the expression       is real, (x + 1) – x – 1. (1980)
  x  71. Find the real values of x and y for which the following
1  2 i sin  2   equation is satisfied
  
then the set of all possible value sof x is... (1987) (1  i) x  2i (2  3i) y  i
 i. (1980)
59. For any two complex numbers z1, z2 and any real numbers 3i 3i
2 2
a and b, |az1 – bz2| + |bz1 + az2| = ... . (1988)
78 COMPLEX NUMBER

72. Let the complex number z1, z2 and z3 be the vertices of an 85. Let a complex number , 1, be a root of the equation
equilateral triangle. Let z0 be the circumcentre of the z
p+q p
–z –z +1=0
q

2 2 2 2
triangle. Then prove that z1  z 2  z3  3z 0 . (1981) where p, q are distinct primes. Show that either
2 p–1
73. A relation R on the set of complex numbers is defined by z1 1 ++  +...+  = 0
2 q–1
z1  z 2 or 1 + +  + ... +  = 0
R z2, if and only if is real.
z1  z 2 but not both together. (2002)
86. If z 1 and z 2 are two complex numbers such that
Show that R is an equivalence relation. (1982)
74. Prove that the complex numbers z1, z2 and the origin form |z1| < 1 < |z2|, then prove that 1  z1 z2  1 . (2003)
2 2 z1  z 2
an equilateral triangle only if z  z  z1z2  0 . (1983)
1 2

75. If 1, a1, a2, ...., an–1 are the n roots of unity, then show that 87. Prove that there exists no complex number z such that
(1–a1) (1–a2) (1–a3) ... (1–an–1) = n. (1984) 1 n
r
76. Show that the area of the triangle on the Argand diagram
| z |
3
and a z
r 1
r  1, where |ar| < 2. (2003)

1 2 88. Find the centre and radius of the circle formed by all the
formed by the complex number z, iz and z + iz is |z| .
2 points represented by z = x + iy satisfying the relation
(1986) z
 k (k  1), where and are the constant complex
77. Complex numbers z 1,z 2,z 3 are the vertices A, B, C z 
respectively of an isosceles right angled triangle with right
2 numbers given by = 1 + i2, = 1 + i2.
angle at C. Show that (z1–z2) = 2(z1–z3) (z3–z2). (1986)
(2004)
78. Let z1 = 10 + 6i and z2 = 4 + 6i. If z is any complex number
such that the argument of (z–z1)/(z–z2) is /4, then prove 89. If one of the vertices of the square circumscribing the

that | z  7  9i | 3 2 . (1991) circle | z  1| 2 is 2  3 i . Find the other vertices of


3 2 square. (2005)
79. If iz + z – z + i = 0, then show that |z| = 1. (1995)
Single Answer (AIEEE)
80. |z|  1, |w|  1, show that
2 2 2 90. The locus of the centre of a circle which touches the circle
|z–w|  (|z| – |w|) + (arg z – arg w) (1995)
|z – z 1 | = a and |z – z 2 | = b externally
81. Find all non-zero complex numbers z satisfying z  iz 2 . (z, z1 and z2 complex numbers) will be (AIEEE 2002)
(1996) (a) an ellipse (b) a hyperbola
82.
2
Let z1 and z2 be the roots of the equation z + pz + q = 0, (c) a circle (d) none of these
where the coefficients p and q may be complex numbers.
z
Let A and B represent z1 and z2 in the complex plane. If 91. If   are |  | 1 , then z lies on
z  1/ 3 i
AOB = 0 and OA = OB, where O is the origin prove
(AIEEE 2005)
2  2
(a) a circle (b) an ellipse
that p = 4q cos   . (1997)
2 (c) a parabola (d) a straight line
83. Let bz  bz  c, b  0, be a line in the complex plane, where z2
92. If z  1 and is real, then the point represented by the
b is the complex conjugate of b. If a point z1 is the reflection z 1
of the point z 2 through the line, then show that complex number z lies (AIEEE 2012)
(a) either on the real axis or on a circle passing through the
c  z1 b  z 2 b . (1997)
origin
84. For complex numbers z and w, prove that (b) on a circle with centre at the origin
2 2
|z| w – |w| z = z – w, if and only if z = w or z w  1 . (c) either on the real axis or on a circle not passing through
the origin
(1999)
(d) on the imaginary axis
79 COMPLEX NUMBER

ANSWER KEY
EXERCISE - 1 : BASIC OBJECTIVE QUESTIONS
1. (b) 2. (b) 3. (d) 4. (b) 5. (a) 6. (b) 7. (b) 8. (c)
9. (b) 10. (c) 11. (c) 12. (c) 13. (c) 14. (d) 15. (c) 16. (a)
17. (a) 18. (c) 19. (d) 20. (b) 21. (a) 22. (b) 23. (c) 24. (a)
25. (c) 26. (b) 27. (d) 28. (b) 29. (d) 30. (a) 31. (a) 32. (b)
33. (c) 34. (c) 35. (a) 36. (b) 37. (d) 38. (c) 39. (d) 40. (b)
41. (d) 42. (c) 43. (b) 44. (c) 45. (b) 46. (d) 47. (c) 48. (d)
49. (a) 50. (b)

EXERCISE - 2 : PREVIOUS YEAR JEE MAINS QUESTIONS


1. (b) 2. (c) 3. (d) 4. (c) 5. (b) 6. (d) 7. (b) 8. (d)
9. (d) 10. (d) 11. (a) 12. (d) 13. (b) 14. (b) 15. (c) 16. (a)
17. (c) 18. (c) 19. (d) 20. (a) 21. (c) 22. (b) 23. (a) 24. (d)
25. (b) 26. (c) 27. (b) 28. (d) 29. (a) 30. (b) 31. (d) 32. (b)
33. (c) 34. (b)

EXERCISE - 3 : ADVANCED OBJECTIVE QUESTIONS


1. (c) 2. (c) 3. (c) 4. (a) 5. (d) 6. (a) 7. (b) 8. (c)
9. (d) 10. (d) 11. (b) 12. (d) 13. (b) 14. (b) 15. (a) 16. (c)
17. (a) 18. (a) 19. (a) 20. (c) 21. (b) 22. (a) 23. (c) 24. (a)
25. (d) 26. (d) 27. (d) 28. (d) 29. (d) 30. (a) 31. (b) 32. (d)
33. (c) 34. (c) 35. (c) 36. (a) 37. (c) 38. (c) 39. (a) 40. (a)
41. (b) 42. (d) 43. (c) 44. (d) 45. (d) 46. (b) 47. (a) 48. (b)
49. (c) 50. (b) 51. (a) 52. (b) 53. (c) 54. (b) 55. (a,b,c,d) 56. (a)
57. (a,c) 58. (a,b,c) 59. (a, b) 60. (a,c,d) 61. (a,b,c) 62. (a,d) 63. (c,d) 64. (b,c)
65. A–s; B–q,r; C–q,p; D–p 66. (d) 67. (b) 68. (c) 69. (a) 70. (b)

EXERCISE - 4 : PREVIOUS YEAR JEE ADVANCED QUESTIONS


1. (d) 2. (a) 3. (b) 4. (d) 5. (b) 6. (b) 7. (b) 8. (d)
9. (c) 10. (d) 11. (b) 12. (d) 13. (c) 14. (d) 15. (d) 16. (b)
17. (d) 18. (c) 19. (a) 20. (a) 21. (d) 22. (c) 23. (b) 24. (b)
25. (a) 26. (b) 27. (c) 28. (a) 29. (b) 30. (d) 31. (d) 32. (d)
33. (d) 34. (a) 35. (a) 36. (d) 37. (c) 38. (b) 39. (c) 40. (c)
41. (a,b,c) 42. (a,d) 43. (a,c,d) 44. (c,d) 45. (b,c,d) 46. (a,c,d) 47. (a,d) 48. (a,b,d)
49. (a,c,d) 50. A–q; B–p 51. A–q,r; B–p; C–p,t; D–q,r,s,t 52. (b) 53. (c) 54. (d)
–1 2 2 2 2
55. (5) 56. (3) 57. (4) 58. x = 2n+ 2, = tan k, where k (1, 2) or x = 2n 59. (a + b ) (|z1| + |z2| )
i 3i 1
60. a  b  2  3 61. 3  or 1  62. z 2  2, z3  1  i 3 63. n(n  1) (n 2  3n  4)
2 2 4

cot
1 2
64. True 65. True 66. True 69. A  iB  i 71. x = 3 and y = –1
   2
2  1  3cos 2  1  3cos
 2 2

3 i   k 2 k (  )
81. z  i,   88. Centre  2
, Radius  89. z 2   3 i, z3  (1  3)  i and z 4  (1  3)  i
2 2 1 k 1 k2

90. (b) 91. (d) 92. (a)

Dream on !!

03
SEQUENCE AND SERIES
81 SEQUENCE AND SERIES

SEQUENCE AND SERIES


(iii) The common difference can be zero, positive or
1. DEFINITION negative.
Sequence is a function whose domain is the set N of natural (iv) The sum of the two terms of an AP equidistant
numbers. from the beginning & end is constant and equal
to the sum of first & last terms.
Real Sequence : A sequence whose range is a subset of R
is called a real sequence. (v) Any term of an AP (except the first) is equal to
half the sum of terms which are equidistant from
Series : If a1, a2, a3, a4, ........., an, .......... is a sequence, it. an = 1/2 (an – k + an + k), k < n.
then the expression
For k = 1, an = (1/2) (an – 1 + an + 1) ;
a1 + a2 + a3 + a4 + a5 + ........ + ......... + an + ......... is a
series. For k = 2, an = (1/2) (an – 2 + an + 2) and so on.
A series if finite or infinite according as the number of (vi) tr = Sr – Sr – 1
terms in the corresponding sequence is finite or infinite. (vii) If a, b, c are in AP  2 b = a + c.
Progressions : It is not necessary that the terms of a (viii) A sequence is an AP, iff its nth terms is of the form
sequence always follow a certain pattern or they are An + B i.e., a linear expression in n. The common
described by some explicit formula for the nth term. Those difference in such a case is A i.e., the coefficient of n.
sequences whose terms follow certain patterns are called
progressions. 1.2 Geometric Progression (GP)
1.1 An Arithmetic Progression (AP)
GP is a sequence of numbers whose first term is non zero
& each of the succeeding terms is equal to the proceeding
AP is a sequence whose terms increase or decrease by a
terms multiplied by a constant. Thus in a GP the ratio of
fixed number. This fixed number is called the common
successive terms is constant. This constant factor is called
difference. If a is the first term & d the common difference,
the COMMON RATIO of the series & obtained by dividing
then AP can be written as a n th term of this AP as
any term by that which immediately proceeds it. Therefore
tn = a + (n – 1) d, where d = an – an – 1.
a, ar, ar2, ar3, ar4, ........... is a GP with a as the first term &
The sum of the first n terms the AP is given by ; r as common ratio.
n n (i) nth term = a rn – 1
Sn   2a  (n  1)d   a  .
2 2
where  is the last term. (ii) Sum of the Ist n terms i.e. Sn =

a rn 1  ,if r  1.
r 1
(iii) Sum of an infinite GP when |r| < 1 when n  
a
rn  0 if |r| < 1 therefore, S = | r | 1 .
1 r
Properties of Arithmetic Progression
(iv) If each term of a GP be multiplied or divided by the
(i) If each term of an A.P. is increased, decreased, same non-zero quantity, the resulting sequence is
multiplied or divided by the same non zero also a GP.
number, then the resulting sequence is also an AP.
(v) Any 3 consecutive terms of a GP can be taken as
(ii) 3 numbers in AP are a – d, a, a + d; a/r, a, ar ; any 4 consecutive terms of a GP can be
4 numbers in AP are a – 3d, a – d, a + d, a + 3d ; taken as a/r3, a/r, ar ar3 & so on.
5 numbers in AP are a – 2d, a – d, a, a + d, a + 2d; (vi) If a, b, c are in GP  b2 = ac.
6 numbers in AP are a – 5d, a – 3d, a – d, a + d,
a + 3d; a + 5d.
SEQUENCE
82 AND SERIES 82
SEQUENCE AND SERIES

Properties of Geometric Progressions


Sum of n AM’s inserted between a & b is equal to n
1. If all the terms of a GP be multiplied or divided
n
by the same non–zero constant, then it remains a times the single AM between a & b i.e.  A r  nA where
GP with the same common ratio. r 1

2. The reciprocals of the terms of a given GP forms A is the single AM between a & b.
a GP.
3. If each term of a GP be raised to the same power, 2.3 Geometric Mean
the resulting sequence also forms a G.P.
4. In a finite GP the product of the terms equidistant If a, b, c are in GP, b is the GM between a & c. b2 = ac,
form the beginning and the end is always same
therefore b = ac ; a > 0, c > 0.
and is equal to the product of the first and the last
term. 2.4 n-Geometric Means between a & b
5. Three non–zero numbers, a, b, c are in GP, if
b2 = ac. If a, b are two given numbers & a, G1, G2, ........, Gn, b are
6. If the terms of a given GP are chosen at regular in GP. Then G1, G2, G3, ............., Gn are n GMs between a
intervals, then the new sequence so formed also & b. G1 = a (b/a)1/n + 1 = ar, G2 = a (b/a)2/n + 1 = ar2, ............,
forms a GP. Gn a (b/a)n/n + 1 = arn where r = (b/a)1/ n + 1

7. If a1, a2, a 3, .... , an, .... is a GP of non–zero


non–negative terms, then log a 1 , log a 2 , ....
log an, .... is an AP and vice versa.

2. MEANS The product of n GMs between a & b is equal to the nth


n
n
2.1 Arithmetic Mean power of the single GM between a & b i.e.  G r  (G)
r 1

where G is the single GM between a & b.


If three terms are in AP then the middle term is called the
AM between the other two, so if a, b, c, are in AP, b is AM
of a & c. 2.5 Arithmetic, Geometric and Harmonic
AM for any n positive number a 1, a 2, ........., a n is ;
means between two given numbers
a1  a 2  a 3  .................  a n Let A, G and H be arithmetic, geometric and harmonic
A
n means of two positive numbers a and b. Then,

2.2 n-Arithmetic Means between Two Numbers ab 2ab


A , G  ab and H 
2 ab
If a, b are any two given numbers & a, A1, A2, ....., An, b are These three means possess the following properties
in AP then A1, A2, ..... An are n AM’s between a & b.
1. A>G>H
2. A, G, H form a GP i.e., G2 = AH.
ba 2(b  a) n (b  a)
A1  a  , A2  a  ,......, A n  a  3. The equation having a and b as its roots is
n 1 n 1 n 1 x2 – 2Ax + G2 = 0
A1= a + d, A2 = a + 2d, ............, An = a + nd, where
4. If A, G, H are arithmetic, geometric and harmonic
ba means between three given numbers a, b and c, then
d
n 1 the equation having a, b, c as its roots is

3G 2
x 3  3Ax 2  x  G 3  0.
H
SEQUENCE
83 AND SERIES 83
SEQUENCE AND SERIES

n
4 n
k  (n  1)(2n  1)(3n 2  3n  1)
k 1 30

4.4 Sum of first n odd numbers


Some important properties of Arithmetic & Geometric
Means between two quantities
n
2
1. If A and G are respectively arithmetic and   2 k  1  1  3  ...  (2n  1)  n
geometric means between two positive quantities k 1

a and b, then the quadratic equation having a, b


as its roots is x2 – 2Ax + G2 = 0. 5. ARITHMETICO-GEOMETRIC SERIES
2. If A and G be the AM and GM between two
A series each term of which is formed by multiplying the
positive numbers, then the number are
corresponding term of an AP & GP is called the
A  A2  G2 . Arithmetico-Geometric Series. e.g.
1 + 3x + 5x2 + 7x3 +............... Here, 1, 3, 5, ........ are in
3. SIGMA NOTATIONS AP & 1, x, x2, x3 ......... are in GP.

5.1 Sum of n terms of an Arithmetico-


3.1 Theorems
Geometric Series
n n n
Let Sn = a + (a + d) r + (a + 2 d) r2 + ..... +
(i)   a r  b r    a r   br
r 1 r 1 r 1 [a + (n – 1) d] rn – 1
n n n
a dr (1  r n 1 )  a  (n  1) d  r
(ii) ka
r 1
r  k a r
r 1
then Sn 
1 r

(1  r) 2

1 r
, r  1.

n
5.2 Sum to Infinity
(iii)  k  k  k  k........... n times = nk ; where k is a
r 1

constant. If |r| < 1 & n 

a dr
4. SUM TO n TERMS OF SOME then Limit r n  0. S  
n  1  r (1  r) 2
SPECIAL SEQUENCES
6. HARMONIC PROGRESSION (HP)
4.1 Sum of first n natural numbers
A sequence is said to HP if the reciprocals of its terms are
in AP.
n
n  n  1
 k  1  2  3  .....  n  2
. If the sequence a1, a2, a3, ..............., an is an HP then
k 1 1/a1, 1/a2, .........., 1/an is an AP & converse. Here we do
not have the formula for the sum of the n terms of an HP.
4.2 Sum of the squares of first n For HP whose first terms is a & second term is b, then nth
natural numbers ab
term is t n 
b  (n  1)(a  b)
n
2 n  n  1  2n  1
k  12  2 2  .....  n 2  . 2ac a ab
k 1 6 If a, b, c are in HP  b  or  .
ac c bc
4.3 Sum of the higher powers of first n
7. HARMONIC MEAN
natural numbers
If a, b, c are in HP, b is the HM between a & c, then
2 2
n
3 3 3  n  n  1   n 
3
b = 2ac/[a + c].]
 k  1  2  ........  n     k 
2   k 1 
k 1 
SEQUENCE
84 AND SERIES 84
SEQUENCE AND SERIES

SOLVED EXAMPLES

Example - 1 Example - 3

Find Find the value of n if 1 + 4 + 7 + 10 +... to n terms = 590


th
(i) 24 term of the A.P. 5, 8, 11, 14...
Sol. 1 + 4 + 7 + 10 + .... to n terms = 590 a = 1, d = 4 – 1 = 3
(ii) 15th term of the A.P. 21, 16, 11, 6,...
n
 Sn  [2a  (n  1) d]
Sol. (i) 5, 8, 11, 14... 2
 a = 5; d = 8 – 5 = 3; n = 24
 n
 tn = a + (n–1) d 590  [2(1)  (n  1)3]
2
 t24 = 5 + (24 – 1) 3
 2 × 590 = n [2 +3n – 3]
= 5 + 23 × 3
 1180 = n [3n – 1]
= 5 + 69
 3n2 – n – 1180 = 0
 t24 = 74
 3n2 – 60n + 59n – 1180 = 0
(ii) 21, 16, 11, 6...
 3n(n – 20) + 59 (n – 20) = 0
a = 21, d = 16 – 21 = – 5; n = 15
 (3n + 59) (n – 20) = 0
 t15 = 21 + (15 – 1) (–5)
= 21 + (14) (–5) 59
 n or n  20
= 21 – 70 3

t15 = –49 59


‘n’ can not be negative, n 
Example - 2 3
 n = 20
If for a sequence (tn), Sn = 4n2 – 3n, show that sequence is
an A.P. Example - 4

Sol. Sn = 4n2 – 3n If for an A.P. S16 = 784, a = 4, find d

tn = Sn + 1 – Sn
Sol. S16 = 784, a = 4
= [4 (n + 1)2 – 3 (n + 1)] – [4n2 – 3n]
n
= 4n2 + 8n + 4 – 3n –3 – 4n2 + 3n  Sn  [2a  (n  1) d]
2
tn = 8n + 1
tn–1 = 8(n – 1) + 1 16
S16  [2(4)  (16  1) d]
= 8n – 8 + 1 2
= 8n – 7 784 = 8 [8 + 15d]
tn – tn–1 = (8n + 1) – (8n – 7) 784 = 64 + 120 d
= 8 = constant  720 = 120 d
Hence as the difference between two conseuctive terms is 720
constant, it is A.P.  d
120
 d=6
SEQUENCE AND SERIES 85

Example - 5
1 
a   1  r   35
For the following G.P.’s find tn r 

(i) 1, –4, 16, –64, ... 1 35 35


1 r  
1 1 1 r a 10
(ii) 3, , ,
3 3 3 9 3 2 (1 + r + r2) = 7r
2r2 – 5r + 2 = 0
Sol.(i) 1, –4, 16, –64,... (r – 2) (2r – 1) = 0
a=1 r – 2 = 0 or 2r – 1= 0
4 r = 2 or 2r = 1
r  4
1 If a = 10, r = 2,
 tn = a(r)n–1 1
r = 2, r 
= 1 (–4)n–1 2
= (–4)n–1
a 10
for r = 2,   5, ar = 10 × 2 = 20
1 1 1 r 2
(ii) 3, , , , .....
3 3 3 9 3 the number are 5, 10, 20

a 3 1 a 10 1
for r  ,   20; ar  10   5
2 r 1/ 2 2
1
1 the number are 20, 10, 5
r 3 
3 3 a
(ii) Let , a, ar be three numbers in G.P..
r
 tn = a (rn–1)
n 1 a 13
1  a  ar  ... (i)
 3  r 3
3
a2 91
Example - 6  a 2  a 2r2  ... (ii)
r2 9
(i) Find three numbers in G.P. such that their sum is Taking square of (i)
35 and their product is 1000.
2 2
(ii) Find three numbers in G.P. such that their sum is a   13 
  a  ar    
91 r   3
13/3 and the sum of their squares is
3
a2 2a 2 169
2
 a 2  a 2r2   2a 2  2a  r 
r r 9
a
Sol.(i) Let three number are , a, ar
r  a2 2 2 2 a  169
 2  a  a r   2a   a  ar  
 r   r  9
a
 × a × ar = 1000
r 91  13  169
3
 2a   
a = 1000 9  3 9
a = 10 ... (1)
26a 169 91
 
3 9 9
86 SEQUENCE AND SERIES

26a 26 Example - 8

3 3
2n  2
 a=1 If for a squence, t n  , show that the sequence is a
5n  3
1 13 G.P. Find its first term and the common ratio.
1 r 
r 3

1  r  r 2 13 2n 2
 Sol. t n 
r 3 5 n 3

3 + 3r + 3r2 = 13r 2 n.52



3r2 – 10r + 3 = 0 5n.53
(r – 3) (3r – 1) = 0
n
 53   2 
1  2  
r = 3 or r  2 5
3
n
a 1 125  2 
for r = 3,  , ar = 1 ×3 = 3   
r 3 4 5

1 125  2 
n 1
 three numbers are , 1, 3 Let t (n 1)   
3 4 5

1 a 1 1 1 1
for r  ,   3; ar  1  three numbers are 3, 1, n 1
3 r 1 3 3 3 125  2 
t (n 1)   2
3 4 5
 n
 = constant
tn 125  2  5
Example - 7  
4 5

If x, y and z are pth, qth and rth terms of a G.P. respectively Hence sequence is in GP
then show that xq–r. yr–p. zp–q = 1
125 2 25
t1  a   
Sol. Let A be the first term and R be the common ratio of the 4 5 2
given G.P. Then,
t n 1 2
x = pth term x = AR(p–1)  
tn 5
y = qth term y = AR(q–1)
and z = rth term z = AR(r–1) Example - 9
L.H.S.
For a G.P.
(p 1) q  r (q 1) r  p ( r 1) p  q
 AR  .AR  . AR  (i) If a = 2, r = 3, Sn = 242, find n.
(ii) If S3 = 125, S6 = 152. find r.
= A(q–r) R(p–1) (q–r) A(r–p) R(q–1) (r–p) A(p-q) R(r–1) (p–q)
= A(q–r+r–p+q–q) R(p–1) (q–r) + (q–1) (r–p) + (r–1) (p–q)
Sol.(i) a = 2, r = 3, Sn = 242
= A° Rpq–pr–q+t+qr–pq–r+p+pr–qr–p+q
= A° R° = 1  r n  1
Sn  a  
= R.H.S.  r 1 

 3n  1 
242  2  
 3 1 
SEQUENCE AND SERIES 87

242 = 3n – 1
243 = 3n  0.1  1 
n 1  n 
 0.9  10 
35 = 3n
n=5
(ii) S3 = 125, S6 = 152,  (10 n )  1 
 10n 
 r3  1  r 6  1 n  
S3  a   and S6  a   9
 r 1   r 1 

 r 6  1 10 n  1 
a n n 
S6 r 1  r6 1  9(10 ) 
  3  3
S3  r  1 r  1
a 
 r 1 
1   1 
n 1   10 n 
9   
152 r 6  1

125 r 3  1 (ii) Sn = 0.5 + 0.55 + 0.555 + ....
By dividendo = 5 (0.1 + 0.11 + 0.111 + .....)

152  125 r 6  1  (r 3  1)
 5
125 r3 1  (0.9 + 0.99 + 0.999 +.....)
9
27 r 6  1  r 3  1 r 3 (r 3  1)
  3 = [0.9 + 0.99 + 0.999 + ............]
125 r3 1 (r  1)
5
3  (1  0.1)  (1  0.01)  (1  0.001)  1000
3 9
3
  r
5
 
5
3  [(1+1+1+...) – (0.1 + 0.01 + 0.001 +...)]
 r 9
5

Example - 10 5  n  sum of n terms of GP with 



9  a  0.1 and r  0.1 

Find the sum to n terms.
(i) 0.9 + 0.99 + 0.999 + ....
5   (0.1) (1  0.1n )  
(ii) 0.5 + 0.55 + 0.555 + ....  n   
9   1  0.1  

Sol. Sn = [0.9 + 0.99 + 0.999 +.....]


= [(1–0.1) + (1 – 0.01) + (1–0.001)....] 5  0.1  1  
 n   1  n   
9  0.9  10  
= [(1+1+1+...) – (0.1 + 0.01 + 0.001 + ......)
= n – (sum of n terms in GP with a = 0.1 and r = 0.1)
5 1   1  
n
 n  1   n   
 (0.1) (1  0.1 )  9 9   10   
Sn  n   
 1  0.1 
88 SEQUENCE AND SERIES

Example - 11 n
3
 Sn  2    2
Determine whether the sum of infinity of the following 2
G.P.s exist, in the case they exist then find
(i) 1, 2, 4, 8, 16.......... 3
 r 1
2
3 9 27
(ii) 1, , , ......... Sum of infinity does not exist.
2 4 8

5
5 5 5 5
(iii) 5, , , , ......... 1
2 4 8 16 (iii) a = 5, r  2 
5 2

Sol.(i) a = 1; r = 2
 1  rn 
Sn  a  
 1  rn   1 r 
Sn  a  
 1 r 
  1  n 
n
1  (2)  1  (2) n 1    
2
1  1  5   
 1  2   1    1  
 1  2  
  
Sn = [1–(2)n]
r=2>1
 The sum of infinity does not exist.
  1  n 
1    
2
(ii) a  1; r 
3 5   
2  3 
 2 
 
 1  rn 
 Sn  a  
 1 r 
10   1 n 
 Sn  1    
3   2  
  3 n 
1    
2
1    1
 3  |r| 1
 1 2  2
 

10   1 n 
 Sn  [1  0]      0 
3   2  
  3 n 
1    
2
1   
 1  10
  Sn 
 2  3

10
 Sum of infinity is of G.P..
 3 n
 3
Sn  2 1    
  2  
SEQUENCE AND SERIES 89

Example - 12
3  xn  x2 
Sn  (1  x)   nx n 
Find Sn of the following arithmetic geometric sequence. 1 x  x  1 
(i) 3, 6x, 9x2, 12x3, 15x4 ........
3  n xn  x2 
(ii) 1, 3x, 5x2, 7x3, 9x4 ........  Sn  (1  x  nx )  
1 x  x 1 

Sol.(i) In the given sequence AP is (ii) 1, 3x, 5x2, 7x3, 9x4 ..........
3, 6, 9, 12........ In the given sequence A.P. will be
 a = 3, d = 6 – 3 = 3 1, 3, 5, 7, 9..........
nth term will be  a = 1, d = 2
tn = a + (n – 1) d  tn = a + (n–1) d
tn = 3 + (n – 1) 3 = 1 + 2n – 2
tn = 3n = 2n –1
And, G.P. is 1, x, x2, x3, x4, ........ In the given sequence GP will be
x  1, x, x2, x3 .......
 a  1, r  x
1  a = 1, r = x
 nth term will be  tn = arn–1
tn = arn–1 tn = xn–1
= (1) (x)n–1  Sn = 1 + 3x + 5x2 + 7x3 ....... + (2n – 3)
 tn = xn–1 . xn–2 + (2n – 1) xn ... (i)
Sn = 3 + 6n + 9x2 + 12x3 + 15x4 .... + 3(n–1) . xn–2 + 3n.xn–1 multiplying both the side by x.
Sn = 3 + 6x + 9x2 + 12x3 ......... + 3 (n – 1). xn–2 + 3n.xn–1  x Sn = x + 3x2 + 5x3 + 7x4 ........ + (2n –3) xn–1 + (2n–1) xn
...... (i) ... (ii)
multiplying both the side x. subtracting (ii) from (i)
x.Sn = 3x + 6x2 + 9x3 + 12x4 ... + 3 (n–1) xn–1 + 3n.xn ... (ii)  Sn –x Sn = [(1 + 3x + 5x2 + 7x3 ...... + (2n–3)
Subtracting (ii) from (i) xn–2 + (2n –1) xn – (x + 3x2 + 5x3 ..... + (2n–3) xn–1 + (2n–1) xn]
 Sn – x Sn = (3 + 6 x + 9x2 + 12x3... + 3 (n–1) xn–2 (1–x) Sn = 1 + 2x + 2x2 + 2x3 ... + 2xn–1 – (2n –1) xn
+ 3(n) xn-1 – (3x + 6x2 + 9x3+ 12x4+....+3 (n–1) xn–1+3nxn, = 1 [2 (x + x2 + x3 +.... xn–1)] – (2n – 1) xn
(1–x) Sn = 3 + 3x + 3x2 + 9x3 .. + 3xn–1 – 3nxn
 x 
= 3 + 3x [1 + x + x2 + x3 .... + xn–2] – 3nxn
 1  2.
 x 1
 x n 1  1  (2n  1) x n

 x 
 3  3x 1  (x n 2  1)   3nx n 2x n 2x
 x 1     (2n  1) x n
x 1 x 1

 x n 1 x  n 2x n  2x
 3  3x 1     3nx   (2n  1) x n
 x  1 x  1 x 1

2x(x n 1  1)
3x n 3x 2   (2n  1) x n
(1–x) Sn = 3 + 3x +   3nx n x 1
x 1 x 1
2x(x n 1  1) (2n  1) x n
 Sn  
(x  1) 2 (x  1)
90 SEQUENCE AND SERIES

Example - 13 Example - 14

Find the following sums For a sequence, if Sn = 7 (4n–1), find tn and show that the
1.2.3 + 2. 3.4 + 3.4.5 +....+ n(n+1) (n+2) sequence is a G.P.

Sol. Tr1 = 1, 2, 3 = 1 + (r–1) 1 = r Sol. Sn = 7 (4n – 1)


Tr2 = 2, 3, 4 = 2 + (r–1) = r + 1 tn = Sn – Sn–1 = 7 [4n–1] – 7 [4n–1 – 1]
Tr3 = 3, 4, 5 = 3 (r–1) 1 = r + 2 (4) n
 7(4n )  7  7 7
 1.2.3 + 2.3.4 + 3.4.5 + .............. n terms 4
n n
  Tr1 . Tr2 .Tr3   (r (r  1) . (r  2))  1
r 1 r 1
 7  4n 1  
 4
n n n
  r 3   3r 2   2r 3
 7  4n   21[4n 1 ]
r 1 r 1 r 1 4

n 2 (n  1) 2 3n(n  1) (2n  1) 2n(n  1) t n 1 21[4 n 11 ] 4n


   r   4
4 6 2 tn 21[4 n 1 ] 4 n 1

n(n  1)  n(n  1)  common ratio is constant. Hence the given sequence is GP.
  2  (2n  1)  2 
2 Example - 15

n(n  1)  n(n  1)  4n  4  Find the sum of 1 + (1 + x) + (1 + x + x 2) +....+


  
2 2 (1 + x + x2 + ... + xn–1)

n(n  1)  n(n  1)  4n  6  Sol. Sn = 1 + (1 + x) + (1 + x + x2) +...+ (1 + x + x2 + ... + xn–1)


  
2 2 Tr = 1 + x + x2 +... xn–1
This is GP with first term 1 and common ratio ‘x’
n(n  1)  n 2  5n  6 
  
2  2  1 1  x r 
 tr  
1 x
n(n  1) (n  2) (n  3)

4 n
1  xr 1 n
1 n
r
Sn    1  1  x  x
r 1 1  x 1  x r 1 r 1

1 1  x(1  x n ) 
 (n)   
1 x 1 x  1 x 

n x(1  x n )
 
1  x (1  x)2
91 SEQUENCE AND SERIES

EXERCISE - 1 : BASIC OBJECTIVE QUESTIONS


AP 8. The 10th common term between the two arithmetic
1. nth term of the sequence progressions 3, 7, 11, 15 ..... and 1, 6, 11, 16 ......... is
a, a + d, a + 2d, ............. is (a) 191 (b) 193
(a) n + nd (b) a + (n – 1) d (c) 211 (d) None of these
(c) a + (n + 1) d (d) none of these 9. Three numbers are in A.P, such that their sum is 18 and sum
of their squares is 158. the greatest among them is
2. Let Tr be the rth term of an A.P., for r = 1, 2, 3,....... If for some
(a) 10 (b) 11
1 1
positive integers m, n. We have Tm  and Tn  , (c) 12 (d) None of these
n m
10. If roots of the equation x – 12 x2 + 39x – 28 = 0 are in AP, then
3
then Tmn equals :
its common difference is
1 1 1
(a) (b)  (a)  1 (b)  2
mn m n
(c) 1 (d) 0 (c)  3 (d)  4

3. If (x + 1), 3x and (4x + 2) are first three terms of an AP then its 11. The sum of first ten terms of a AP is four times the sum of its
5th term is first five terms, then ratio of first term and common difference
(a) 14 (b) 19 is

(c) 24 (d) 28 (a) 2 (b) 1/2

4. If 7th and 13th terms of an A.P. be 34 and 64 respectively, (c) 4 (d) 1/4
then its 18th term is 12. The sum of all odd numbers of two digits is
(a) 87 (b) 88 (a) 2530 (b) 2475
(c) 89 (d) 90 (c) 4905 (d) none of these
th
5. If an be the n term of an AP and if a7 = 15, then the value of 13. Sum of first n odd natural numbers is
the common difference that would make a2a7a12 greatest is (a) 2n + 1 (b) n2
(a) 9 (b) 9/4 (c) 2n – 1 (d) none of these
(c) 0 (d) 18 14. The sum of integers in between 1 and 100 which are divisible
by 2 or 5 is
6. Which of the following sequences is an A.P. with common
difference 3 ? (a) 3100 (b) 3600
(c) 2950 (d) 3500
(a) a n  2n 2  3n, n  N (b) a n  3n  5, n  N
15. If first term of an AP is 5, last term is 45 and the sum of the
2 terms is 400, then the number of terms is
(c) a n  3n 2  1, n  N (d) a n  2n  3, n  N
(a) 8 (b) 10
7. If a1 ,a 2 ,a 3 ,...., a n 1 are in A.P., then (c) 16 (d) 20

1 1 1 3  5  7  ........  n terms
  ,  7, then the value of n is
a1a 2 a 2a 3 a n a n 1 is 16. If
5  8  11  ........  10 terms

(a) 35 (b) 36
n 1 1
(a) a a (b) a a (c) 37 (d) 40
1 n 1 1 n 1

n 1 n
(c) a a (d) a a
1 n 1 1 n 1
SEQUENCE
92 AND SERIES 92
SEQUENCE AND SERIES

17. If for an A.P. T3 = 18 and T7 = 30 then S17 is equal to 25. The sum of first n (odd) terms of an A.P. whose middle term
(a) 612 (b) 622 is m is

(c) 306 (d) none of these (a) mn (b) mn


(c) nm (d) none of these
n(n  1)
18. If Sn  n P  Q, where Sn denotes the sum of the GP
2
26. If x, 2x + 2 and 3x + 3 are first three terms of a G.P., then its 4th
first n terms of an A.P., then the common difference is
term is
(a) P + Q (b) 2P + 3Q
(a) 27 (b) – 27
(c) 2Q (d) Q
(c) – 27/2 (d) 27/2
19. Let Sn denote the sum of first n terms of an A.P. If S2n = 3 Sn
27. If first, second and eight terms of a G.P. are respectively
then the ratio S3n/Sn is equal to n–4, nn, n52, then the value of n is
(a) 4 (b) 6 (a) 1 (b) 10
(c) 8 (d) 10 (c) 4 (d) none of these
20. If a1 ,a 2 ,a 3 , ......... is an A.P such that
a4 1
28. Let a1 ,a 2 ,a 3 ,.........., a n ,........... be a GP such that 
a1  a 5  a10  a15  a 20  a 24  225, a6 4

then a1  a 2  a 3  ........  a 23  a 24 is equal to and a 2  a 5  216. Then, a1 =

(a) 909 (b) 75 108


(a) 12 or, (b) 10
(c) 750 (d) 900 7
21. The first, second and middle term of an AP are a, b, c
54
respectively. Sum of all terms is (c) 7 or, (d) None of these
7
2 c  a  2c  c  a 
(a) (b) c 29. If a1 ,a 2 ,a 3 (a1  0) are three successive terms of a G.P. with
ba ba
common ratio r, the value of r for which a 3  4a 2  3a1 holds
2c  b  a  2b  c  a  is given by
(c) (d)
ca ba (a) 1 < r < 3 (b) –3 < r < – 1
22. The sum of the series (c) r > 3 or r < 1 (d) None of these
a – (a + d) + (a + 2d) – (a + 3d) + .... upto (2n + 1) terms is 30. If the first and the nth terms of a G.P. are a and b respectively
2
(a) – nd (b) a + 2 nd and P is the product of the first n terms, then P =
n
(c) a + nd (d) 2nd (a) ab (b) (ab)
n/2 2n
bca ca b a bc (c) (ab) (d) (ab)
23. If , , are in A.P. then
a b c 31. The second, third and sixth terms of an A.P. are consecutive
which of the following is in A.P. terms of a G.P. The common ratio of the G.P. is

(a) a, b, c (b) a2, b2, c2 (a) 1 (b) –1


(c) 3 (d) –3
1 1 1
(c) , , (d) none of these 32. The fourth, seventh and tenth terms of a G.P. are p, q, r
a b c
respectively, then
x x
24. If log 2, log (2 – 1) and log (2 + 3) are in AP, then the value
(a) p2  q 2  r 2 (b) q 2  pr
of x is given by
(a) 5/2 (b) log2 5 (c) p2  qr (d) pqr + pq + 1 = 0
(c) log3 5 (d) log5 3
SEQUENCE
93 AND SERIES 93
SEQUENCE AND SERIES

33. If pth, qth and rth terms of an A.P. are equal to corresponding 42. If the sum of first two terms of an infinite GP is 1 and every
terms of a G.P. and these terms are respectively x, y, z, then term is twice the sum of all the successive terms, then its
xy – z . yz – x . zx – y equals first term is
(a) 0 (b) 1 (a) 1/3 (b) 2/3
(c) 2 (d) none of these (c) 1/4 (d) 3/4
34. The sum of first n terms of the series.
43. The value of .423 is
1 – 1 + 1 – 1 + ... is
(a) 1 if n is odd and 0 when n is even 419 423
(a) (b)
999 999
(b) –1
(c) (–1)n (d) ± 1 423
(c) (d) none
th
35. The n term of a GP is 128 and the sum of its n terms is 255. 100
If its common ratio is 2 then its first term is 44. Three numbers form an increasing GP. If the middle number
(a) 1 (b) 3 is doubled, then the new numbers are in AP. The common
(c) 8 (d) none of these ratio of the GP is

36. If the sum of first 6 terms of a G.P. is nine times of the sum of (a) 2  3 (b) 2  3
its first three terms, then its common ratio is
(c) 3 2 (d) 3  2
(a) 1 (b) 3/2
(c) 2 (d) – 2 45. If a, b, c, d are in G.P. then an + bn, bn + cn, cn + dn are in

37. In a G.P. of even numbers of terms, the sum of all terms is 5 (a) A.P. (b) G.P.
times the sum of odd terms. The common ration of the GP is (c) H.P. (d) none of these
46. If a, b, c, d are in G.P., then (a 3 + b 3) –1 , (b 3 + c 3) –1,
4 1
(a)  (b) (c3 + d3)–1 are in
5 5
(a) A.P. (b) G.P.
(c) 4 (d) None of these
(c) H.P. (d) none of these
2
38. If S = 1 + a + a +........ to  (a  1), then the value of a is 47. If a, b, c are in G.P. and a = b1/y = c1/z then x, y, z are in
1/x

S S (a) A.P. (b) G.P.


(a) (b) (c) H.P. (d) none of these
S 1 1 S
48. The product of first n (odd) terms of a G.P. whose middle
S 1 1 S term is m is
(c) (d)
S S
(a) mn (b) mn
39. The value of 91/3  91/9  91/27 ....... to , is (c) nm (d) none of these

(a) 1 (b) 3 AGP

(c) 9 (d) None of these 49. The sum to n terms of the series

40. The sum of an infinite G.P. is 4 and the sum of the cubes of 2
 1  1
its terms is 192. The common ratio of the original G.P. is 1  2  1    3  1    .... is given by
 n   n
(a) 1/2 (b) 2/3
(c) 1/3 (d) –1/2 (a) n2 (b) n (n + 1)
2
41. If the sum of an infinitely decreasing GP is 3, and the sum of (c) n (1 + 1/n) (d) none of these
the squares of its terms is 9/2, the sum of the cubes of the 50. 1 + 2.2 + 3.2 + 4.2 + .... + 100.299 equals
2 3

terms is (a) 99.2100 (b) 100.2100


(a) 105/13 (b) 108/13 (c) 1 + 99.2100 (d) none of these
(c) 729/8 (d) none of these
94 SEQUENCE AND SERIES

58. If A1, A2 are two A.M.S. between two numbers a and b, then
1 1
51. Sum of infinite terms of series 3  5 .  7 . 2  .... is (2A1 – A2) (2A2 – A1) is equal to
4 4
(a) 33/4 (b) 11/4 ab
(a) a + b (b)
ab
(c) 44/9 (d) 44/8
(c) ab (d) none of these
52. If r term of a series is (2r + 1) 2–r, then sum of its infinite
th

terms is 59. If 4 AM’s are inserted between 1/2 and 3 then 3rd AM is
(a) 10 (b) 8 (a) – 2 (b) 2
(c) 5 (d) 0 (c) – 1 (d) 1
th
60. Six arithmetic means are inserted between 1 and 9/2, the 4
1 1
53. If 3  (3  d)  2 (3  2d) + ......... to   8, then the arithmetic mean is
4 4
(a) 2 (b) 1
value of d is
(c) 3 (d) 4
(a) 9 (b) 5
61. n AM’s are inserted between 2 and 38. If third AM is 14 then
(c) 1 (d) None of these n is equal to
HP (a) 9 (b) 7
3 (c) 8 (d) 10
54. The fourth term of the sequence 3, , 1, ......... is
2 62. If 4 GM’s be inserted between 160 and 5, then third GM will
be
3 4
(a) (b) (a) 8 (b) 118
4 3
(c) 20 (d) 40
2 63. If g1, g2 are two G.M.S. between two numbers a and b, then
(c) (d) none of these
3
g12 g 22
55. Let the positive numbers a,b,c,d be in A.P. Then, abc, abd,  is equal to
g 2 g1
acd, bcd are
(a) not in A.P./G.P./H.P. (b) in A.P. (a) a + b (b) ab
(c) in G.P. (d) in H.P. ab
(c) (d) none of these
56. If a1 ,a 2 ,a 3 , .........., a n are in H.P. then ab
64. If A1, A2 be two AM’s and G1, G2 be two GM’s between two
a1 a2 a3
, , , A1  A 2
a 2  a 3  ...  a n a1  a 3  ...  a n a1  a 2  a 4  ...  a n
numbers a and b, then G G is equal to
1 2

an
.....,
a1  a 2  ....  a n are in (a)
ab
(b)
2ab
2ab ab
(a) A.P. (b) G.P.
(c) H.P. (d) A.G.P. ab ab
(c) (d)
ab ab
AM and GM
65. If one A.M. ‘a’ and two G.M.’s p and q be inserted between
a n  bn
57. The value of n, for which n 1 is A.M. between a and any two numbers, then the value of p3  q3 is
a  b n 1
b is
2 pq
(a) 0 (b) 1 (a) (b) 2 apq
a
(c) –1/2 (d) –1
(c) 2 ap2q 2 (d) None of these
SEQUENCE AND SERIES 95

SIGMANOTATION 4006 4003


66. The sum of all numbers between 100 and 10,000 which are of (a) (b)
3006 3007
the form n3  n  N is equal to
4006 4006
(a) 55216 (b) 53261 (c) (d)
3008 3009
(c) 51261 (d) none of these 75. The limiting value of the sum to n terms of the series
67. Sum of n term of series 1.3 + 3.5 + 5.7 + ...... is
3 5 7
(a) 1/3n (n + 1) (2n + 1) + n    .......... as n   is
12.22 22.32 32.42
(b) 2/3n (n + 1) (2n + 1) – n
(a) 0 (b) 2
(c) 2/3n (n – 1) (2n – 1) – n
(d) none of these 1
(c) (d) 1
2
68. If 1 + 2 + 3 + ..... + n = 45, then 13 + 23 + 33 +....+ n3 is
76. The sum to n terms of the series
(a) (45)2 (b) (45)3
(c) (45)2 + 45 (d) none of these 3 5 7
 2  2  ......., is
2 2 2
69. The sum of series 1.3 + 2.5 + 3.7 + . . . upto 20 terms is 1 1  2 1  2 2  32
2 2

(a) 188090 (b) 189080


6n 9n
(c) 199080 (d) None (a) (b)
n 1 n 1
DIFFERENCEMETHOD
12n 3n
70. Sum of the series 4 + 6 + 9 + 13 + 18 + ......... n terms, is (c) (d)
n 1 n 1
n 2
(a)
6

n  3n  20  2
(b) n + 3n + 20 MISCELLANEOUS

1 3 7
n 2 77. nth term of sequence    ......... is
(c)
3

n  3n  20  (d) None of these 2 4 8

1  2(n  1) 1
71. Sum of the series 3 + 7 + 14 + 24 + 37 + .... 10 terms, is (a) (b) 1 
2n 2n
(a) 560 (b) 570 –n
(c) 2 – 1 (d) none of these
(c) 580 (d) none of these
1 1 1
72. Sum of the series 1 + 4 + 13 + 40 + 121 + ....... 16 terms, is 78. The nth term of the sequence , , , ....... is
17 17
2 6 12
(a) (3 – 35)/4 (b) 3 – 35
17 17 1 1 1
(c) (3 – 33)/2 (d) (3 – 32)/4 (a) (b) 
n2  n n n 1
CANCELLATION METHOD
1 1
1 1 1 (c) (d)
73. The sum to n terms of the series    ..... is n2 1 n 1
1.3 3.5 5.7
79. 9th term of the sequence
1 2n 1, 1, 2, 3, 5, ...... is
(a) (b)
2n  1 2n  1 (a) 13 (b) 34

n 2n (c) 21 (d) none of these


(c) (d) 80. The sum of the series
2n  1 n 1

1 1  3x  6x 2  10x3  ...... is
74. If t n  (n  2) (n  3) for n = 1, 2, 3, .........., then
4
1 1
(a) (b)
1 1 1 1 (1  x) 2 1 x
   .......  
t1 t 2 t 3 t 2003
1 1
(c) (d)
(1  x)2 (1  x)3
SEQUENCE AND SERIES 96

EXERCISE - 2 : PREVIOUS YEAR JEE MAINS QUESTIONS

1. If 1, log9 (31–x + 2), log3 [4. 3x – 1] are in AP, then x equals   

(2002) 7. If x   n 0
an , y  
n 0
bn , z  c
n 0
n
where a, b, c are in A.P..

(a) log3 4 (b) 1 – log3 4


and |a| < 1, |b| < 1, |c| < 1 then x, y, z are in (2005)
(c) 1 – log4 3 (d) log 4 3
(a) HP
2. Sum of infinite number of terms in GP is 20 and sum of their
(b) Arithmetic–Geometric progression
square is 100. The common ratio of GP is (2002)
(c) AP
(a) 5 (b) 3/5
(d) GP
(c) 8.5 (d) 1/5
8. If a 1 , a 2 , ...., a n are in H.P., then the expression
3. The value of 21/4 . 41/8 . 81/16 . . is (2002)
a1a2 + a2a3 +....+ an–1 an is equal to (2006)
(a) 1 (b) 2
(a) n (a1 – an)
(c) 3/2 (d) 4
(b) (n – 1) (a1 – an)
4. Fifth term of an GP is 2, then the product of its first 9 terms
(c) na1 an
is (2002)
(d) (n – 1) a1 an
(a) 256 (b) 512
(c) 1024 (d) none of these a1  a 2  ...  a p p2
9. Let a1, a2, a3, ... be terms of an AP. If  ,
5. Let Tr be the rth term of an A.P. whose first term is a and a1  a 2  ...  a q q2
common difference is d. If for some positive integers m, n,
a6
1 1 p  q, then equals (2006)
m n, Tm  , and Tn  , then a – d equals a 21
n m

(2004)
7 2
(a) (b)
1 2 7
(a) (b) 1
mn
11 41
(c) (d)
1 1 41 11
(c) 0 (d) 
m n 10. If p and q are positive real numbers such that p2 + q2 = 1,
6. The sum of the first n terms of the series then the maximum value of (p + q) is (2007)

n (n  1)2 1
12 + 2.22 + 32 + 2.42 + 52 + 2.62 + ... is when n is (a) 2 (b)
2 2

even. When n is odd the sum is (2004)


1
(c) (d) 2
3n (n  1)
2
n (n  1) 2
(a) (b)
2 2

2
n (n  1) 2  n (n  1) 
(c) (d)  
4  2 
SEQUENCE AND SERIES 97

11. In a geometric progression consisting of positive terms, 16. A man saves Rs. 200 in each of the first three months of
each term equals the sum of the next two terms. Then the his service. In each of the subsequent months his saving
common ratio of this progression, equals (2007) increases by Rs. 40 more than the saving of immediately
previous month. His total saving from the start of service
1 will be Rs. 11040 after (2011)
(a) 5 (b) ( 5  1)
2
(a) 19 months (b) 20 months
1 1 (c) 21 months (d) 18 months
(c) (1  5 ) (d) 5
2 2 17. Statement 1 The sum of the series 1 + (1 + 2 + 4) +
12. The first two terms of a geometric progression add up to (4 + 6 + 9) + (9 + 12 + 16) + ... + (361 + 380 + 400) is 8000.
12. The sum of the third and the fourth terms is 48. If the n
3
terms of the geometric progression are alternately positive Statement 2  (k  (k  1)3 )  n 3 , for any natural
k 1
and negative, then the first term is (2008)
(a) 4 (b) – 4 number n. (2012)

(c) – 12 (d) 12 (a) Statement 1 is false, statement 2 is tgrue.

13. The sum to the infinity of the series (b) Statement 1 is true, statement 2 is true;
Statement 2 is a correct explanation for statement 1.
2 6 10 14
1   2  3  4  ... is (2009) (c) Statement 1 is true, statement 2 is true;
3 3 3 3
statement 2 is not a correct explanation for statement 1.
(a) 3 (b) 4
(d) Statement 1 is true, statement 2 is false.
(c) 6 (d) 2
18. If 100 times the 100th term of an AP with non-zero common
14. A person is to count 4500 currency notes. Let an denotes difference equals the 50 times its 50th term, then the 150th
the number of notes he counts in the nth minute. If term of this AP is (2012)
a1 = a2 = .... = a10 = 150 and a10, a11,... are in AP with common
(a) – 150 (b) 150 times its 50th term
difference –2, then the time taken by him to count all notes,
is (2010) (c) 150 (d) zero

(a) 24 min (b) 34 min 19. The sum of first 20 terms of the sequence 0.7, 0.77, 0.777,...,
is (2013)
(c) 125 min (d) 135 min
15. Let a n be the nth term of an AP. If 7 7
(a) (179  10 20 ) (b) (99  10 20 )
100 100
81 9
 a 2r   and  a 2r 1  , then the common difference
r 1 r 1
7 7
(c) (179  10 20 ) (d) (99  10 20 )
of the AP is (2011) 81 9

  20. If x, y and z are in AP and tan–1 x, tan–1 y and tan–1 z are also
(a) (b) –  in AP, then (2013)
200
(a) x = y = z (b) 2x = 3y = 6z
  (c) 6x = 3y = 2z (d) 6x = 4y = 3z
(c) (d) – 
100
21. Three positive numbers form an increasing G.P. If the middle
term in this G.P. is doubled, the new numbers are in A.P.
Then the common ratio of the G.P. is : (2014)

(a) 2  3 (b) 2 3

(c) 3  2 (d) 2  3
98 SEQUENCE AND SERIES

22. If (10)9 + 2(11)1 (10)8 + 3 (11)2 (10)7 +...+10 (11)9 = k (10)9, then 28. The least positive integer n such that
k is equal to : (2014)
2 2 2 1
1   .....  n 1  , is: (2014/Online Set–3)
121 3 32 3 100
(a) 110 (b)
10
(a) 4 (b) 5
441 (c) 6 (d) 7
(c) (d) 100
100
29. The number of terms in an A.P. is even; the sum of the odd
23. Given an A.P. whose terms are all positive intergers. The terms in it is 24 and that the even terms is 30. If the last
sum of its first nine terms is greater than 200 and less than
1
220. If the second term in it is 12, then its 4th term is term exceeds the first term by 10 then the number of
2
(2014/Online Set–1)
terms in the A.P. is : (2014/Online Set–4)
(a) 8 (b) 16
(a) 4 (b) 8
(c) 20 (d) 24
(c) 12 (d) 16
3 5 7
24. if then sum 2
 2 2
  .....  up to 20  1 3n 
1 1 2 1  2 2  32
2
30. Let f (n)     n, where [n] denotes the greatest
 3 100 
k
terms is equal to , then k is equal to : 56
21
integer less than or equal to n. Then  f (n) is equal to:
n 1
(2014/Online Set–1)
(2014/Online Set–4)
(a) 120 (b) 180
(a) 56 (b) 689
(c) 240 (d) 60
(c) 1, 287 (d) 1, 399
25. In a geometric progression, if the ratio of the sum of first 5
31. The sum of first 9 terms of the series
terms to the sum of their reciprocals is 49, and the sum of
the first and the third term is 35. Then the first term of this 13 13  23 13  23  33
   ....... is (2015)
geometric progression is: (2014/Online Set–2) 1 1 3 1 3  5
(a) 7 (b) 21
(a) 142 (b) 192
(c) 28 (d) 42
(c) 71 (d) 96
26. The sum of the first 20 terms common between the series
32. If m is the A.M. of two distinct real number l and n
3 + 7 + 11 + 15 + .... and 1 + 6 + 11 + 16 + ...., is
(l, n > 1) and G1, G2 and G3 are three geometric means
(2014/Online Set–2)
between l and n, then G14  2G24  G34 equals. (2015)
(a) 4000 (b) 4020
(a) 4 lmn2 (b) 4 l 2m2n2
(c) 4200 (d) 4220
(c) 4 l 2mn (d) 4 lm2n
27. Let G be the geometric mean of two positive numbers a
33. Let the sum of the first three terms of an A.P. be 39 and the
1 1 1 sum of its last four terms be 178. If the first term of this
and b, and M be the arithmetic mean of and If :G
a b M A.P. is 10, then the median of the A.P. is :
is 4 : 5, then a : b can be: (2014/Online Set–3) (2015/Online Set–1)
(a) 1 : 4 (b) 1 : 2 (a) 29.5 (b) 26.5
(c) 2 : 3 (d) 3 : 4 (c) 28 (d) 31
SEQUENCE AND SERIES 99

34. The sum of the 3rd and the 4th term of a G.P. is 60 and the 40. If the arithmetic mean of two numbers a and b, a > b > 0, is
product of its first three terms is 1000. If the first term of
ab
this G.P. is positive, then its 7th term is : five times their geometric mean, then is equal to :
ab
(2015/Online Set–2)
(2017/Online Set–1)
(a) 7290 (b) 640
(c) 2430 (d) 320 6 3 2
(a) (b)
35. If the 2nd, 5th and 9th terms of a non-constant A.P. are in G.P, 2 4
then the common ratio of this G.P. is : (2016)
7 3 5 6
4 (c) (d)
(a) (b) 1 12 12
3
41. If the sum of the first n terms of the series
7 8
(c) (d) 3  75  243  507  ...... is 435 3, then n equal :
4 5

36. If the sum of the first ten terms of the series (2017/Online Set–1)
2 2 2 (a) 18 (b) 15
 3  2  1 2  4 16
1    2    3   4   4   ......, is 5 m, then (c) 13 (d) 29
 5  5  5  5

m is equal to : (2016) 42. If three positive numbers a, b and c are in A.P. such that
abc = 8, then the minimum possible value of b is :
(a) 101 (b) 100
(2017/Online Set–2)
(c) 99 (d) 102
37. Let x, y, z be positive real numbers such that 1
(a) 2 (b) 4 3
x + y + z = 12 and x3y4z5 = (0.1) (600)3. Then x3 + y3 + z3 is
equal to : (2016/Online Set–1) 2
(c) 4 3 (d) 4
(a) 270 (b) 258
(c) 342 (d) 216 1 1 2 1 2  3
43. Let Sn   3  3 
38. Let a1,a 2 ,a 3 ....., a n ,..... be in A.P. If 1 1  2 1  23  33
3 3

a 3  a 7  a11  a15  72, then the sum of its first 17 1  2  ...........  n


.  . If 100 S = n, then n is equal
terms is equal to : (2016/Online Set–2) 13  23  ...........  n 3 n

(a) 306 (b) 153 to : (2017/Online Set–2)


(c) 612 (d) 204 (a) 199 (b) 99
39. For any three positive real numbers (c) 200 (d) 19
2 2 2
a, b and c, 9(25a + b ) + 25 (c – 3ac) = 15b (3a + c). Then:
44. Let a1 ,a 2 , a 3 ,....., a 49 be in A.P. such that
(2017)
12
(a) b, c and a are in G.P (b) b, c and a are in A. P  a 4k 1  416 and a 9  a 43  66 . If
k 0
(c) a, b and c are in A.P (d) a, b and c are in G.P
a12  a 22  ....  a17
2
 140m, then m is equal to :

(2018)
(a) 33 (b) 66
(c) 68 (d) 34
100 SEQUENCE AND SERIES

2 3 n
1 1 1  3  3  3 n-1  3 
LetA n =   -   +   - ... +  -1    and B = 1 – A .
45. If x1, x2, . . ., xn and , ,...., are two A.P.s such that 48.
h1 h2 hn 4 4 4 4 n n

x3= h2 = 8 and x8= h7 = 20, then x5Ŕh10 equals : Then, the least odd natural number p, so that Bn > A n , for
(2018/Online Set–1) all n  p, is : (2018/Online Set–2)
(a) 2560 (b) 2650 (a) 9 (b) 7
(c) 3200 (d) 1600 (c) 11 (d) 5
46. If b is the first term of an infinite G.P. whose sum is five,
then b lies in the interval : (2018/Online Set–1) 1 1 1
49. Let , ,...,  x i  0,fori = 1,2,...,n  be in A.P. such
x1 x 2 xn
(a)  - , - 10 (b)  -10, 0 
that x1=4 and x21 =20. If n is the least positive integer for
(c)  0, 10  (d) 10,   n
1
47. If a, b, c are in A.P. and a2,b2, c2 are in G.P. such that
which xn > 50, then   x  is equal to :
i=1 i

3 (2018/Online Set–3)
a <b < c and a + b + c = , then the value of a is :
4
1
(2018/Online Set–2) (a) (b) 3
8
1 1 1 1
(a) - (b) - 13 13
4 4 2 4 3 2 (c) (d)
8 4
1 1 1 1 50. The sum of the first 20 terms of the series
(c) - (d) -
4 2 2 4 2
3 7 15 31
1 + + + + + ..., is (2018/Online Set–3)
2 4 8 16

1 1
(a) 38 + (b) 38 +
219 220

1 1
(c) 39 + (d) 39 +
220 219
101 SEQUENCE AND SERIES

EXERCISE - 3 : ADVANCED OBJECTIVE QUESTIONS


2 2
Single Type Question 6. If ,  be roots of x – 3x + a = 0 and ,  are the roots of x
– 12x + b = 0 and , , ,  (in order) form an increasing G.P.,
1. The first and last term of an A.P. are a and l respectively. If
then
s be the sum of all terms of the A.P., then common difference
is (a) a = 3, b = 12 (b) a = 12, b = 13
(c) a = 2, b = 32 (d) a = 4, b = 16
2  a 2 2  a 2
(a) (b) 7. If a, b, c are three unequal numbers such that a, b, c are in
2s  (  a ) 2s  (  a ) A.P. and b – a, c – b, a are in G.P., then a : b : c =
(a) 2 : 3 : 5 (b) 1 : 2 : 4
2  a 2 2  a 2
(c) (d) (c) 1 : 3 : 5 (d) 1 : 2 : 3
2s  (  a ) 2s  (  a )
8. The sum of n terms of the following series
2
3  5  7  ...to n terms 1 + (1 + x) + (1 + x + x ) + . . . will be
2. If  7, then n =
5  8  11  ...to 10 terms
1 x n x (1  x n )
(a) (b)
(a) 35 (b) 36 1 x 1 x
(c) 37 (d) 40
n (1  x )  x (1  x n )
3. If a1, a2, a3, . . ., an are in A.P. where ai > 0 Vi , then (c) (d) none
(1  x ) 2

1 1 1 9. If the third term of a G.P. is 4, then the product of its first 5


  ...  
a1  a 2 a 2  a3 a n 1  a n terms is
3 4
(a) 4 (b) 4
5
n 1 n 1 (c) 4 (d) None
(a) a1  a n (b) a1  a n 10. If a, b, c are three unequal numbers such that a, b, c are in
A.P. and b – a, c – b, 4a – c are in G.P. then a : b : c is
n 1 n 1 (a) 1 : 2 : 3 (b) 1 : 3 : 4
(c) a1  a n (d) a1  a n (c) 2 : 3 : 4 (d) 1 : 2 : 4
2
4. If the sum of the 10 terms of an A.P. is 4 times to the sum of 11. The least value of n for which the sum 1 + 3 + 3 + .... to n
its 5 terms, then the ratio of first term and common terms is greater than 7000. is
difference is (a) 7 (b) 9
(a) 1 : 2 (b) 2 : 1 (c) 11 (d) 13
(c) 2 : 3 (d) 3 : 2 12. If a, 4, b are in AP; a, 2, b are in G.P., then a, 1, b are in
(a) HP (b) AP
n
5. A series whose nth term is  y, the sum of r terms will be (c) GP (d) none of these
x
13. If p, q, r are in A.P., then pth, qth and rth terms of any G.P.
r (r  1) r ( r  1) are in
(a)  ry (b)
2x 2x (a) A.P. (b) G.P.
(c) H.P. (d) A.G.P.
r r (r  1)
(c) (r  1)  ry (d) 2 y  rx
2x
SEQUENCE
102 AND SERIES 102
SEQUENCE AND SERIES
th th 2 2 2
14. In a G.P. if the (m + n) term be p and (m – n) term be q then 22. If a, b, c are 3 positive numbers in A.P. and a , b , c are in
th
the m term is H.P., then
(a) a = b = c (b) 2b = 3a + c
(a) pq (b) p /q
1/ 2
2  ac 
(c) q /p (d) p /q (c) b =   (d) None
8
15. The fourth, seventh and tenth terms of a G.P. are p, q, r
respectively then, ba bc
2 2 2 2
23. If a, b, c are in H.P., then the value of  is
(a) p = q + r (b) q = pr ba bc
2
(c) p = qr (d) pqr + pq + 1 = 0 (a) 0 (b) 1
16. The sum of an infinite G.P. series is 3. A series which is (c) 2 (d) 3
formed by squares of its terms have the sum also 3. First 24. The sum of three consecutive terms in G.P. is 14. If 1 is
series will be added to the first and the second term and 1 subtracted
from the third, the resulting new terms are in A.P. Then the
3 3 3 3 1 1 1 1
(a) , , , ... (b) , , , ,... lowest of the original terms is
2 4 8 16 2 4 8 16
(a) 1 (b) 2
1 1 1 1 1 1 1 (c) 4 (d) 8
(c) , , , ,... (d) 1, , 2 , 3 ,...
3 9 27 81 3 3 3 25. If ln (x + z) + ln (x – 2y + z) = 2 ln (x – z), then x, y, z are in
(a) A.P (b) G.P
a 1 2
17. If a, b, c are in A.P., then , . are in
bc c b (c) H.P (d) none of these
1/3 1/9 1/27
26. The value of 9 . 9 . 9 ...... will be
(a) A.P. (b) G.P.
2 3
(a) 3 (b) 3
(c) H.P. (d) None

18. If a, b, c are in H.P., then which one of the following is true (c) 3 (d) 3

1 1 1
1 1 1 ac y
(a)   (b) b 27. If ax  b  c z and a, b, c in G.P., then x, y, z are in
ba bc b ac
(a) A.P. (b) G.P.
ba bc (c) H.P. (d) none
(c)  1 (d) None
ba bc
28. The harmonic mean of roots of the equation
19. 1 + 3 + 7 + 15 + 31 + . . . to n terms =
n+1 n+1 (5  2 ) x 2  (4  5 ) x  (8  2 5 )  0 is
(a) 2 –n (b) 2 –n–2
n
(c) 2 – n – 2 (d) None (a) 2 (b) 4
20. If x, 1, z are in A.P. and x, 2, z are in G.P., then x, 4, z will be in (c) 6 (d) 8
(a) A.P. (b) G.P. 2
(c) H.P. (d) None 29. The harmonic mean between two numbers is 14 and the
5
21. If the sum of n terms of G.P. is S, product is P and sum of geometric mean is 24. The greatest number between them
2
their inverses is R, then P = is :
(a) R/S (b) S/R (a) 72 (b) 36
n n
(c) (R/S) (d) (S/R) (c) 18 (d) 60
SEQUENCE AND SERIES 103

30. If a, b and c are positive real numbers, then the least value 37. The sum of the n terms of the series

1 1 1 1 + (1 + 3) + (1 + 3 + 5) ....
of (a + b + c)     is
a b c
2
2  n (n  1) 
(a) 9 (b) 3 (a) n (b)  
 2 
(c) 10/3 (d) none of these

a b c  n (n  1) (2n  1) 
31. If a, b and c are positive real numbers then   is (c)   (d) none of these
b c a  6 
greater than or equal to
38. The sum of the series 5.05 + 1.212 + 0.29088 + ...  is
(a) 3 (b) 6
(a) 6.93378 (b) 6.87342
(c) 27 (d) none of these
(c) 6.74384 (d) 6.64474
32. Let x be the arithmetic mean and y, z be the two geometric 2
means between any two positive number. Then value of 39. If the sum to n terms of a series be 5n + 2n, then second
term is
y3  z3
is (a) 15 (b) 17
xyz
(c) 10 (d) 5
(a) 2 (b) 3
n n
(c) 1/2 (d) 3/2
33. Six arithmetic means are inserted between 1 and 9/2, the 4
th
40. Let  r 1
r 4  f (n ), then  (2r  1)
r 1
4
is equal to

arithmetic mean is
(a) f (2n) – 16 f (n) (b) f (2n) –7 f (n)
(a) 2 (b) 1
(c) f (2n – 1) –8( f (n) (d) none of these
(c) 3 (d) 4
34.
2 2 2
The sum of series 1.3 + 2.5 + 3.7 + . . . upto 20 terms is 41. Coefficient of x99 in the polynomial (x – 1) (x – 2) ... (x – 100)
is
(a) 188090 (b) 189080
(a) 100 ! (b) – 5050
(c) 199080 (d) None
(c) 5050 (d) – 100
 n
35.  equals to: 1
n 1 4n4  1 42.
2 2 2 2
If x1  x 2  x 3  ....  x 50  50 and  A then
2 2
2
x x ...x 50
1 2
(a) 0 (b) 1
(c)  (d) 1/4 (a) Aminimum = 1 (b) Amaximum = 1
(c) Aminimum = 50 (d) Amaximum = 50
1
36. If nth term of a series is (n  1) (n  3) , then sum of infinite 43. Given p A.P.’s, each of which consists of n terms. If their
first terms are 1, 2, 3, ...., p and common differences are
terms of the series 1, 3, 5, ...., 2p – 1 respectively, then sum of the terms of all
the progressions is
(a) 3/2 (b) 1/2
(c) 5/2 (d) 5/12 1 1
(a) np (np  1) (b) n (p  1)
2 2
(c) np (n + 1) (d) none of these
104 SEQUENCE AND SERIES

44. If one G.M., g and two A.M.’s p and q are inserted between 51. A G.P. consist of even number of terms. If the sum of the
terms occupying the odd places is S1 and that of the terms
(2p  q) (p  2q)
two number a and b, then  in the even places is S2, then the common ratio of the G.P.
g2
is

(a) 1 (b) –1
S1 S2
(c) 2 (d) –3 (a) (b)
S2 S1
45. If x = 111 ... 1 (20 digits), y = 333...3 (10 digits) and z = 222...2

x  y2 2S1 S2
(10 digits), then  (c) (d)
z S2 2S1

2 2
(a) 1 (b) 2 52. If 5x – y, 2x + y, x + 2y are in A.P. and (x–1) , (xy + 1), (y+1)
are in G.P., x  0, then x + y =
1
(c) (d) 3
2 3
(a) (b) 3
4
3 5 9 17
46. The sum of the first 10 terms of     ... is (c) –5 (d) none of these
2 4 8 16
4
53. If the sum of m consecutive odd integers is m , then the
–10 –10
(a) 10 – 2 (b) 9 – 2 first integer is
–10
(c) 11 – 2 (d) none of these 3
(a) m + m + 1
3
(b) m + m – 1
3 3 3
47. The sum of the series 1 + 3 + 5 + ... to 20 terms is 3
(c) m – m – 1
3
(d) m – m + 1
(a) 319600 (b) 321760 54. The largest positive term of the H.P., whose first two terms
(c) 306000 (d) 347500
2 12
2 2 2 2 are and is
48. 1 – 2 + 3 – 4 + ... to 21 terms = 5 23
(a) 210 (b) 231

(c) –210 (d) – 231 13


(a) (b) 6
2
49. Consider the sequence 1, 2, 2, 3, 3, 3 ... where n occurs n
th
times. The number that occurs as 2007 term is
15
(a) 61 (b) 62 (c) (d) 8
2
(c) 63 (d) 64 2 2 2
55. Four distinct integers a, b, c d are in A.P. If a + b + c = d,
50. If the A.M. of two numbers a and b, a > b > 0 is twice their
then a + b + c + d =

a (a) 1 (b) 0
G..M., then 
b
(c) –1 (d) none of these

(a) 2  3 (b) 7  2 3

(c) 4  2 3 (d) 7  4 3
SEQUENCE AND SERIES 105

Assertion Reason Type Question 59. Assertion : The maximum number of acute angles in a
convex polygon of n sides is 3
(A) ASSERTION is true, REASON is true, REASON is a
correct explanation for ASSERTION. Reason : The sum of internal angles of any convex polygon
is (n – 2) 180°
(B) ASSERTION is true, REASON is true, REASON is not
a correct explanation for ASSERTION. (a) A (b) B (c) C

(C) ASSERTION is true, REASON is false (d) D (e) E

(D) ASSERTION is false, REASON is true 60. Assertion : The sum of an infinite A.G.P.

(E) ASSERTION is false, REASON is false a + (a + d) x + (a + 2d) x2 + (a + 3d) x3 +.........., where


| x | < 1 always exist.
56. Assertion : If all terms of a series with positive terms are
smaller than 10–5, then the sum of the series upto infinity Reason : The sum of the infinite series
will be finite. a + ar + ar2 + ......... converges if | r | < 1.
(a) A (b) B (c) C
n
Reason : If Sn  5 then nlim S is finite.
 n (d) D (e) E
10
Multiple
(a) A (b) B (c) C
61. If the first two terms of a progression are log2 256 and log3
(d) D (e) E
81 respectively, then which of the following statements
57. Assertion : If three positive numbers in G.P. represent sides are true :
of a triangle, then the common ratio of the G.P. must lie
(a) If third term is log4 16, then the terms are in G.P.
5 1 5 1 (b) If third term is 2 log6 1, then the terms are in A.P.
between and .
2 2
2
Reason : Three positive real numbers can form sides of a (c) If third term is log2 16, then the terms are in H.P..
3
triangle if sum of any two is greater than the third.
(d) If the third term is log2 8, then terms are in A.P.
(a) A (b) B (c) C
(d) D (e) E Subjective Type Question
58. Assertion : There exists an A.P. whose three terms are 62. The value of n for which
1.21 + 2.22 + 3.23 + .......+ n.2n = 2n + 10 + 2, is
2 , 3 , 5.
15
Reason : There exists distinct real numbers p, q, r satisfying 1 1 1 135
63. If Sn    then Sn  , then the
2n n  1 2(n  2) n 1 k
2 = A + (p – 1) d, 3 = A + (q – 1) d,
numerical quantity k must be

5 = A + (r – 1) d. 64. The only integer solution of the equation


(x–1)3 + (x–2)3 + (x – 3)3 + ...... + (x – 2007)3 = 0 is ...........
(a) A (b) B (c) C
65. Two consecutive numbers from 1, 2, 3 .... n are removed.
(d) D (e) E
105
The arithmetic mean of remaining n – 2 numbers is .
4
Then n must be
106 SEQUENCE AND SERIES
4 3 2
66. The sum of the squares of three distinct real numbers, 71. If the equation x – 4x + ax + bx + 1 = 0 has four positive
2
which are in G.P., is S . If their sum is S, show that roots, then find a and b.

1  72. If a, b, c are different positive numbers prove that


 2   ,1  1, 3 . 4 4 4
a + b + c > abc (a + b + c).
3 
73. If x, y, z are positive real numbers satisfying the equation
2 2 2
1 4
2 3 4 4
n 4 x + 9y + 25z = 3xy + 15 yz + 5zx then find the progression
67. Show that 1.3  3.5  5.7  ....  2n  1 2n  1 of x, y and z.
  
Match the Column


 2
n 4n  6n  5  n
.
74. Match the conditions for the equation ax3 + bx2 + cx +d=0
having roots in
48 16  2n  1
Column - I Column - II
3 3
1 2 3 (A) AP (P) b d=ac
68. Find the sum of the series    ... n terms 3 2 3
1.3 1.3.5 1.3.5.7 (B) GP (Q) 27ad =9bcd – 2c d
3 2
69. A sequence of real numbers a1, a2, a3, ..., an is such that (C) HP (R) 2b – 9abc + 27a d = 0
a1 = 0, |a2| = |a1 + 1|, |a3| = |a2 + 1|, ...., |an| = |an – 1 + 1|. 75. Match the following. If a, b, c are in HP, then
Column - I Column - II
1 n  1
Prove that   ai    .
n  i1  2 a b c
(A) , , (P) HP
bca ca b a bc
70. For positive real numbers x, y, z prove that

 x  yz   x  y z 
1 1 1
 x 2  y2  z2   xyz (B) , , (Q) GP
 
x
X Y Z  y

z ba b bc
 xyz   3 
b b b
(C) a , ,c (R) AP
2 2 2

a b c
(D) , ,
bc ca ab
107 SEQUENCE AND SERIES

EXERCISE - 4 : PREVIOUS YEAR JEE ADVANCED QUESTIONS

Single Type Questions 8. If a, b, c, d and p are distinct real numbers such that
1. The series of natural numbers is divided into group (a2 + b2 + c2) p2 – 2(ab + bc + dc) p + (b2 + c2 + d2) < 0, then
(1); (2, 3, 4); (5, 6, 7, 8, 9); . . . and so on. The sum of number a, b, c, d : (1987)
in the nth group is (1963) (a) are in A. P. (b) are in G. P.
3 3 3 2
(a) (n + 1) – n (b) (n + 1) – (n + 2) (c) are in H. P. (d) satisfy ab = cd
3 3
(c) (n – 1) + n (d) None 9. In a triangle the lengths of the two larger sides are 10 and
2. The sum of first three terms of a G.P. is to the sum of first six 9, respectively. If the angles are in A. P., then the length of
terms as 125 : 152. The common ratio of G.P. is the third side can be : (1987)
(1974) (a) (b) 3 3
91
1 2 (c) 5 (d) none of these
(a) (b)
5 5
1 3 7 15
10. Sum of the first n terms of the series     ......
3 4 2 4 8 16
(c) (d) is equal to : (1988)
5 5
(a) 2n – n – 1 (b) 1 – 2–n
3 3 3 3 3 3
1 1 2 1 2 3 (c) n + 2–n – 1 (d) 2n + 1
3. Sum of the series    ... to 16
1 1 3 1 3  5
11. If the first and (2n – 1)th terms of an A.P., G.P. and H.P. are
terms is (1976) equal and their nth terms are respectively a, b, c, then
(a) 346 (b) 446 (1988)
(c) 546 (d) None (a) a  b  c (b) a + c = b
2
4. The third term of a G.P. is 4. The product of the first five (c) ac – b = 0 (d) (a) and (c) both.
terms is : (1982) 12. Ths sum of first n terms of given series
(a) 43 (b) 45
2 2 2 2 2 2 n
(c) 44 (d) none of these 1 + 2.2 + 3 + 2.4 + 5 + 2.6 + . . . is (n  1) 2 , when n is
2
5. If sum of integers from 1 to 100 that are divisible by 2 or 5 even. When n is odd the sum will be (1988)
is (1984)
(a) 3000 (b) 3050 n n2
(a) (n  1) 2 (b) (n  1)
(c) 4050 (d) None 2 2
2
6. If a, b, c are in A.P., then straight line ax + by + c = 0 will (c) n (n + 1) (d) None
always pass through the point (1984)
x  7
(a) (–1, –2) (b) (1, –2) 13. If log3 2, log3 (2 – 5) and log3  2 x   are in A.P., then x =
 2
(c) (–1, 2) (d) (1, 2)
7. If a, b, c are in G. P. then the equations ax2 + 2bx + c = 0 and (1990)

d e f 1 1
dx2 + 2ex + f = 0 have a common root if , , are in : (a) 1, (b) 1,
a b c 2 3
(1985)
(a) A. P. (b) G. P. 3
(c) 1, (d) None
2
(c) H. P. (d) none of these
SEQUENCE
108 AND SERIES 108
SEQUENCE AND SERIES

14. The product of n positive numbers is unity, then their sum 21. Let Tr be the rth term of an A. P., for r = 1, 2, 3........ if for some
is : (1991)
1 1
(a) positive integer (b) divisible by n positive integers m, n we have Tm  and Tn  then
n m
1 Tmn equals : (1998)
(c) equal to n  (d) never less than n
n
1 1
15. Let an be nth term of G.P. of positive numbers. Let (a) 1/mn (b) m  n
100 100
(c) 1 (d) 0
 a 2 n   and a 2 n 1   such that a  b, then
n 1 n 1 22. If x > 1, y > 1, z > 1 are in G. P.,
common ratio is (1992)
1 1 1
  then 1  n x , 1  n y , 1  n z are in : (1998)
(a) (b)
 
(a) A. P. (b) H. P.
1/ 2 1/ 2
  (c) G. P. (d) none of the above
(c)   (d)  
  23. If x1, x2, x3 as well as y1, y2, y3 are in G. P. with the same
16. The sum of first p terms of an A.P. is q and the sum of first common ratio, then the points (x1, y1), (x2, y2) and (x3, y3) :
q terms is p. The sum of first (p + q) terms is (1993) (1999)
(a) p + q (b) 0 (a) lie on a straight line (b) lie on an elipse
(c) – (p + q) (d) – 2 (p + q) (c) lie on a circle (d) are vertices of triangle
17. If ln (a + c), ln (c – a), ln (a – 2b + c) are in A.P., then 24. The harmonic mean of the roots of the equation
(1994)
(a) a, b, c are in A.P.
2 2 2
(b) a , b , c are in A.P.
5  2  x   4  5  x  8  2
2
5  0 is: (1999)

(c) a, b, c are in G.P. (d) a, b, c are in H.P. (a) 2 (b) 4


18. Let n (> 1) be a positive integer, then the largest integer m (c) 6 (d) 8
m
such that (n + 1) divides
2 127
25. Let a1, a2 ....... a10 be in A. P. and h1, h2, ........ h10 be in H. P.
(1 + n + n + . . .+ n )is (1995) If a1 = h1 = 2 and a10 = h10 = 3, then a4h7 is : (1999)
(a) 32 (b) 63
(a) 2 (b) 3
(c) 64 (d) 127
(c) 5 (d) 6
19. If p, q, r are in A.P. and are positive, then roots of the
2
quadratic equation px + qx + r = 0 are all real for (1995) 26. If a, b, c and d are positive real numbers such that
a + b + c + d = 2, then M = (a + b) (c + d) satisfies the
r p relation : (2000)
(a) p  7  4 3 (b)  7  4 3
r
(a) 0 < M < 1 (b) 1 < M < 2
(c) All p and r (d) No p and r (c) 2 < M < 3 (d) 3 < M < 4
20. For any odd integer n  1, 27. Consider an infinite geometric series with first term a and
3 3 n–1 3
n – (n – 1) + . . . + (–1) 1 = (1996) common ratio r. If its sum is 4 and the second term is 3/4,
then : (2000)
1 2 (n  1) 2
(a) (n –1) (2n – 1) (b) (2n  1)
2 4 4 3
(a) a  , r  (b) a = 2, r = 3/8
7 7
(n  1) 2 1 2
(c) (2n  1) (d) (n + 1) (2n – 1) (c) a = 3/2, r = 1/2 (d) a = 3, r = 1/4
2 4
SEQUENCE AND SERIES 109

28. Let  be the roots of x2 – x + p = 0 and  be the roots 36. If the sum of first n terms of an A.P. is cn2, then the sum of
of x2 – 4x + q = 0 If  are in G. P., then the integer squares of these n terms is : (2009)
values of p and q respectively are : (2001)
n (4n 2  1)c 2 n (4n 2  1)c 2
(a) –2, –32 (b) – 2, 3 (a) (b)
6 3
(c) –6, 3 (d) –6, –32
2 2 2 2
29. If the sum of the first 2n terms of the A.P. 2, 5, 8, ........ is
(c) n (4n  1)c (d) n (4n  1)c
equal to the sum of the first n terms of the A. P. 57, 59, 61 3 6
......... then n equals : (2001)
37. Let a1, a2, a3,... be in a harmonic progression with a1 = 5 and
(a) 10 (b) 12 a20 = 25. The least positive integer n for which an < 0 is
(c) 11 (d) 13 (2012)
30. Let the positive numbers a, b, c, d be in A. P. Then abc, (a) 22 (b) 23
abd, acd, bcd are : (2001)
(c) 24 (d) 25
(a) in H.P. (b) in A. P.
38. Let bi > 1 for i = 1, 2, …., 101. Suppose loge b1, loge b2, …..,
(c) in G. P. (d) none of these loge b101 are in Arithmetic Progression (A.P.) with the
31. If a1, a2 .......... an are positive real numbers whose product common difference loge 2. Suppose a1, a2, …, a101 are in
A.P. such that a1 = b1 and a51 = b51. If t = b1 + b2 + … + b51
is a fixed number c, then the minimum value of
and s = a1 + a2 + … + a51, then (2016)
a1 + a2 + ........ + an–1 + 2an is : (2002)
(a) s > t and a101 > b101 (b) s > t and a101 < b101
(a) n (2c)1/n (b) (n + 1)c1/n
(c) s < t and a101 > b101 (d) s < t and a101 < b101
(c) 2nc1/n (d) (n + 1) (2c)1/n
Assertion Reason Type Questions
32. Suppose a, b, c are in A. P. and a2, b2, c2 are in G. P.
If a < b < c and a + b + c = 3/2, then the value of a is : (A) If ASSERTION is true, REASON is true, REASON is a
(2002) correct explanation for ASSERTION.
(B) If ASSERTION is true, REASON is true, REASON is
1 1 not a correct explanation for ASSERTION.
(a) (b)
2 2 2 3
(C) If ASSERTION is true, REASON is false

1 1 1 1 (D) If ASSERTION is false, REASON is true


(c)  (d) 
2 3 2 2 39. Suppose four distinct positive numbers a1, a2, a3, a4 are in
G.P. Let b1 = a1, b2 = b1 + a2, b3 = b2 + a3 and b4 = b3 + a4.
  tan 2  is always greater Assertion : The numbers b1, b2, b3, b4 are neither in AP nor
33. If    0,  then x2  x 
 2 x2  x in G. P.
than or equal to : (2003) Reason : The numbers b1, b2, b3, b4 are in HP. (2008)
(a) 2 tan  (b) 1 Paragraph Type Questions
(c) 2 (d) sec2  PASSAGE–1
34. An infinite G.P. has first term ‘x’ and sum 5, then x belongs Let Vr denote the sum of the first r terms of an arithmetic
to : (2004) progression (A.P.) whose first term is r and the common
difference is (2r – 1). Let Tr = Vr+1 – Vr – 2 and Qr = Tr+1 – Tr
(a) x < – 10 (b) –10 < x < 0
for r = 1, 2, ... (2007)
(c) 0 < x < 10 (d) x > 10
40. The sum V1 + V2 + ... + Vn is :
35. ,are roots of ax2 + bx + c = 0, a 0 and  = b2 – 4ac.
If  and  are in G. P., then : (2005) 1 1
(a) n (n  1) (3n 2  n  1) (b) n (n  1) (3n 2  n  2)
(a)  0 (b) b = 0 12 12
(c) c = 0 (d) bc 0 1 1
(c) n (2n 2  n  1) (d) (2n 3  2n  3)
2 3
110 SEQUENCE AND SERIES

41. Tr is always : 48. For a positive integer n let (1998)


(a) an odd number (b) an even number
1 1 1 1
(c) a prime number (d) a composite number a (n)  1     ....  n , then
2 3 4 (2 )  1
42. Which one of the following is a correct statement ?
(a) a (100)  100 (b) a (100) > 100
(a) Q1, Q2, Q3, ... are in A.P. with common difference 5
(c) a (200)  100 (d) a (200) > 100
(b) Q1, Q2, Q3 ... are in A.P. with common difference 6
49. Let S1, S2... be squares such that for each n  1 the length
(c) Q1, Q2, Q3 ... are in A.P. with common difference 11
of a side of Sn equals the length of a diagonal of Sn+1. If the
(d) Q1 = Q2 = Q3 = ... length of a side of S1 is 10 cm, then for which of the
following values of n is the area of Sn less than 1 sq. cm ?
PASSAGE–2
(1999)
Let A1, G1, H1 denote the arithmetic, geometric and
(a) 7 (b) 8
harmonic means, respectively, of two distinct positive
numbers. For n  2, let An–1 and Hn–1 has arithmetic, (c) 9 (d) 10
geometric and harmonic means as An, Gn, Hn respectively. 4n k (k 1)

(2007) 50. Let Sn   (1) 2


k 2 . Then, Sn can take value(s)
1
43. Which one of the following statements is correct ?
(2013)
(a) G1 > G2 > G3 > ...
(a) 1056 (b) 1088
(b) G1 < G2 < G3 < ...
(c) 1120 (d) 1332
(c) G1 = G2 = G3 = ...
Integer Answer Type Questions
(d) G1 < G3 < G5 < ... and G2 > G4 > G6 > ...
51. Let Sk, k = 1, 2, .... 100, denote the sum of the infinite
44. Which of the following statements is correct ?
(a) A1 > A2 > ... k 1
geometric series whose first term is and the common
k!
(b) A1 < A2 < A3 < ...
(c) A1 > A3 > A5 > ... and A2 < A4 < A6 < ... 1
ratio is . Then the value of
(d) A1 < A3 < A5 < ... and A2 > A4 > A6 > ... k
45. Which of the following statements is correct ?
1002 100 2
(a) H1 > H2 > H3 >...   | (k  3k  1) | Sk is... (2010)
100 ! k 1
(b) H1 < H2 < H3 < ...
(c) H1 > H3 > H5 > ... and H2 < H4 < H6 <... 52. Let a 1 , a 2 , a 3 ,..., a 11 be real numbers satisfying
a1 = 15, 27–2a2 > 0 and ak = 2ak–1 – ak – 2 for k = 3, 4,..., 11. If
(d) H1 < H3 < H5 < ... and H2 > H4 > H6 >...
Multiple Type Questions a12  a 22  ...  a11
2
a  a 2  ...  a11
 90, then the value of 1
11 11
46. If the first and the (2n – 1) th term of an AP, GP and HP are
equal and their nth terms are a, b and c respectively, then is equal to ..... (2010)

(1988) 53. Let a1, a2, a3,..., a100 be an arithmetic progression with a1 = 3
p
(a) a = b = c (b) a  b  c
and Sp   a i , 1  p  100. For any integer n with
2
(c) a + c = b (d) ac – b = 0 i 1

47. If the ratio of H.M. and G.M. between two numbers a and b Sm
is 4 : 5, then the ratio of two numbers will be (1992) 1  n  20, let m = 5n. If does not depend on n, then a2
Sn
(a) 1 : 2 (b) 2 : 1
is ...... (2011)
(c) 4 : 1 (d) 1 : 4
SEQUENCE AND SERIES 111

54. The minimum value of the sum of real numbers 63. Let X be the set consisting of the first 2018 terms of the
–5 –4 –3 8 10 arithmetic progression 1, 6, 11, ...., and Y be the set
a , a , 3a , 1,a and a with a > 0 is ... (2011)
consisting of the first 2018 terms of the arithmetic
b progression 9, 16, 23, ... . Then, the number of elements in
55. Let a,b,c, be positive integers such that is an integer. If the set X  Y is _______. (2018)
a
a,b,c are in geometric progression and the arithmetic mean True/False
a 2  a  14 64. If three complex numbers are in AP. Then, they lie on a
of a,b,c is b + 2, then the value of is (2014)
a 1 circle in the complex plane. (1985)
56. Suppose that all the terms of an arithmetic progression Subjective Type Questions
(A.P.) are natural numbers. If the ratio of the sum of the 65. (a) The vaue of x + y + z is 15 if a, x, y, z, b are in AP while the
first seven terms to the sum of the first eleven terms is 6 :
11 and the seventh term lies in between 130 and 140, then 1 1 1 5
value of   is if a, x, y, z, b are in HP,,
the common difference of this A.P. is. (2015) x y z 3
Fill in the Blanks
then find a and b.
57. The sum of integers from 1 to 100 that are divisible by
(b) If x, y, z are in HP, then show that
2 or 5 is.... (1984)
log (x + z) + log (x + z – 2y) = 2 log (x – z) (1978)
58. The sum of the first n terms of the series
66. If the mth, nth and pth terms of an AP and GP are equal
2
n (n  1) and are x, y, z then prove that xy–z . yz–x. zx–y = 1. (1979)
12 + 2 . 22 + 32 + 2.42 + 52 + 2.62 + ... is , when n is
2 67. The interior angles of a polygon are in arithmetic
even. When n is odd, the sum is ... . (1988) progression. The smallest angle is 120° and the common
59. Let the harmonic mean and geometric mean of two positive difference is 5°. Find the number of sides of the polygon.
numbers be in the ratio 4 : 5. Then, the two numbers are in
(1980)
the ratio... . (1992)
68. Let the angles A, B, C of a triangle ABC be in AP and let
60. Let x be the arithmetic mean and y, z be the two geometric
means between any two positive numbers, then b : c  3 : 2 . Find the angle A. (1981)

y3  z 3 69. (a) If a1, a2, ....., an are in arithmetic progression, where ai > 0 for
 ... . (1997)
xyz
1 1
61. If x is not an integral multiple of  and all i show that   ........
a1  a 2 a2  a3
cos (x – y), cos x and cos (x + y) are in HP.

y
Then find cos x sec   (1997) 1 n 1
2    
a n 1  a n a1  a n (1982)
2
62. Let p and q be the roots of the equation x – 2x + A = 0 and
let r and s be the roots of the equation x2 – 18x + B = 0. If (b) Does there exist a geometric progression containing 27,8
p < q < r < s are in arithmetic progression, then A = ... and
and 12 as three of its terms ? If it exists how many such
B = ... . (1997)
progression are possible ? (1982)
112 SEQUENCE AND SERIES

70. Find three numbers a, b, c between 2 and 18 such that (i) 77. Let a1, a2, .... be positive real numbers in geometric
their sum is 25 (ii) the numbers 2, a, b are consecutive progression. For each n, let An, Gn, Hn be respectively, the
arithmetic mean, geometric mean, and harmonic mean of
terms of an AP and (iii) the numbers b, c 18 are consecutive
a1, a2, ...., an . Find an expression for the geometric mean of
terms of a GP. (1983)
G1, G2,..., Gn in terms of A1, A2,..., An, H1, H2,..., Hn. (2001)
71. The sum of the squares of three distinct real numbers,
78. Let a, b be positive real numbers. If a,A1, A2, b are in
which are in GP, is S2. If their sum is aS then show that arithmetic progression, a,G1, G2, b are in geometric
progression and a,H1,H2,b are in harmonic progression,
2 1 
a   , 1  (1,3) (1986)
3  G1G 2 A1  A 2 (2a  b) (a  2b)
show that   (2002)
H1H 2 H1  H 2 9ab

 7 79. If a, b, c are in AP, a2, b2, c2 are in HP, then prove that either
72. If log3 2, log3 (2x – 5) and log3  2 x   are in arithmetic
 2
c
a = b = c or a, b, – form a GP (2003)
2
progression, determine the value of x. (1991)

73. Let p be the first of the n arithmetic means between two 80. If a, b, c are positive real numbers, then prove that

numbers and q be the first of n harmonic means between {(1 + a) (1 + b) (1 + c)}7 > 77 a4b4c4 (2004)
the same numbers. Show that q does not lie between 2 3 n
3 3 3 3
81. Let A n           ....  (1)n 1  
 n 1 .
2 4 4 4 4
p and   p (1991)
 n 1
Bn = 1 – An. Find a least odd natural number n0, so that
Bn>An,  n  n0. (2006)
74. If S1, S2, S3, .... , Sn are the sums of infinite geometric series,
whose first terms are 1, 2, 3, ...., n and whose common 82. The value of

1 1 1 1
ratios are , , ,......., respectively, then find the  
2 3 4 n 1 1 1 1 1
6  log 3 / 2  4 4 4 ...  is
3 2 3 2 3 2 3 2 
2 2 2 2  
values of S  S  S  .....  S
1 2 3 2n 1 . (1991)

(2012)
75. The real numbers x 1, x2, x 3 satisfying the equation
x3 – x2 + x + = 0 are in AP. Find the intervals in which 83. A pack contains n card numbered from 1 to n. Two
and lie. (1996) consecutive numbered card are removed from the pack
and the sum of the numbers on the remaining cards is
76. The fourth power of the common difference of an arithmetic
1224. If the smallest of the numbers on the removed cards
progression with integer entries is added to the product
is k, then k – 20 is equal to (2013)
of any four consecutive terms of it. Prove that resulting
84. The sides of a right angled triangle are in arithmetic
sum is the square of an integer. (2000) progression. If the triangle has area 24, then what is the
length of its smallest side ? (2017)
113 SEQUENCE AND SERIES

ANSWER KEY
EXERCISE - 1 : BASIC OBJECTIVE QUESTIONS
1. (b) 2. (c) 3. (c) 4. (c) 5. (c) 6. (b) 7. (d) 8. (a) 9. (b) 10. (c)
11. (b) 12. (b) 13. (b) 14. (c) 15. (c) 16. (a) 17. (a) 18. (d) 19. (b) 20. (d)
21. (b) 22. (c) 23. (c) 24. (b) 25. (a) 26. (c) 27. (a, c) 28. (a) 29. (c) 30. (b)
31. (c) 32. (b) 33. (b) 34. (a) 35. (a) 36. (c) 37. (c) 38. (c) 39. (b) 40. (a)
41. (b) 42. (d) 43. (b) 44. (b) 45. (b) 46. (b) 47. (a) 48. (b) 49. (a) 50. (c)
51. (c) 52. (c) 53. (a) 54. (a) 55. (d) 56. (c) 57. (b) 58. (c) 59. (b) 60. (c)
61. (c) 62. (c) 63. (a) 64. (c) 65. (b) 66. (b) 67. (b) 68. (a) 69. (a) 70. (a)
71. (b) 72. (a) 73. (c) 74. (d) 75. (d) 76. (a) 77. (b) 78. (a) 79. (b) 80. (d)

EXERCISE - 2 : PREVIOUS YEAR JEE MAINS QUESTIONS


1. (b) 2. (b) 3. (b) 4. (b) 5. (c) 6. (b) 7. (a) 8. (d) 9. (c) 10. (d)
11. (b) 12. (c) 13. (a) 14. (b) 15. (c) 16. (c) 17. (b) 18. (d) 19. (c) 20. (a)
21. (a) 22. (d) 23. (c) 24. (a) 25. (c) 26. (b) 27. (a) 28. (c) 29. (b) 30. (d)
31. (d) 32. (d) 33. (a) 34. (d) 35. (a) 36. (a) 37. (d) 38. (a) 39. (b) 40. (d)
41. (d) 42. (a) 43. (a) 44. (d) 45. (a) 46. (c) 47. (c) 48. (b) 49. (d) 50. (a)

EXERCISE - 3 : ADVANCED OBJECTIVE QUESTIONS


1. (a) 2. (a) 3. (a) 4. (a) 5. (a) 6. (c) 7. (d) 8. (c) 9. (c)
10. (a) 11. (b) 12. (a) 13. (b) 14. (a) 15. (b) 16. (a) 17. (d) 18. (d)
19. (b) 20. (c) 21. (d) 22. (a) 23. (c) 24. (b) 25. (c) 26. (c) 27. (a)
28. (b) 29. (a) 30. (a) 31. (a) 32. (a) 33. (c) 34. (a) 35. (d) 36. (d)
37. (c) 38. (d) 39. (b) 40. (a) 41. (b) 42. (a) 43. (a) 44. (b) 45. (a)
46. (c) 47. (a) 48. (b) 49. (c) 50. (d) 51. (b) 52. (a) 53. (d) 54. (b)
55. (d) 56. (e) 57. (a) 58. (d) 59. (b) 60. (a) 61. (a,b,c) 62. (0513) 63. (0544)
64. (1004) 65. (0050)
1 1 
68. 1   71. (a = 6, b = –4) 73. (x, y, z are in H.P.)
2  1.3.5... (2n  1) 
74. A–R; B–P; C–Q 75. A–P; B–R; C–Q; D–P

EXERCISE - 4 : PREVIOUS YEAR JEE ADVANCED QUESTIONS


1. (c) 2. (c) 3. (b) 4. (b) 5. (b) 6. (b) 7. (a) 8. (b) 9. (d) 10. (c)
11. (c) 12. (b) 13. (d) 14. (d) 15. (a) 16. (c) 17. (d) 18. (c) 19. (a) 20. (d)
21. (c) 22. (b) 23. (a) 24. (b) 25. (d) 26. (a) 27. (d) 28. (a) 29. (c) 30. (a)
31. (a) 32. (d) 33. (a) 34. (c) 35. (c) 36. (c) 37. (d) 38. (b) 39. (c) 40. (b)
41. (d) 42. (b) 43. (c) 44. (a) 45. (b) 46. (d) 47. (c,d) 48. (a,d) 49. (b,c,d) 50. (a,d)
2
51. 3 52 .0 53. 3, or 9 54. 8 55. (4) 56. (9) 57. 3050 58. n (n  1) 59. 4 : 1
2
60. 2 61.  2 62. A = –3, B = 77 63. (3748) 64. False 65. (a) a = 1, b = 9 67. 9 68. 75°
69. (b) infinite 70. a = 5, b = 8, c = 12 72. x = 3
1 1 1
74. (2n) (2n  1) (4n  1)  1 75.    ,  and     ,   77. Gm = (A1A2....AnH1H2...Hn)1/2n
6  3  27 
81. 7 82. 4 83. 5 84. (6)

Dream on !!

04
BINOMIAL THEOREM
BINOMIAL THEOREM 115

BINOMIAL THEOREM

BINOMIAL THEOREM n (n  1) (n  2) n 3 3  n (n  1)...(n  r  1)


 a b  ...
3! r! an–r br + ......
If a, b R and n N, then
(a + b)n = nC0 anb0 + nC1 an–1 b1 + nC2 an–2b2 +...+ nCn a0bn (n  1) (n  2)...(n  r  1) n r r
Here t r 1  r!
a b

REMARKS :

1. If the index of the binomial is n then the expansion THEOREM :


contains n + 1 terms.
If n is any real number, a = 1, b = x and |x| < 1 then
2. In each term, the sum of indices of a and b is always n.
3. Coefficients of the terms in binomial expansion n (n  1) 2 n (n  1) (n  2) 3
equidistant from both the ends are equal. (1 + x)n = 1 + nx + x  x  ...
2! 3!
4. (a–b)n = nC0anb0 – nC1an–1 b 1 + nC2an–2b 2– ...+ (–1)n
n
C0a0bn. Here there are infinite number of terms in the expansion,
The general term is given by
GENERAL TERM AND MIDDLE TERMS IN
EXPANSION OF (A + B)N n ( n  1) ( n  2)...(n  r  1) x r
t r 1  ,r  0
r!
tr+1 = nCr an–r br
tr+1 is called a general term for all r  N and 0  r  n.
Using this formula we can find any term of the expansion.
MIDDLE TERM (S) :
1. In (a + b)n if n is even then the number of terms in (i) Expansion is valid only when – 1 < x < 1
the expansion is odd. Therefore there is only one (ii) n
Cr can not be used because it is defined only for
th natural number, so nC r will be written
n2
middle term and it is   term. n (n  1).........(n  r  1)
 2  as
r!
2. In (a + b)n, if n is odd then the number of terms in
(iii) As the series never terminates, the number of terms
the expansion is even. Therefore there are two
in the series is infinite.
middle terms and those are
(iv) General term of the series (1 + x)–n = Tr + 1  (–1)r
th th
 n 1   n 3 1  x
  and   terms. if | x |  |
 2   2  1  x
(v) General term of the series (1 – x) –n  T r + 1
BINOMIAL THEOREM FOR ANY INDEX
n (n  1) (n  2)...(n  r  1) r
= x
If n is negative integer then n! is not defined. We state r!
binomial theorem in another form.
(vi) If first term is not 1, then make it unity in the
n n–1 n (n  1) n–2 2 x
(a+b)n = an + a b+ a b following way. (a  x ) n  a n (1  x / a ) n if 1
1! 2! a
116 BINOMIAL THEOREM

REMARKS :
BINOMIAL COEFFICIENTS
1. If |x| < 1 and n is any real number, then
The coefficients nC0, nC1, nC2,..., nCn in the expansion of (a+b)n
n (n  1) 2 n (n  1) (n  2) 3 are called the binomial coefficients and denoted by C0, C1,
(1–x)n = 1–nx + x  x  ...
2! 3! C2, ....., Cn respectively
Now
The general term is given by (1 + x)n = nC0x0 + nC1x1 + nC2x2 + ... + nCnxn ..... (i)
Put x = 1.
(1)r n(n  1)(n  2)...(n  r  1) r
tr 1  x (1 + 1)n = nC0 + nC1 + nC2 + ... + nCn
r!
 2n = nC0 + nC1 + nC2 +...+ nCn
n
2. If n is any real number and |b| < |a|, then  C0 + nC1 + nC2 +...+ nCn = 2n
 C0 + C1 + C2 +...+ Cn = 2n
  b 
n  The sum of all binomial coefficients is 2n.
 ( a  b ) n  a  1    Put x = –1, in equation (i),
  a 
(1–1)n = nC0 – nC1 + nC2 – ... + (–1)n nCn
n  0 = nC0 – nC1 + nC2 – ... + (–1)n nCn
 b
 a n 1    n
C0 – nC1 + nC2 – nC3 +....+ (–1)n nCn = 0
 a n
 C0 + nC2 + nC4 + ... = nC1 + nC3 + nC5 +...
 C0 + C2 + C4 +... = C1 + C3 + C5 +...
C0, C2, C4, ... are called as even coefficients
C1, C3, C5... are called as odd coefficients

While expanding (a + b)n where n is a negative integer or a Let C0 + C2 + C4 +... = C1 + C3 + C5 +... = k


fraction, reduce the binomial to the form in which the first Now C0 + C1 + C2 + C3+... + Cn = 2n
term is unity and the second term is numerically less than  (C0 + C2 + C4 +...) + (C1 + C3 + C5...) = 2n
unity.  k + k = 2n
Particular expansion of the binomials for negative index, 2k = 2n
|x|<1
2n
 k
1 2
1.  (1  x ) 1
1 x  k = 2n–1
 C0 + C2 + C4 + ... = C1 + C3 + C5 +... = 2n–1
= 1 – x + x2 – x3 + x4 – x5 + .....
 The sum of even coefficients = The sum of odd coefficients
1 = 2n–1
2.  (1  x ) 1
1 x
Properties of Binomial Coefficient
2 3 4 5
= 1 + x + x + x + x + x + .....
For the sake of convenience the coefficients
n
1 C0, nC1, ............, nCr,............... nCn are usually denoted by
3.  (1  x )  2
(1  x ) 2 C0, C1,......, Cr, .........., Cn respectively.
(i) C0 + C1 + C2 + ............ + Cn = 2n
= 1 – 2x + 3x2 – 4x3 + ..... (ii) C0 – C1 + C2 – ............ + (–1)n Cn = 0
(iii) C0 + C2 + C4 + ............ = C1 + C3 + C5 +.......... = 2n – 1.
1
4.  (1  x )  2 n
(1  x ) 2 (iv) C r1  n C r2  r1  r2 or r1  r2  n

= 1 + 2x + 3x2 + 4x3 + ..... (v) nCr + nCr – 1 = n + 1Cr


(vi) rn Cr = nn – 1Cr–1
BINOMIAL THEOREM 117

Some Important Results = (1 + x) (1 + x + x2 + x3 + x4 +...... to )


= [1 + x + x2 + x3 + x4 + ....... to ] +
n 2 n
(i) (1 + x) = C0 + C1x + C2x + .......... + Cnx , [x + x2 + x3 + x4+ ......... to ]
Putting x = 1 and – 1, we get
= 1 + 2x + 2x2 + 2x3 + 2x4 + 2x5 + ...... to 
C0 + C1 + C2 + ... + Cn = 2n and
Hence coefficient of x4 = 2
C0 – C1 + C2 – C3 + ..........(–1)n Cn = 0
(ii) Differentiating (1 + x)n = C0 + C1x + C2x2 + ........ + Cnxn, Illustration
on both sides we have, n(1 + x)n – 1
Find the square root of 99 correct to 4 places of deicmal.
= C1 + 2C2x + 3C3x2 + ....... +nCnxn – 1 ....(1)
x=1
1
 n2n – 1 = C1 + 2C2 + 3C3 + ........ + nCn 1/ 2 1/ 2   1  2
Sol. (99)  (100  1) 100 1  
x = –1   100 
 0 = C1 – 2C2 + ........ +(–1)n – 1 nCn.
Differentiating (1) again and again we will have 1

different results.   1  2
 100 1  
(iii) Integrating (1 + x)n, we have,   100 

(1  x ) n 1 C x 2 C x3 C x n 1 = (100)1/2 [1–0]1/2 = 10 (1 – 01)1/2


 C  C 0 x  1  2  .........  n
n 1 2 3 n 1
 1 11  
(where C is a constant)    1 
2 2
1 10 1  2 (01)    (01) 2  ........to  
 1! 2! 
Put x = 0, we get C = –  
(n  1)  
Therefore
= 10 [1–0.005 – 0.0000125 + ......... to ]
(1  x) n 1  1 C x2 C x3 C x n 1 ... (2) = 10 (.9949875) = 9.94987 = 9.9499
 C0 x  1  2  .........  n
n 1 2 3 n 1
Multinomial Expansion
Put x = 1 in (2) we get
In the expansion of (x1 + x2 +........ +xn)m where m, n  N and
2 n 1  1 C C
 C 0  1  .........  n x1, x2, ..........., xn are independent variables, we have
n 1 2 n 1
(i) Total number of terms = m + n – 1Cn – 1
Put x = – 1 in (2) we get,
(ii) Coefficient of x1r1 x 2 r2 x 3 r3 .........x n rn (where r1 + r2 +
1 C C
 C 0  1  2  .............
n 1 2 3 m!
........... + rn = m, ri  N  {0} is
Illustration r1!r2 !.......rn !

Find the coefficient of x 4 in the expansion of (iii) Sum of all the coefficients is obtained by putting all
the variables x1 equal to 1.
1 x
if | x |  1
1 x Illustration

Find the total number of terms in the expansion of


1 x (1 + a + b)10 and coefficient of a2b3.
Sol.  (1  x ) (1  x ) 1
1 x
Sol. Total number of terms = 10 + 3 – 1 C3 – 1 =12C2 = 66
(1) (1) (1  1)
 (1  x ) [1  ( x ) ( x ) 2
1! 2! 10!
Coefficient of a2b3 =
2! 3! 5! =2520
(1) (1  1) (1  2)
 ( x ) 3 .....to 
3!
118 BINOMIAL THEOREM

SOLVED EXAMPLES

Example – 1 Example – 4

If nC8 = nC6 determine n and hence nC2. Find n, if nC6 : n–3C3 = 33 : 4.

Sol. Given, nC8 = nC6 Sol. Given, nC6 : n–3C3 = 33 : 4.


n = 8 + 6

 n = 14 n! 3! n  3  3! 33
  
6! n  6 !  n  3 ! 4
14 13
Now nC2 = 14C2  2!
 91

Example – 2 n! 3! 33 n  n 1 n  2  33
or .  or 
 
n  3 ! 6! 4 6.5.4 4
If 15C3r = 15Cr+3, find r.

Sol. We know that if nCx = nCy, then x = y or x + y = n


or n(n – 1) (n – 2) = 6.5.33 = 11.3.3.2.5
15
C3r = 15Cr+3
or n(n – 1) (n – 2) = 11.(3.3).(2.5) = 11.10.9  n = 11
3
 Either 3r = r + 3  r= , Example – 5
2

5
which is not possible, since r is an integer. Find the value of the expression 47
C4   52  j
C3 .
j 1
or 3r + r + 3 = 15  r = 3.

Hence r = 3.
5
47 52  j
Example – 3 Sol. Given expression = C4   C3 .
j 1

5
Prove that  5Cr  31
r 1
= 47C4 + (51C3 + 50C3 + 49C3 + 48C3 + 47C3)

5 = 47C4 + (47C3 + 48C3 + 49C3 + 50C3 + 51C3)


Sol.  5Cr  5C1  5C2  5C3  5C4  5C5
r 1

= (47C4 + 47C3) + (48C3 + 49C3 + 50C3 + 51C3)

5! 5! 5! 5! 5!
     = 48C4 +48C3 + 49C3 + 50C3 + 51C3; [ 47C4 + 47C3 = 48C4]
1!4! 2!3! 3!2! 4!1! 5!10!

= (48C4 + 48C3) + (49C3 + 50C3 + 51C3)


5 5.4 5.4 5
    1
1 2 2 1 = (49C4 + 49C3) + (50C3 + 51C3)

 = (50C4 + 50C3) + 51C3 = 51C4 + 51C3 = 52C4.


BINOMIAL THEOREM 119

Example – 6 Example – 7
Expand Expand (1 + x + x2)3.
6
 1 Sol. Let y = x + x2. Then,
(i) (2x2 + 3)4 (ii)  2 x 2  
 x (1 + x + x2)3 = (1 + y)3 = 3C0 + 3C1 y + 3C2 y2 + 3C3 y3
= 1 + 3y + 3y2 + y3 = 1 + 3 (x + x2) + 3 (x + x2)2 + (x + x2)3
Sol. (i) (2x2 + 3)4 =
4
C0 (2x2)4 (3)0 + 4C1 (2x2)3 (3)1 + 4C2 (2x2)2 = 1 + 3 (x + x2) + 3(x2 + 2x3 + x4) + {3C0x3 (x2)0 + 3C1 x3–1 (x2)1

(3)2 + 4C3 (2x2)1 (3)3 + 4C4 (2x2)0 (3)4 + 3C2x3–2 (x2)2 + 3C3 x0 (x2)3}
= 1 + 3 (x + x2) + 3(x2 + 2x3 + x4) + (x3 + 3x4 + 3x5 + x6)
4 4 4 4
 C0  C4 1, C1  C3  4
  = x6 + 3x5 + 6x4 + 7x3 + 6x2 + 3x + 1
  4 4! 4  3 2! 
C  
 2 2 ! 2 ! 2! 2  6  Example – 8
 

= (1) 16x8 (1) + 4 (8x6) (3) + 6 (4x4) (9) + 4 (2x2) 27 + (1) (1) 81 Prove that ( 5  1)5  ( 5  1)5  352
= 16x8 + 96x6 + 216x4 + 216x2 + 81
Sol. ( 5  1)5  ( 5  1)5 
6 0
 2 1 6 2 6 1
(ii)  2x    C0 (2x )   
 x x
 5 C0 ( 5 )5  5 C1 ( 5 ) 4 (1)  5C 2 ( 5 )3 (1) 2 
 
1
1
1
2
  5C3 ( 5 ) 2 (1)3  5C 4 ( 5 ) (1) 4 
(2x 2 )5   6 C1  6C 2 (2x 2 ) 4     5 0 5 
x x  C5 ( 5 ) (1)
 

3
6 1
C3 (2x )    6 C 4 (2x 2 )1
2 3

x  5 C 0 ( 5 ) 5  5 C1 ( 5 ) 4 (1)  5 C 2 ( 5 ) 3 (1) 2 


 
  5 C 3 ( 5 ) 2 (1) 3  5 C 4 ( 5 ) (1) 4 
4 5 6
1 6 2 1 6 2 01   5 C 5 ( 5 ) 0 (1) 5 
   C5 ( 2 x )    C 6 ( 2 x )    
x x x

   5C0  5C5  1; 5C 4  5; 
 6 6 6 6   
 C0  C6 1, C1  C5  6  5 5 5 .4 5 
 6! 6  5 4!   C 2  C3   10; C1  5
  6 C2   15   2.1 
 2!4! 2  4! 
6 6! 6  5 4  3! 
 C3    20 
 3!3! 3 2  3!  1(25) 5  5 (25)  10 (5 2 )  10 (5)
 
  5 5 1 
1 1
= (1) 64x12 (1) –(6) (32) x10 × + 15 (16) x8 × 2
x x
1(25) 5  5 (25)  10 (5 2 )  10 (5) 
1 1  
– 20 × 8x × 3 + 154 x4 × 4
6
  5 5 1 
x x
1 1
 6( 2x 2 )   (1) (1) 6  25 5  125  50 2  50  5 5  1
x5 x
12 1
= 64 x12 – 192x9 + 240x6 – 160x3 + 154 –   25 5  125  50 2  50  5 5  1 = 352
x3 x6
120 BINOMIAL THEOREM

Example – 9 Example – 12

Using binomial theorem compute (99)5. n


r n n
Prove that r 0 3 . Cr  4 .
Sol. (99)5 = (100–1)5 – 5C0 (100)5 – 5C1 (100)4 + 5C2 (100)3

Sol. (1 + x)n = nC0 x0 + nC1 x + nC2 x2 + ... + nCn xn


– 5C3 (100)2 + 5C4 (100)1 – 5C5 (100)0
n
 C0 x0 + nC1 x + nC2 x2 + .... + nCn xn = (1 + x)n ....(1)
5 4 3 2
= (100) –5 ×(100) + 10 × (100) – 10 × (100) + 5 × 100 – 1
n
Now  3r . n Cr
= 1010 – 5 × 108 + 107 – 105 + 5 × 102 – 1 r 0

= (1010 + 107 + 5 × 102) – (5 × 108 + 105 + 1) = nC0 30 + nC1 31 + nC2 32 + .... + nCn 3n

= (1 + 3)n
= 10010000500 – 500100001 = 9509900499
= 4n.
Example – 10

Use the binomial theorem to find the exact value of (10.1)5. Example – 13

11
Sol. (10.1)5 = (10 + 0.1)5  4
Find the fifith term in the expansion of  x 2  3 
 x 
= 105 + 5C1 104 (.1) + 5C2 103 (.1)2 + 5C3 102 (.1)3

4
+ 5C4 10 (.1)4 + 5C5 (.1)5 Sol. Let, a = x2, b  , n  11
x3
= 100000 + 5 × 104 (.1) + 10 × (103) (.01) + 10 × 102 (.001)
For fifth term, r = 4
+5 × 10 (.0001) + 0.00001
 tr+1 = nCr an–r . br
= 100000 + 5000 + 100 + 1 + 0.005 + 0.00001 = 105101. 00501

Example – 11 4
4

 t5 = 11C4 (x2)11–4  3 
Which number is larger, (1.2)4000 or 800 ? x 

Sol. (1.2)4000 = (1 + 0.2)4000


11 ! 2 7 4 4
= 4000C0 + 4000C1 (0.2) + sum of positive terms  t5  ( x )  12
4! 7! x
= 1 + 4000 (0.2) + a positive number

= 1 + 800 + a positive number


11 10  9  8  7 ! 14 256
 t5  x  12
> 800 4  3  2  7! x

Hence (1.2)4000 > 800.


 t5 = 330 × 256x2  t5 = 84480x2
BINOMIAL THEOREM 121

Example – 14 Example – 16
8 Find the middle term (s) in the expansion of
 3 
Find the third term in the expansion of  2 x 2  
 2 x 12 11
 1 
(i)  x  y  (ii)  x 4  3 
y x  x 
3  
Sol. Let a = 2x2, b  ,n  8
2x
For third term, r = 2 x y
tr+1 = nCr an–r br Sol. (i) Let a  , b  , n  12
y x
2
8 2 8 2  3  n is even
 C 2 (2x )  
 2x 
 n  2   12  2  14
8.7.6! 9  8!      7
 (2x 2 ) 6  2  8C 2    2   2  2
2!6! 4x  2!6!
7th term is middle term,
8.7 6 12 9
 2 x  2 tr+1 = nCr an–r . br
2 4x
= 63 × 64x = 4032x10
10 For 7th term, r = 6
Example – 15
12 6 6
x y
 1
7 t 7  13C 6    
Find the middle terms(s) in the expansion of  x 2   y x
 x
6
2
1 12!  x   y 
Sol. Let a = x , b = , n = 7 t7     
x 6!6 !  y   x 
n is odd

 n 1   7 1   n 3  73 12 1110  9  8  7  6 !


    4 and   5 t7 
 2   2   2   2  6  5  4  3 2  6 !
4th and 5th terms are middle terms, for t4, r = 3
t7 = 77 × 12 = 924
tr+1 = nCran–r . br
 Middle term = 924
3
7 2 7 3 1
t 4  C3 ( x )   1
x (ii) Let a = x4, b  , n  11
x3
7! 1
t4   (x 2 )4  3
4!3! x  n  1   11  1 
n is odd     6,
7  6  5  4! 8 1  2   2 
t4  x  3
4! 3  2 x
t4 = 35x5  n  3   11  3 
  7
For t5, r = 4  2   2 

4 6th and 7th term are middle term,


1
t 5  7 C 4 a 7 4 .b  
x tr+1 = nCran–r . br
For t6, r = 5
! 1
t5   x6  
4!3! x 5
 1 
t 6  11C5 ( x 4 )115  3 
7  6  5  4! 2 x 
t5   x  35x 2
4! 3  2
122 BINOMIAL THEOREM

11! 24   1  r
t6  x  15  9 9 r  1  9 3 r
5! 6! x  t r 1  C r (2)   x
3

11 10  9  8  7  6! To get term independent of x, must have


t6  ( x 9 )
5  4  3  2  1 6!
x9–3r = x0
t6 = –11 × 3 × 2 × 7x = –462 x9 9 – 3r = 0  – 3r = –9  r = 3
For t7, r = 6
3
 9 C (2) 93  1 
6 3
 1  3
t 7  11C6 ( x 4 )116  3 
x 
9! 1 9  8  7  6! 1
 26    64 
11! 20 1 3!6! 27 3  2  6! 27
t7  x  18
6!5! x
28  64 1792
1110  9  8  7  6! 2  
t7  x 9 9
6! 5  4  3  2
1792
t7 = –11 × 3 × 2 × 7 = – 462x2 Constant term independent of x 
9
11! 20 1
t7  x  18
6!5! x 2
(ii) Let a = x, b = , n = 15
x2
1110  9  8  7  6! 2
t7  x tr+1 = nCr an–r . br
6! 5  4  3  2
r
t7 = –11 × 3 × 2 × 7 = – 462 x2 2
15
tr+1 = Cr (x) 15–r  2
Example – 17 x 

Find the constant term (term independent of x) in the tr+1 = 15Cr (x)15–r (–2)r x–2r
expansion of
tr+1 = 15Cr (–2)r (x)15–3r
15
 1 
9
 2  To get constant term independent of x,
(i)  2x  2  (ii)  x  2 
 3x   x  x15–3r = x0
15 – 3r = 0  – 3r = – 15  r = 5
1
Sol. Let a = 2x, b  ,n  9
3x 2 15 15!
 C5 (2)5  (32)
5!10!
tr+1 = nCr an–r . br

r
 1  15 15 14  13  12 11 10!
t r 1  9 C r (2x )9r  2  C5(2) 5   32
 3x  5  4  3  2 10!

15
r
C5 (–2)5 = – 77×39×32= – 96096
1
tr 1  9Cr (2)9r   x2 r . x9r Constant term independent of
3
x = – 96096
BINOMIAL THEOREM 123

Example – 18 Example – 20
9 2 3 15
Find the coefficient of x in (1 + 3x + 3x + x ) .
n
Sol. (1 + 3x + 3x2 + x3)15 = [(1 + x)3]15 = (1 + x)45  1
Find the term independent of x in (1 + x)m  1  
 Coefficient of x9 in (1 + 3x + 3x2 + x3)15  x

= co-eff. of x9 in (1 + x)45
= 45C9 [Since in the expansion of (1 + x)n, n n
m 1 m  x 1 
coefficient of x = Cr]
r n Sol. Given expression  1  x  1    1  x   
 x  x 
5!

9!36! mn


1 x   x  n 1 x 
mn

Example – 19 xn
n
 1
Given that the 4th term in the expansion of  px    Required term independent of x
 x
= coefficient of x0 in x–n (1 + x)m+n
5
is , find n an p. = coefficient of xn in (1 + x)m+n
2

1
n
 mn
Cn 
m  n
Sol. Given expansion is  px   n!m!
 x

Given, T4  5 [Since in the expansion of (1 + x)n, co-efficient of xr = nCr]


2
Example – 21
3
n n 3 1 5 Find the coefficient of x5 in the expansion of the product
 C3  px    
x 2 (1 + 2x)6 (1 – x)7.

n 1 5 Sol. (1 + 2x)6 = [1 + 6C1 (2x) + 6C2 (2x)2 + 6C3 (2x)3 + 6C4 (2x)4
 C3 p n  3 x n  3 . 
x3 2
+ 6C5 (2x)5 + 6C6 (2x)6] .....(1)
n! 5
 .p n  3 x n  6  ...(1) Again, (1 – x)7 = 1 – 7C1 x + 7C2 x2 – 7C3 x3 + 7C4 x4 – 7C5 x5
3! n  3! 2
+ 7C6 x6 – 7C7 x7
Since R.H.S. of (1) is independent of x,
= 1 – 7x + 21x2 – 35x3 + 35x4 – 21x5 + 7x6 – x7 .....(2)
therefore n – 6 = 0  n = 6.
Now (1 + 2x)6 (1 – x)7
6! 3 5
From 1 , .p 
3! 3! 2 = (1 + 12x + 60x2 + 160x3 + 240x4 + 192x5 + .....)

× (1 – 7x + 21x2 – 35x3 + 35x4 – 21x5 + .....)


5
 20p3 
2  Required coefficient of x5 in the product

3 = 1 × (–21) + 12 × 35 + 60 × (–35)
1 1 3 1
 p    p .
8 2 2 + 160 × 21 + 240 × (–7) + 192 × 1

= –21 + 420 – 2100 + 3360 – 1680 + 192 = 171


1
Hence n = 6 and p = .
2
124 BINOMIAL THEOREM

Example – 22 Example – 24
If the coefficients of three consecutive terms in the
Show that 24n – 2n (7n + 1) is some multiple of the square of
expansion of (1 + a)n are in the ratio 1 : 7 : 42, find n.
14, where n is a postive integer.
Sol. Let the three consecutive terms in the expansion of (1+a)n
Sol. 24n – 2n (7n + 1) = (16)n – 2n (7n + 1)
be rth, (r + 1)th and (r + 2)th terms respectively.
n n n
= (2 + 14) – 2 . 7n – 2 In the expansion of (1 + a)n,

= (2n + nC1 2n–1 . 14 + nC2 2n–2 . 142 + ... + 14n) – 2n . 7n – 2n coefficient of rth term = nCr–1,

coefficient of (r + 1)th term = nCr.


= 142 (nC2 2n–2 + nC3 2n–3 14 + ... + 14n–2)
coefficient or (r + 2)th term = nCr+1
n n n–1 n n
+ (2 + C1 . 2 . 14 –2 . 7n – 2 )
Given, nCr–1 : nCr : nCr+1 = 1 : 7 : 42.
= 142 (nC2 2n–2 + nC3 2n–3 . 14 + ... + 14n–2)
n
C r 1 1
n n–1 n
+ (2 + n2 . 2 . 7 – 2 . 7n – 2 ) n  n

Cr 7

= 142 (nC2 . 2n–2 + nC3 . 2n–3 . 14 + ... + 14n–2)

n! r! n  r ! 1
This is divisible by 142 i.e. by 196 for all positive integral . 

 
r 1 ! n  r  1 ! n! 7
value of n.

Note : If n = 1, nC2 = 0, nC3 = 0 etc.


r 1
 
 Given expression = 142 ×0 =0, which is divisible by 196. n  r 1 7

Example – 23  7r = n – r + 1
n
Using binomial theorem, prove that 6 – 5n always  n – 8r = –1 ............ (1)
leaves the remainder 1 when divided by 25 for all positive
n
integers n. Cr 7
And n

Cr 1 42
Sol. 6n – 5n = (1 + 5)n – 5n

= (1 + nC1 5 + nC2 52 + ... + nCn 5n) – 5n


r!  r 1! n  r 1!  1
 r! n  r !.
= (1 + n . 5 + nC2 . 52 + ... + nCn 5n) – 5n
  n! 6

= 1 + nC2 52 + nC3 53 + ... + nCn 5n


r 1 1
 
= 1 + 25 (nC2 + nC3 . 5 + ... + nCn 5n–2) n r 6

= 1 + 25.k where k is a positive integer.  6r + 6 = n – r  n – 7r = 6 ............ (2)

 When 6n – 5n is divided by 25, remainder will be 1 for all Now, (2) – (1)  r=7

positive integer n. From (1), n = 55.


BINOMIAL THEOREM 125

Example – 25 Example – 26
If in the expansion of (1 + x)n, the coefficients of 14th, 15th Simplify first three terms in the expansion of the following
and 16th terms in A.P. find n.
(i) (1 + 2x)–4 (ii) (5+4x)–1/2
Sol. The coefficients of 14th, 15th and 16th tems in the
Sol. (i) (1 + 2x)–4 =
expansion of (1 + x)n will be nC13, nC14 and nC15 respectively.
Given, nC13, nC14 and nC15 are in A.P. (4) (4  1)
1  (4) (2x )  (2x ) 2
2!
Given, nC13, nC14 and nC15 are in A.P.
n
 C14 – nC13 = nC15 – nC14
(4) (5)
 1  8x  (4x 2 )
or 2 . nC14 = nC13 + nC15 2

n! n! n! = 1–8x + 40x2 + .....


or 2.  
14 ! n 14 ! 13! n 13! 15 ! n 15 !
1
1 1
2  4x  2
2
Multiplying both sides by 15! (n – 13)!, we get (ii) (5  4x )  5 1  
 5 

15! n 13! 15! n 13! 15! n 13!


2.  
14! n 14 ! 13! n 13! 15! n 15!
  1   1  

1    1 2 
  1   4x   2   2   4 x 
or 2.15 (n – 13) = 15.14 + (n – 13) (n – 14)  5 1      
2
   ...
  2  5  2!  5  
or 30n – 390 = 210 + n2 – 27n + 182  
 
or n2 – 57n + 782 = 0

or (n – 34) (n – 23) = 0

Hence n = 23 or 34.   1   3  
 2x  2   2  16x 2
1 
 5 1 
2      ...
 5 2 25 
 
 

1
 2x 6 x 2 
 5 2 1    ....
 5 25 
126 BINOMIAL THEOREM

EXERCISE - 1 : BASIC OBJECTIVE QUESTIONS


Binomial Co-efficient Binomial Theorem
1. Which of the following is not true ? 12. The expansion (x + a)n = nC0 xn + nC1 xn–1 a1 + ..... + nCn an is valid
(a) nCr = nCn–r (b) nCr + nCr–1 = n+1Cr when n is
(a) an integer (b) a natural number
(c) r. n Cr  n. n 1Cr 1 (d) nCr + nCr–1 = nCr+1
(c) a rational number (d) none of these
2. If n is a positive integer; then nC0 is equal to
(a) n (b) 0 13. ( x  x 3  1) 5  ( x  x 3  1 ) 5 is a polynomial of degree
(c) 1 (d) none of these (a) 5 (b) 6
2n n
3. If C3 : C2 : : 44 : 1, then the value of n is (c) 7 (d) 8
(a) 17 (b) 6 14. The total number of terms in the expansion of
(c) 11 (d) none of these (x + a)100 + (x – a)100 after simplification is
(a) 202 (b) 51
4. If n  2  210 n  1, then the value of n is equal to
(c) 50 (d) None
(a) 6 (b)5 4
15. (1.003) is nearly equal to
(c) 7 (d) none of these
(a) 1.012 (b) 1.0012
1 1 x (c) 0.988 (d) 1.003
5. If   , then x is equal to
8 9 10 16. The number of non-zero terms in the expansion of
(a) 100 (b) 90 9 9
 13 2 x 2 x  is
(c) 170 (d) none of these 

  1 3  
6. If nC3 = nC2, then n is equal to (a) 9 (b) 10
(a) 2 (b) 3 (c) 5 (d) None of these
(c) 5 (d) none of these General Term
7. If nC8 = nC6 , then nC2
18
3
(a) 21 (b) 20 17. The term void of x in the expanion of  x  2  is
(c) 91 (d) 28  x 
32 32
8. If C2n–1 = Cn–3 , then n = (a) 18C6 (b) 18C6 36
(a) 10 (b) 9 (c) 18C12 (d) 18C6 312
(c) 12 (d) 11 18. If n  N and the coefficients of x–7 and x–8 to the expansion of
9. If n+1C4 = 9 nC2 , then n = n
 1 
(a) 10 (b) 9  2   are equal then n =
 3x 
(c) 12 (d) 11
n n n
(a) 56 (b) 15
10. If Cr–1 = 36, Cr = 84 and Cr+1 = 126, then r =
(c) 45 (d) 55
(a) 1 (b) 2
19. If n,p  N and in the expansion of (1 + x)n the coefficient of
(c) 3 (d) 4
pth and (p + 1)th terms are respectively p and q. The p + q =
11. If nC4, nC5 and nC6 are in A.P., then possible value of n is
(a) n + 3 (b) n + 1
(a) 6 (b) 12
(c) n + 2 (d) n
(c) 14 (d) 21
BINOMIAL THEOREM 127

6
27. The term independent of x in the expansion of
 3 
20. The coefficient of x3 in  x 5   is :  t 1 1 x  t 1  1 1 8

 x3  
    x 1  is :

(a) 0 (b) 120
3 3
(c) 420 (d) 540  1 t   1 t 
(a) 56   (b) 56  
n
 1 t   1 t 
 1/3 1 
21. If the last term in the binomial expansion of  2   is
 2 4 4
 1 t   1 t 
(c) 70   (d) 70  
log 3 8  1 t   1 t 
 1 
 5/3  , then the 5th term form the beginning is :
3  28. The greatest value of the term independent of x, as  varies
20
(a) 210 (b) 420  sin  
over R, in the expansion of  x cos    is :
(c) 103 (d) None of these  x 

12 (a) 20C10 (b) 20C19


22. If rth term in the expansion of  x 2  1  is independent of
 x 10
x, then r = (c) 20C6 (d) 20C10  1 
2
(a) 9 (b) 8
29. If the sum of binomial coefficients in the expansion
(c) 10 (d) none of these
n
 1
12  2x   is 256, then term independent of x is
 x3 2   x
23. 5th term from the end in the expansion of   2  is
 2 x 
  (a) 1120 (b) 1020
(a) – 7920 x–4 (b) 7920 x4 (c) 512 (d) None
(c) 7920 x–4 (d) –7920 x4 30. The coefficient of x y in the expansion of (x + y)18 is
8 10

24. If the coefficients of (r + 4)th term and (2r + 1)th term in the (a) 18C8 (b) 18P10
expansion of (1 + x)18 are equal, then r = (c) 218 (d) None of these
(a) 3 (b) 5 0
  
(c) 3 or 5 (d) none of these 31. If the term independent of x in the expansion of  x  2 
 x 
9
 3  is 405, then  equals
25. The term independent of x in the expansion of  2x  2  is
 x  (a) –3 (b) 3
(a) 33 . 9C3 (b) 26 . 33 9C3 (c) 3 or –3 (d) None of these
(c) –33 . 9C3 (d) –26 . 33 . 9C3 10
 2x 2 3 
32. The middle term in the expansion of   2  is
 3
8 2x 
1 1 1
 
26. In the expansion of  x 3  x 5  , the term independent of
2 
  (a) 251 (b) 252
x is (c) 250 (d) None
(a) T5 (b) T7 33. If n  N, then middle term(s) in the expansion of (1 + x)2n+1 is (are)
(c) T6 (d) T8 (a) 2n+1Cn xn and 2n+1Cx+1 xn+1
(b) 2n+1Cn xn+1 and 2n+1Cn+1 xn
(c) 2n+1Cn xn
(d) 2n+1Cn+1 xn+1
128 BINOMIAL THEOREM

34. If T 2/T 3 in the expansion of (a + b) n and T 3/T 4 in the 43. The number of integral terms in the expansion of
(a + b)n+3 are equal, then n = 500
(a) 3 (b) 4  3 2  is :

(c) 5 (d) 6 (a) 128 (b) 129


35. If n  N and second, third and fourth terms in the expansion (c) 251 (d) 512
of (x + a)n are 240, 720 and 1080 respectively, then the value
44. The number of irrational terms in the expansion of
of n is
45
(a) 15 (b) 20 4 1/5
 71/10  is
(c) 10 (d) 5 (a) 40 (b) 5
36. If the coefficients of 5th, 6th and 7th terms in the expansion (c) 41 (d) none of these
of (1 + x)n, n  N, are in A.P., then n is equal to
45. If n  N and the number of dissimilar terms in the expansion
(a) 5 (b) 6 of (a + b +c)n is
(c) 7 (d) 7 or 14 (a) n + 1 (b) n
37. If n  N and (1+x) = 1 + a1x + a2x2 + ... + anxn. If a1, a2 and a3
n
(c) n + 2 (d) 1 + 2 + 3 +....+ (n + 1)
are in A.P., then the value of n is 99
46. The coefficient of x in
(a) 4 (b) 5
(x + 1) (x + 3) (x + 5) ..... (x + 199) is
(c) 6 (d) 7
(a) 1 + 2 + 3 + .... + 99 (b) 1 + 3 + 5 + .... + 199
38. a, b, c, d are any four consecutive co-efficients of any
(c) 1.3.5. ....... 199 (d) None of these
a b bc cd 17
binomial expansion, then , , are in : 47. The coefficient of x in the expansion of
a b c
(x – 1) (x – 2) ........ (x – 18) is
(a) A.P.
(a) 342 (b) –171
(b) G.P.
171
(c) H.P. (c) (d) 684
2
(d) arithmetico geometric progression
48. The sum of coefficient in the expansion of (1 + x – 3x2)3148 is
39. If (1 + ax)m =1 + 8x + 24x2 + ..., then the value of a and m are
respectively. (a) 8 (b) 7
(a) 4,2 (b) 2,4 (c) 1 (d) –1
(c) 1,8 (d) None of these 49. The sum of coefficient in (1 + x – 3x2)2134 is
40. Coefficient of x5 in the expansion of (1 + x2)5 (1 + x)4 is (a) –1 (b) 1
(a) 61 (b) 59 (c) 0 (d) 22134
(c) 0 (d) 60 50. In the expansion of (1 + x)50, the sum of the coeficient of odd
powers of x is
41. The co-efficient of x4 in the expansion of (1 + x + x2 + x3)n is :
(a) 0 (b) 249
(a) nC4 (b) nC4 + nC2
(c) 250 (d) 251
(c) nC4 + nC1 + nC4 . nC2 (d) nC4 + nC2 + nC1 . nC2
51. If the sum of the coefficients in the expansion of
42. Co-efficient of x5 in the expansion of (x2 – x – 2)5 is :
(a2x2 – 6ax + 11)10, where a is constant, is 1024, then the
(a) –83 (b) –82 value of a is :
(c) –81 (d) 0 (a) 5 (b) 1
(c) 2 (d) 3
BINOMIAL THEOREM 129

52. Sum of the last 30 coefficients in the expansion of (1 + x)59, 56. 10


C1 + 10C2 + 10C3 + .............+ 10C10 =
when expanded in ascending powers of x is (a) 512 (b) 511
(a) 259 (b) 258 (c) 1024 (d) none of these
(c) 230 (d) 229 57. The value of C0 – C1 + C2 – .... + (–1)n nCn is
n n n

53. If n  N and (1 – x + x2)n = a0 + a1x + a2x2 + ...... + a2nx2n, then (a) 1 (b) n
a0+ a2+ a4 + ... +a2n is equal to :
(c) 2n (d) 0
3n  1 3n 1 n
2 n 1
(a) (b) 58.  C r is equal to
2 2 r 1

1 1 (a) 22n–1 (b) 22n


n n
(c) 3  (d) 3  (c) 22n+1 –1 (d) 22n+1
2 2
54. If n  N then 49n + 16 n – 1 is divisible by 59. The sum rCr + r+1Cr + r+2Cr + ... + nCr (n > r) equals

(a) 3 (b) 19 (a) nCr+1 (b) n+1


Cr+1
n+1 n+1
(c) 64 (d) 29 (c) Cr–1 (d) Cr

55. Remainder when 7100 is divided by 25 is 60. The value of

(a) 1 (b) 24  7
C0  7 C1    7

C1  7 C2  ...   7

C6  7 C7 is
(c) 18 (d) none of these
(a) 27 – 1 (b) 28 – 2
(c) 28 – 1 (d) 28
130 BINOMIAL THEOREM

EXERCISE - 2 : PREVIOUS YEAR JEE MAINS QUESTIONS


1. The number of integral terms in the expansion of 6
50
256 7. The value of C4   56  r C3 is (2005)
 3 8 5  is (2003) r 1

(a) 32 (b) 33 (a) 56C4 (b) 56C3

(c) 34 (d) 35 (c) 55C3 (d) 55C4


8. For natural numbers m, n if (1–y)m (1 + y)n =1 + a1y + a2y2 +...
n n
1 r t and a1 = a2 = 10, then (m, n) is (2006)
2. If s n   n and t n   n then n is equal to
r 0 Cr r 0 Cr sn (a) (35, 20) (b) (45, 35)
(2004) (c) (35, 45) (d) (20, 45)
9. The sum of the series
n n
(a) (b)  1 20
C0 – 20C1 + 20C2 – 20C3 + ... + 20C10 is (2007)
2 2
1 20
2n  1 (a) – 20C10 (b) C10
(c) n – 1 (d) 2
2
(c) 0 (d) 20C10
3. The coefficient of xn in expansion of (1 + x) (1 –x)n is
10. In the binomial expansion of (a – b)n, n  5, the sum of 5th
(2004)
(a) (n – 1) (b) (–1)n (1 – n) a
and 6th terms is zero, then equals (2007)
b
(c) (–1)n–1 (n–1)2 (d) (–1)n–1 n
4. The coefficient of the middle term in the binomial expansion 5 6
in powers of x of (1 + x)4 and of (1 – x)6 is the same, if  (a) (b)
n4 n 5
equals (2004)
5 10 n 5 n4
(a)  (b) (c) (d)
3 3 6 5

3 3 n
n
(c)  (d) 11. Statement I :  (r  1) . Cr  (n  2) 2n 1
10 5 r 0

11
 1  n
5. If the coefficient of x7 in ax 2   equals the coefficient n
Cr .x r  (1  x) n  nx (1  x) n 1
 bx  Statement II :  (r  1)
r 0
11
 1  (2008)
of x–7 in ax  2  , then a and b satisfy the relation
 bx  (a) Statement I is false, Statement II is true
(2005) (b) Statement I is true, Statement II is true;
a Statement II is a correct explanation for Statement I
(a) ab = 1 (b) 1
b (c) Statement I is true, Statement II is true;
(c) a + b = 1 (d) a – b = 1 Statement II is not a correct explanation for Statement I
6. If the coefficients of rth, (r + 1) th and (r + 2)th terms in the (d) Statement I is true, Statement II is false
binomial expansion of (1 + y)m are in AP, then m and r 12. The remainder left out when 82n – (62)2n+1 is divided by 9 is
satisfy the equation (2005) (2009)
(a) m2 – m (4r – 1) + 4r2 + 2 = 0 (a) 0 (b) 2
(b) m2 – m (4r + 1) + 4r2 – 2 = 0 (c) 7 (d) 8
(c) m2 – m (4r + 1) + 4r2 + 2 = 0
(d) m2 – m (4r – 1) + 4r2 – 2 = 0
BINOMIAL THEOREM 131

10 10 19. The coefficient of x 50 in the binomial expansion of


13. Let S1   j ( j  1) 10 C j , S2   j 10 C j and (1 + x)1000 + x(1 + x)999 + x2(1 + x)998 + ..... + x1000 is:
j1 j1
(2014/Online Set–2)
10
S3   j2 10
Cj 1000 ! 1000 !
(a) 50 ! 950 ! (b) 49 ! 951 !
j1
     
Statement I : S3 = 55×29.
Statement II : S1 = 90 × 28 and S2 = 10×28. (2010) 1001! 1001!
(c) 51 ! 950 ! (d) 50 ! 951 !
(a) Statement I is false, Statement II is true      
(b) Statement I is true, Statement II is true;
55
Statement II is a correct explanation for Statement I  x
20. If  2   is expanded in the ascending powers of x and
 3
(c) Statement I is true, Statement II is true;
the coefficients of powers of x in two conscutive terms of
Statement II is not a correct explanation for Statement I
the expansion are equal, then these terms are:
(d) Statement I is true, Statement II is false
(2014/Online Set–3)
14. The coefficient of x7 in the expansion of (1 – x – x2 + x3)6 is th th
(a) 7 and 8 (b) 8 and 9th
th

(2011)
(c) 28th and 2th (d) 27th and 28th
(a) –132 (b) –144
21. The coefficient of x 1012 in the expansion of
(c) 132 (d) 144 (1 + xn + x253)10, (where n < 22 in any positive integer), is:
2n 2n (2014/Online Set–4)
15. If n is a positive integer, then   
3 1 
3 1 is
(a) 1 (b) C410

(2012) (c) 4n (d) 253C4


(a) an irrational number 22. The sum of coefficients of integral powers of x in the
(b) an odd positive integer 50

(c) an even positive integer


binomial expansion of 1  2 x   is: (2015)

(d) a rational number other than positive integers. 1 50 1 50


(a)
2

3 1  (b)
2

2 1 
16. The term independent of x in expansion of
10 1 50 1 50
 x 1
 2 / 3 1/ 3 
x 1 
 is (2013)
(c)
2

3 1  (d)
2
3  
 x  x  1 x  x1/ 2 
30
(a) 4 (b) 120
23. The value of   r  2 r – 3 is equal to :
r 16
(c) 210 (d) 310
17. If the coefficients of x 3 and x 4 in the expansion of (2015/Online Set–1)
(1 + ax + bx2) (1 – 2x)18 in powers of x are both zero, then (a) 7775 (b) 7785
(a, b) is equal to : (2014)
(c) 7770 (d) 7780
 272   251  24. The term independent of x in the binomial expansion of is
(a) 16, 3  (b) 16, 3 
    8
 1 5  2 1
 251   272  1   3x  2x   : (2015/Online Set–2)
(c) 14, 3  (d) 14, 3   x  x
   
(a) 496 (b) 496
18. The number of terms in the expansion of (1 + x)101 (1 + x2 -
x)100 in powers of x is (2014/Online Set–1) (c) 400 (d) 400

(a) 302 (b) 301


(c) 202 (d) 101
132 BINOMIAL THEOREM

25. If the number of terms is the expansion of


n
28. The value of  21
C1 10 C1    21
C 2  10 C 2  
 2 4 
1   2  , x  0, is 28, then the sum of the
 x x   21
C3 10 C3    21

C 4  10 C 4  .....   21
C10 10 C10 
coefficients of all the terms in this expansion, is : (2016)
is: (2017)
(a) 2187 (b) 243
(a) 221 – 211 (b) 221 – 210
(c) 729 (d) 64
(c) 220 – 29 (d) 220 – 210
26. For x  R, x  –1, if –5
29. The coefficient of x in the binomial expansion of
(1 + x)2016 + x(1 + x)2015 + x2(1 + x)2014 + …… + x2016 =
10
2016  
a i x i , then a is equal to :  x  1  x 1  ,
 17
(2016/Online Set–1)  2 1 1
 where x  0, 1, is :
i 0
 x 3  x3 1 x  x2 
2017! 2016! (2017/Online Set–2)
(a) 17! 2000! (b) 17!1999!
(a) 1 (b) 4
(c) –4 (d) –1
2017! 2016!
(c) (d) 30. The sum of the co-efficients of all odd degree terms in the
2000! 16!
expansion of
27. If the coefficients of x-2 and x-4 in the expansion of
5 5

 1 
18
x  x3  1   x  
x 3  1 ,  x  1 is : (2018)
 x3  1  ,
 1  (x > 0), are m and n respectively, then (a) 2 (b) -1
 2x 3 
(c) 0 (d) 1
m 31. If n is the degree of the polynomial
is equal to : (2016/Online Set–2)
n 8 8
 2   2 
4   +  and m is the
3 3 3 3
(a) 182 (b)  5x + 1 - 5x - 1   5x + 1 + 5x - 1 
5
coefficient of xn in it, then the ordered pair (n, m) is equal
5 to : (2018/Online Set–1)
(c) (d) 27
4 (a) (24, (10)8) (b) (8, 5(10)4)
(c) (12, (20)4) (d) (12, 8(10)4)
10
32. The coefficient of x in the expansion of
(1 + x)2 (1 + x2)3 (1 + x3)4 is equal to :(2018/Online Set–2)
(a) 52 (b) 56
(c) 50 (d) 44
33. The coefficient of x in the expansion of the product (2–x2)
2

. ((1+2x+3x2)6 + (1–4x2)6) is : (2018/Online Set–3)


(a) 107 (b) 106
(c) 108 (d) 155
133 BINOMIAL THEOREM

EXERCISE - 3 : ADVANCED OBJECTIVE QUESTIONS

1. If 0  r,  n, then the coefficient of xr in the expansion of 8. The greatest term (numerically) in the expansion of
P = 1 + (1 + x) + (1 + x)2 + ..... + (1 + x)n is 3
(2 + 3x)9, when x  , is
(a) nCr (b) n+1Cr+1 2
(c) nCr+1 (d) none of these
2. If rth and (r + 1)th term in the expansion of (1 + x)n are 5  311 5  313
(a) (b)
equal, then n = 2 2

1  x  r  x 1  x  r  x 7  313
(a) (b) (c) (d) none of these
4x 3x 2

1  x  r  x 1  x  r  x 9. The greatest term (numerically) in the expansion of


(c) (d)
x r 1
(3 – 5x)11, when x  is
3. The greatest coefficent in the expansion of (1 + x) is 2n 5

(a) 2nCn (b) 2nCn–1 (a) 55 × 39 (b) 46 × 39


(c) 2nCn–2 (d) none of these (c) 55 × 36 (d) none of these
4. The coefficient of x 20 in the expansion of 10. The sum of the coefficients in the expansion of
5 (6a – 5b)n, where n is a positive integer, is
40  1 
1  x 2
. x 2  2  2 
 x 
is (a) 1 (b) –1
(c) 2n (d) 2n–1
(a) 30C10 (b) 30C25
n
(c) 1 (d) none of these  1
11. If the fourth term in the expansion of  px   is
5. The ratio of the coefficient of x10 in (1-x2)10 and the term  x

2
10 independent of x, then the value of term is

independent of x in  x   , is
 x (a) 5p3 (b) 10p3
(c) 20p3 (d) none of these
(a) 1 : 16 (b) 1 : 32
12. Which of the following expression is divisible by 1225 ?
(c) 1 : 64 (d) none of these
6. The number of distinct terms in the expansion of (a) 62n – 35n–1 (b) 62n – 35n+1
(x + y – z)16 is (c) 62n – 35n (d) 62n – 35n+2

(a) 136 (b) 153 13. If 7103 is divided by 25, then the remainder is
(c) 16 (d) 17 (a) 20 (b) 16
7. The total number of terms in the expansion of (c) 18 (d) 15
(a + b + c + d)n, n  N is 14. The last digit of the number (32)32 is
(a) 4 (b) 6
n  n  1 n  2  n  n  1 n  2  n  3
(a) (b) (c) 8 (d) none of these
6 6
15. 97 + 79 is divisible by
 n  1 n  2  n  3 (a) 6 (b) 24
(c) (d) none of these
6 (c) 64 (d) 72
BINOMIAL
134 THEOREM 134
BINOMIAL THEOREM

16. The sum 20C0 + 20C1 + 20C2 + ...... 20C10 is equal to 24. The greatest value of the term independent of x in the
expansion of (x sin  + x–1 cos )10, R, is
20! 19 1 20!
(a) 220  (b) 2  . 10!
10!
2
2 10!2
(a) 25 (b)
(5!)2
(c) 219 + 20C10 (d) none of these
10!
17. The sum 1. C1 –2. C2 + 3. 20C3 – .....–20C20 is equal to
20 20
(c) (d) none of these
2  (5!)2
5
19
(a) 2 (b) 0
(c) 2 – 1 20
(d) none of these 25. The coefficient of the middle term in the binomial
n n n n expansion, in powers of x, of (1 + x)4 and of (1–x)6 is
18. 1. C1 + 2, C2 + 3. C3 + ....+ n. Cn is equal to
same, if equals
n  n  1
(a) .2 n (b) 2n+1–3 3 10
4 (a) (b)
5 3
(c) n, 2n–1 (d) none of these
19. If (1 +x) 15 = C 0 + C 1 x C 2 x 2 +....+ C 15 x 15 , then 3 5
(c)  (d) 
10 3
15
C02 15 C12 15 C22 15 C32  ....15 C15
2
is equal to
26. The sum of the numerical coefficients in the expansion of
(a) 0 (b) 1
12
(c) –1 (d) none of these  x 2y 
1    , is
 3 3 
20. If A = 2nC0. 2nC1 + 2nC1 2n–1C1 + 2nC2 2n–2C1 + .....then A is
(a) 0 (b) 2n (a) 1 (b) 2
(c) n22n (d) 1 (c) 212 (d) none of these
21. In the binomial expansion of (a – b)n, n  5, the sum of the 27. The number of terms in the expansion of (2x + 3y – 4z)n, is
5th and 6th terms is zero. Then, a/b equals (a) n + 1 (b) n + 3

n 5 n4 (n  1) (n  2)
(a) (b) (c) (d) none of these
6 5 2

28. The coefficient of x 4 in the expansion of


5 6 (1 + x + x2 + x3)11, is
(c) (d)
n4 n 5
(a) 900 (b) 909
22. The number of irrational terms in the expansion of (c) 990 (d) 999
100 29. The coefficient of x in the expansion of (1 + x2)5 (1 + x)4, is
5

 8
56 2  , is
(a) 30 (b) 60
(a) 97 (b) 98 (c) 40 (d) none of these
(c) 96 (d) 99 30. If the ratio of 7th term from the beginning to the seventh

23. The sum of the rational terms in the expansion of  1 


x
1
term from the end in the expansion of  3 2   is 6
3
10  3
 253  is
then x, is
(a) 32 (b) 9 (a) 9 (b) 6, 15
(c) 41 (d) none of these (c) 12, 9 (d) none of these
BINOMIAL THEOREM 135

31. The term independent of x in the expansion of (1 + x)n 36. Numerically greatest term, in the expansion of (8 – 5x)18,
(where x = 2/5) is :
n
 1  is : (a) 1623 × 224 (b) 1623 × 222
1  
 x
(c) 1623 × 223 (d) none of these

37. The coefficient of (a3 b 6 c8 d 9 e f) in the expansion of


(a) C02  2C12  3C22  ...  (n  1)Cn2
(a + b + c – d – e – f)31 is :
(b) (C0 + C1 + C2 + ... + Cn)2 (a) 123210 (b) 23110

(c) 3110 (d) none of these


(c) C02  C12  ...  Cn2
38. The number 525 – 325 is divisible by :
(d) none of these
(a) 2 (b) 3
32. The term independent of x in the expansion of
(c) 5 (d)7

9 39. The sum of coefficients of the two middle terms in the


3 1 
(1 + x + 2x3)  x 2   is : expansions of (1 + x)2n – 1 is euqal to :
2 3x 
(a) (2n – 1)Cn (b) (2n – 1)Cn + 1
(a) 7/18 (b) 2/27 (c) 2nCn – 1 (d) 2nCn

32
7 2 7 2 40. If 7 divides 3232 , the remainder is :
(c)  (d) 
18 27 18 27
(a) 1 (b) 0
33. If (1 – x + x2)n = a0 + a1x + a2x2 + ... + a2nx2n, then (c) 4 (d) 6
a0 + a2 + a4 + ... + a2n equals :
n
 1 
41. The number of terms in the expansion of  x 2  1  2  ,
3n  1  x 
3n  1
(a) (b)
2 2
n  N is.

(a) 2n (b) 3n
1  3n n1
(c) (d) 3  (c) 2n + 1 (d) 3n + 1
2 2

n
1
42. The middle term in expansion of  x 2  2  2  is :
34. The integral part of ( 2  1)6 is:  x 

(a) 198 (b) 197


n! 2n!
(c) 196 (d)163 (a) (b)
[(n / 2)!]2 [(n / 2)!]2
35. 49n + 16 n – 1 is divisible by :

(a) 3 (b) 19 1.3.5........(2n  1) n (2n)!


(c) 2 (d)
(c) 64 (d)29 n! (n!) 2
136 BINOMIAL THEOREM

10 47. Integral part of (7  4 3) n is (n  N)


 x  3 
43. The term independent of x in      2   is:
  3   2x   (a) an even number
(b) an odd number
10
(a) 1 (b) C1 (c) an even or an odd number depending upon the of n
(c) 5/12 (d) none of these
(d) none of these
50 1000
44. The coefficient of x in the expansion : (1 + x) + 2x
48. The sum 20C0 + 20C1 + 20C2 + ...+ 20C10 is
999 2 998
(1 + x) + 3x (1 + x) +...+1001 terms

(a) 1002C50 (b) 1002C51 20! 1 20!


(a) 220  (b) 219  .
(c) 1005
C50 (d) 1005
C48 (10!)2 2 (10!) 2

45. The greater term (numerically) in the expansion of (c) 219 + 20C10 (d) none of these

3 49. If n is a positive integer greater than I, then


(2 + 3x)9, when x  ;
2 a – nC1(a – 1) + nC2(a – 2) – ....+ (–1)n (a – n) is equal to

(a) n (b) a
11 13
53 53 (c) 0 (d) none of these
(a) (b)
2 2

1
50. If =1 + a1x + a2x2 + ..., then the value of ar is
73 13 1  2x  x 2
(c) (d) none of these
2
(a) 2r (b) r + 1
11
46. The greatest term (numarically) in expansion of (3 – 5x) , (c) r (d) r – 1
where x = 1/5 is :

(a) 55 × 39 (b) 46 × 39
(c) 55 × 36 (d) none of these
137 BINOMIAL THEOREM

EXERCISE - 4 : PREVIOUS YEAR JEE ADVANCED QUESTIONS


Objective Question I [Only one correct option]
n  n   n 
1. Given positive integers r > 1, n > 2 and the coefficient of 7. For 2  r  n,    2    is equal to : (2000)
 r   r 1   r  2 
(3r)th and (r + 2)th terms in the binomial expansion of
(1 + x)2n are equal . Then : (1980)
 n  1  n 1
(a) n = 2r (b) n = 2r + 1 (a)   (b) 2  
 r 1   r 1 
(c) n = 3r (d) none of these
10  n  2 n 2
x 3 
4 (c) 2   (d)  
2. The coefficient of x in   2  is : (1983)  r 
2 x   r 
8. In the binomial expansion of (a – b)n, n  5 the sum of the
405 504 5th and 6th terms is zero. Then a/b equals : (2001)
(a) (b)
256 259
n 5 n4
450 (a) (b)
(c) (d) none of these 6 5
263
5 6
3. If Cr stands for nCr, then the sum of the series (c) (d)
n4 n 5
n n 9. Let Tn denote the number of triangles which can be formed
2   !  !
 2   2   C 2  2C2  3C 2  ....  (1) n (n  1) C 2  using the vertices of a regular polygon of n sides. If
n!  0 1 2 n
Tn + 1 – Tn = 21, then n equals : (2001)
where n is an even positive integer, is equal to : (1986) (a) 5 (b) 7
n/2
(a) (–1) (n + 2) n
(b) (–1) (n + 1) (c) 6 (d) 4
n/2
(c) (–1) (n + 1) (d) none of these m 10
  20  p
3 1/2 5 3 1/2 5 10. The sum     , where    0 if p > q, is maximum
4. The expression [x + (x – 1) ] + [x – (x – 1) ] is a i  0  i  m  i  q
polynomial of degree : (1992)
when m is : (2002)
(a) 5 (b) 6
(a) 5 (b) 10
(c) 7 (d) 8
(c) 15 (d) 20
n n
1 r 11. Coefficient of t24 in (1 + t2)12 (1 + t12) (1 + t24) is : (2003)
5. If a n   n , then  n equals : (1998)
r 0 Cr r 0 Cr (a) 12C6 + 3 (b) 12C6 + 1
(a) (n – 1) an (b) n an (c) 12C6 (d) 12C6 + 2
12. If n – 1Cr = (k2 – 3) nCr + 1, then k belong to : (2004)
1
(c) na n (d) none of these
2 (a) (,  2] (b) [2, )
6. If in the expansion of (1 + x)m (1 – x)n, the coefficients of x
and x2 are 3 and –6 respectively, then m is : (1999) (c)   3, 3  (d) ( 3, 2]

(a) 6 (b) 9
 30  30   30  30   30  30 
(c) 12 (d) 24 13.         ....     is equal to
 0  10   1  11   20  30 
(2005)
30 60
(a) C11 (b) C10
30
(c) C10 (d) 65C55
BINOMIAL
138 THEOREM 138
BINOMIAL THEOREM

14. For r = 0, 1, ...., 10, let Ar, Br and Cr denote, respectively, the 25. Find the sum of the series :
coefficient of xr in the expansions of (1 + x)10, (1+x)20 and
n
r n  1 3r 7r 15r 
30
(1 + x) . Then
10

A (B10 Br  C10 A r ) is equal to (2010)  (1) Cr  r  2r  3r  4r ....upto mterms 


2 2 2 2
(1985)
r r 0  
r 1

26. Prove that C0 – 22 . C1 + 32 . C2 – ..... + (–1)n (n + 1)2 . Cn = 0,


(a) B10 – C10
2

(b) A10 B10  C10 A10  n > 2 where Cr = nCr. (1989)
(c) 0 (d) C10 – B10 2n 2n
r r
15. Coefficient of x in 11
the expansion of
27. If  a (x  2)   b (x  3)
r 0
r
r 0
r and ak = 1 for all k > n, then
(1 + x2)4 (1 + x3)7 (1 + x4)12 is (2014)
show that bn = 2n + 1Cn + 1. (1992)
(a) 1051 (b) 1106
k
r 1 3n
(c) 1113 (d) 1120 28. Prove that  (3)
r 1
C2r 1  0 , where k = (3n)/2 and n is
Fill in the Blanks
an even positive integer. (1993)
16. The sum of the coefficients of the polynomial
29. Let n be a positive integer and
(1 + x – 3x2)2163 is ...... (1982)
(1 + x + x2)n = a0 + a1x + ..... + a2nx2n.
17. The coefficient of x99 in the polynomial
2 2 2
(x – 1) (x – 2) .... (x – 100) is ..... (1982) Show that a 0  a1  .....  a 2n  a n . (1994)
n 2
18. If (1 + ax) = 1 + 8x + 24x + ...., then a = ..... and n = .....
(1983) 3! n  nC 
30. Prove that   (1) r  r  3 r  (1997)
2(n  3) r  0  Cr 
19. Let n be a positive integer. If the coefficients of 2nd, 3rd,
and 4th terms in the expansion of (1 + x)n are in A.P., then 31. For any positive integers m, n (with n > m),
the value of n is ..... (1994)
n
20. Let X = (10C1)2 + 2(10C2)2 + 3(10C3)2 +..... + 10(10C10)2, where let    n C m . Prove that
10
Cr , r  {1, 2, ..., 10} denote binomial coefficients. Then, m
1  n   n 1   n  2   m   n 1 
the value of X is ______ (2018)
1430      ...      
m  m   m   m   m  1
Analytical and Descriptive Questions. Hence, or otherwise, prove that
2n 2 2n 2 2n 2 2n 2
21. Prove that : ( C0) – ( C1) + ( C2) + ... + ( C2n)  n   n 1  n  2   m   n 2 
= (–1)n.2n Cn. (1978)    2   3   ....   n  m  1     
m  m   m   m  m 2
22. Prove that : (2000)
n 32. Prove that
C12  2.C22  3.C32  ......  2n.C 22 n   1 n.C n (1979)
 n  n   n  n 1  k 2  n  n  2 
23. If (1 + x)n = C0 + C1x + C2x2 + .... + Cnxn, then show that the 2 k     2k 1     2     .....
sum of products of the Ci’s taken two at a time represented  0  k   1  k 1   2  k  2 
by CiCj is equal to 0 < i < j < n
 n  n  k   n 
 (1) k     (2003)
2 2n 1 
(2n!)  k  0   k 
(1983)
2(n!) 2
33. The coefficient of three consecutive terms (1 + x)n+5 are in
2 n
24. Given sn = 1 + q + q + ..... + q the ratio 5 : 10 : 14. Then, n is equal to (2013)
2 n 34. The coefficient of x 9 in the expansion of
q 1  q 1   q 1  (1 + x) (1 + x2) (1 + x3) … (1 + x100) is (2015)
Sn  1     ....    ,q  1 .
2  2   2 
35. Let m be the smallest positive integer such that the
Prove that n+1C1 + n+1C2s1 + n+1C3s2 + coefficient of x 2 in the expansion of
(1 + x)2 + (1 + x)3 +…..+ (1 + x)49 + (1 + mx)50 is (3n + 1) 51C3
.... + n+1Cn+1sn = 2nSn (1984)
for some positive integer n. Then the value of n is(2016)
139 BINOMIAL THEOREM

ANSWER KEY
EXERCISE - 1 : BASIC OBJECTIVE QUESTIONS

1. (d) 2. (c) 3. (a) 4. (b) 5. (a) 6. (c) 7. (c) 8. (c) 9. (d) 10. (c)
11. (c) 12. (b) 13. (c) 14. (b) 15. (a) 16. (c) 17. (b) 18. (d) 19. (b) 20. (d)
21. (a) 22. (a) 23. (c) 24. (c) 25. (d) 26. (c) 27. (c) 28. (d) 29. (a) 30. (a)
31. (c) 32. (b) 33. (a) 34. (c) 35. (d) 36. (d) 37. (d) 38. (c) 39. (b) 40. (d)
41. (d) 42. (c) 43. (c) 44. (c) 45. (d) 46. (b) 47. (b) 48. (c) 49. (b) 50. (b)
51. (d) 52. (b) 53. (a) 54. (c) 55. (a) 56. (d) 57. (d) 58. (a) 59. (b) 60. (b)

EXERCISE - 2 : PREVIOUS YEAR JEE MAINS QUESTIONS

1. (b) 2. (a) 3. (b) 4. (c) 5. (a) 6. (b) 7. (a) 8. (c) 9. (b) 10. (d)
11. (b) 12. (b) 13. (d) 14. (b) 15. (a) 16. (c) 17. (a) 18. (c) 19. (d) 20. (b)
21. (b) 22. (c) 23. (d) 24. (c) 25. (c) 26. (a) 27. (a) 28. (d) 29. (a) 30. (a)
31. (c) 32. (a) 33. (b)

EXERCISE - 3 : ADVANCED OBJECTIVE QUESTIONS

1. (b) 2. (c) 3. (a) 4. (b) 5. (b) 6. (b) 7. (c) 8. (c) 9. (a) 10. (a)
11. (c) 12. (a) 13. (c) 14. (b) 15. (c) 16. (d) 17. (b) 18. (c) 19. (a) 20. (c)
21. (b) 22. (a) 23. (c) 24. (c) 25. (c) 26. (c) 27. (c) 28. (c) 29. (b) 30. (a)
31. (c) 32. (d) 33. (a) 34. (b) 35. (c) 36. (d) 37. (d) 38. (a) 39. (d) 40. (c)
41. (c) 42. (d) 43. (d) 44. (a) 45. (c) 46. (a) 47. (b) 48. (d) 49. (c) 50. (b)

EXERCISE - 4 : PREVIOUS YEAR JEE ADVANCED QUESTIONS

1. (a) 2. (a) 3. (a) 4. (c) 5. (c) 6. (c) 7. (d) 8. (b) 9. (b) 10. (c)
11. (d) 12. (d) 13. (c) 14. (d) 15. (c) 16. –1 17. –5050 18. a = 2, n = 4 19. n = 7

2 mn  1
20. (646) 25. 33. 6 34. (8) 35. (5)
2mn (2 n  1)

Dream on !!

05
PERMUTATION AND COMBINATION
PERMUTATION AND COMBINATION 141

PERMUTATION AND COMBINATION

1. FUNDAMENTAL PRINCIPLES OF COUNTING


  
1.1 Fundamental Principle of Multiplication

1. Let  r  and  n  be  positive  integers  such  that  l    r    n.


If an event can occur in m different ways following which
Then, the number of all permutations of n distinct items
another event can occur in n different ways following which
or objects taken r at a time, is
another event can occur in p different ways. Then the total
n
number  of  ways  of  simultaneous  happening  of  all  these Pr = nCr × r !
events in a definite order is m × n × p.
Proof : Total ways = n(n – 1)(n – 2) . . .  (n  r  1)

1.2 Fundamental Principle of Addition

n(n  1)(n  2)...(n  r  1)(n  r)!



If  there  are  two  jobs  such  that  they  can  be  performed (n  r)!
independently in m and n ways respectively, then either of

the two jobs can be performed in (m + n) ways.
n!

(n  r )!
2. SOME BASIC ARRANGEMENTS AND SELECTIONS

= nPr.
2.1 Combinations
So, the total no. of arrangements (permutations) of n-
Each of the different selections made by taking some or all
distinct items, taking r at a time is  nPr or P(n, r).
of  a  number  of  distinct  objects  or  items,  irrespective  of
2. The  number  of  all  permutations  (arrangements)  of  n
their  arrangements  or  order  in  which  they  are  placed,  is
distinct objects taken all at a time is n!.
called a combination.
3. The number of ways of selecting r items or objects from
2.2 Permutations a group of n distinct items or objects, is

Each of the different arrangements which can be made by
n!
taking some or all of a number of distinct objects is called n Cr .
(n  r )!r!
a  permutation.
142 PERMUTATION AND COMBINATION

3. GEOMETRIC APPLICATIONS OF nCr 5. DIVISION OF OBJECTS INTO GROUPS

(i) Out  of  n  non-concurrent  and  non-parallel  straight 5.1 Division of items into groups of unequal sizes
n
lines, points of intersection are  C2.
1. The number of ways in which (m + n) distinct items
(ii)   Out  of  ‘n’  points  the  number  of  straight  lines  are can  be  divided  into  two  unequal  groups  containing
(when no three are collinear)  nC2 .

(iii) If out of n points m are collinear, then No. of straight  m  n  !.
m and n items, is 
m!n!
lines =  nC2 –  mC2 + 1

(iv)  In a polygon total number of diagonals out of n points
2. The number of ways in which (m+ n+ p) items can be
n (n  3) divided into unequal groups containing m, n, p items, is
(no three are collinear) =  nC2 – n   .
2
 m  n  p  !.
(v) Number of triangles formed from n points is  nC3 . m + n + p
Cm .  n + pCm =  m!n ! p!
(when no three points are collinear)

(vi) Number of triangles  out of n points in  which m are 3. The number of ways to distribute (m + n+ p) items among


n m
collinear, is  C3 –  C3 . 3 persons in the groups containing m, n and p items
(vii) Number of triangles that can be formed out of n points = (No. of ways to divide) × (No. of groups)!
(when  none  of  the  side  is  common  to  the  sides  of
polygon), is nC3 –  nC1 –  nC1 .  n – 4C1  m  n  p !  3!.

(viii)Number of parallelograms in two systems of parallel m!n!p!

lines  (when  1st  set  contains  m  parallel  lines  and  2nd


set contains n parallel lines), is =  nC2 ×  mC2
5.2 Division of Objects into groups of equal size
(ix) Number  of  squares  in  two  system  of  perpendicular
The number of ways in which mn different objects can be
parallel lines (when 1st set contains m equally spaced
parallel  lines  and  2nd  set  contains  n  same  spaced divided equally into m groups, each containing n objects
parallel  lines) and the order of the groups is not important, is

m 1
 mn  !  1
  (m  r )(n  r); (m  n )
r 1
 
 n!m  m !
 

4. PERMUTATIONS UNDER CERTAIN CONDITIONS The number of  ways in which mn different  items can be


The  number  of  all  permutations  (arrangements)  of  n divided equally into m groups, each containing n objects
different objects taken r at a time : and the order of groups is important, is
(i) When  a  particular  object  is  to  be  always  included  in
each arrangement, is  n–1Cr–1 × r ! .   mn  ! 1 
   m! 
 mn  !
(ii) When  a  particular  object  is  never  taken  in  each   n!m m!   n!m
 
arrangement, is  n – 1Cr × r!.
PERMUTATION AND COMBINATION 143

6. PERMUTATIONS OF ALIKE OBJECTS   7. DISTRIBUTION OF ALIKE OBJECTS


(i) The total number of ways of dividing n identical items
1. The number of  mutually distinguishable  permutations among r persons, each one of whom, can receive 0, 1,
of n things, taken all at a time, of which p are alike of 2, or more items ( n), is n + r – 1Cr – 1.
one kind, q alike of second kind such that p + q = n, is OR
The total number of ways of dividing n identical objects
n! into r groups, if blank groups are allowed, is n + r – 1Cr – 1.
p!q! (ii) The total number of ways of dividing n identical items
among r persons, each of whom, receives at least one
item is  n – 1Cr – 1.
2. The  number  of  permutations  of  n  things,  of  which  p
OR
are  alike  of  one  kind,  q  are  alike  of  second  kind  and
The number of  ways in which n  identical  items can
be  divided into  r  groups  such that  blank groups  are
n!
remaining all are distinct, is  . Here p + q  n not allowed, is n – 1Cr – 1.
p!q!
(iii) The number of  ways in which n  identical  items can
be  divided  into  r  groups  so  that  no  group  contains
3. The  number  of  permutations  of  n  things,  of  which  p1 less than k items and more than m (m < k) is
are alike of one kind; p2 are alike of second kind; p3 are The coefficient of xn   in the expansion of
alike of third kind; ..... ; pr are alike of rth kind such that (xm + xm + 1 + . . . . xk)r

n! 8. NO. OF INTEGRAL SOLUTIONS OF LINEAR


p1 + p2 + . . . + pr = n, is  .
p1 !p2 !p3 !....pr ! EQUATIONS AND INEQUATIONS

Consider the eqn. x1 + x2 + x3 + x4 + . . . .+ xr = n  ...(i)
4. Suppose  there  are  r  things  to  be  arranged,  allowing
where x1, x2,. . . . ., xr and n are non–negative integers.
repetitions. Let further pl, p2, ...., pr be the integers such This equation may be interpreted as that n identical objects
that the first object occurs exactly p1 times, the second are to be divided into r groups.

occurs  exactly  p 2  times  subject,  etc.  Then  the  total 1. The total no. of non-negative integral solutions of the


equation x1 + x2 + ....+ xr = n is  n  +  r –1Cr –1.
number of permutations of these r objects to the above
2. The total number of solutions of the same equation in
condition,  is the set N of natural numbers is  n – 1Cr– 1.
3. In order to solve inequations of the form
(p1  p 2  ...  p r )! x1 + x2 + . . . . .+ xm  n
.
p!p 2 !p 3 !....p r !
we   introduce   a   dummy   (artificial)   variable   xm  +  1
such   that x1 +x2 + . . . . + xm + xm + 1 = n, where xm + 1  0.
The no. of solutions of this equation are same as the no. of
solutions of in Eq. (i).
144 PERMUTATION AND COMBINATION

9. CIRCULAR PERMUTATIONS 11. THE NUMBER OF DIVISORS AND THE SUM


OF THE DIVISORS OF A GIVEN NATURAL NUMBER
1. The number of circular permutations of n distinct objects
is (n – l)!.
n n n n
2. If  anti-clockwise  and  clockwise  order  of  arrangements Let N  p11 .p22 .p33 .....pk k ...(1)
are not distinct then the number of circular permutations
of n distinct items is 1/2 {(n – 1)!} where  pl,  p2,  .  .  .  .  ,  pk  are  distinct  prime  numbers  and
n1,  n2,....,nk are  positive  integers.
e.g., arrangements of beads in a necklace, arrangements
of flowers in a garland etc. 1. Total number of divisors of N = (n1 + 1)(n2 + 1) .  . .(nk + 1).

2. This  includes  1  and  n  as  divisors.  Therefore,  number  of


10. SELECTION OF ONE OR MORE OBJECTS divisors other than 1 and n, is

(n1 + 1)(n2 + 1)(n3 + 1). . . . .(nk + 1) – 2.
1. The number of ways of selecting one or more items from a
3. The sum of all divisors of (1) is given by
group of n distinct items is 2n – 1.

Proof :  Out  of n  items,  1  item  can  be  selected  in  nCl
 p n 1 1  1  p n 2 1  1  p n 3 1  1  p n k 1  1 
3
n
ways; 2 items can be selected in  C2 ways; 3 items can  1 
2
 ....
k
.
 1p  1  2p  1  3p  1   p k  1 
be selected in nC3 ways and so on......

Hence, the required number of ways
12. DEARRANGEMENTS
= nC1 +  nC2 +  nC3 + . . . . +  nCn

= (nC0 +  nC1 +  nC2 + . . . . +  nCn) –  nC0 If n distinct objects are arranged in a row, then the no. of


ways in which they can be dearranged so that none of them
= 2n – 1.
occupies its original place, is
2. The  number  of  ways  of  selecting  r  items  out  of  n
identical items is 1 .  1 1 1 1 1
n!1      ....   1n 
 1! 2! 3! 4! n!
3. The  total  number  of  ways  of  selecting  zero  or  more
items from a group of n identical items is (n + 1). and it is denoted by D (n).
4. The  total  number  of  selections  of  some  or  all  out  of If r (0  r  n) objects occupy the places assigned to them
p + q + r items where p are alike of one kind, q are alike of i.e., their original places and none of the remaining (n - r)
second kind and rest are alike of third kind, is objects  occupies  its  original  places,  then  the  no.  of  such
ways, is
[(p + 1)(q + 1)(r+ 1)]– 1.
D(n – r) =  nCr . D(n – r)
5. The total number of ways of selecting one or more items
from p identical items of one kind; q identical items of  1 1 1 1 
nr
second kind; r identical items of third kind and n different = nCr . (n – r)  ! 1  1!  2!  3!  ....   1 (n  r )!.
 
items, is (p + 1) (q + 1)(r + 1) 2n – 1
PERMUTATION AND COMBINATION 145

SOLVED EXAMPLES

Example – 1 Example – 3

The flag of a newly formed forum is in the form of three Find the number of different signals that can be generated
blocks, each  to  be  coloured  differently.  If there  are  six by arranging at least 2 flags in order (one below the other)
different colours on the whole to choose from, how many on a vertical staff, if five different flags are available.
such designs are possible ?
Sol. Since a signal may consist of either 2 flags, 3 flags, 4 flags
Sol. Since there are six colours to choose from, therefore, first or 5 flags. Therefore,
block can be coloured in 6 ways. After choosing first block
Total number of signals = Number of 2 flags signals 
second  and  third  can  be  choosen  in  5  and  4  ways
respectively.           + Number of 3 flags signals 
Hence, by the fundamental principle of multiplication, the
number of flag-designs is 6×5×4 = 120.                             + Number of 4 flags signals 

Example – 2
                            + Number of 5 flags signals 
Find  the  number  of  4  letter  words,  with  or  without = 5 × 4 + 5 × 4 × 3 + 5 × 4 × 3 × 2 + 5 × 4 × 3 × 2 × 1
meaning,  which can be  formed out of the letters of the
= 20 + 60 + 120 + 120 = 320
word ROSE, when
Example – 4
(i) the repetition of the letters is not allowed.
(ii) the repetition of the letters is allowed. Find the total number of ways of answering 5 objective
type questions, each question having 4 choices.
Sol. (i)  The  total  number  of  words  is  same  as  the  number  of
Sol. Since  each  question  can  be  answered  in  4  ways.  So,  the
ways of filling in 4 vacant places   by the 4 letters.
total number of ways  of  answering  5  questions  is
The first place can be filled in 4 different ways by any one 4×4×4×4×4 = 45.
of the 4 letters R, O, S, E. Since the repetition of letters is Example – 5
not allowed. Therefore, the second, third and fourth place
How many numbers are there between 100 and 1000 such
can be filled in by any one of the remaining 3, 2, 1 different
that at least one of their digits is 7 ?
ways respectively.
Sol. Clearly, a number between 100 and 1000 has 3-digits

Thus, by the fundamental principle of counting the required  Total no. of 3-digit nos having atleast one of their digits as 7
= (3digit nos) - (3-digit no. in which 7 does not appear)
number of ways is 4×3×2×1 = 24.
Total number of 3-digit number = 9×10×10 = 900.
Hence, required number of words = 24.
Total no. of 3-digit no. in which 7 does not appear at all :
(ii) If the repetition of the letters is allowed, then each of the 4 We have to form 3-digit nos by using the digits 0 to 9, except 7.

vacant places can be filled in succession in 4 different ways. So, hundred’s place can be filled in 8 ways and each of the
ten’s and one’s place can be filled in 9 ways.
Hence, required number of words = 4×4×4×4 = 256. So, required ways = 8×9×9 = 648
Hence,  total  number  of  3-digit  numbers  having  at  least
one of their digits as 7 is 900 – 648 = 252.
146 PERMUTATION AND COMBINATION

Example – 6 Example – 10

How  many  three-digit  numbers  more  than  600  can  be Find the number of triangles obtained by joining 10 points


formed by using the digits 2, 3, 4, 6, 7. on a plane if ?
(i) no three of which are collinear
Sol. Clearly, repetition of digits is allowed. Since a three-digit
number  greater  than  600  will  have  6  or  7  at  hundred’s (ii) four points are collinear
place. So, hundred’s place can be filled in 2 ways. Each of Sol. (i)  Since  no  three  point  are  collinear,  any  three  non-
the ten’s and one’s place can be filled in 5 ways. collinear points can be selected to form a triangle.
Hence, total number of required numbers = 2×5×5=50. Number of triangles required =  10C3 = 120

Example – 7 (ii) If four points are collinear
 Required no. of triangles = 10C3 –  4C3 = 120 – 4 = 116
How  many  numbers  divisible  by  5  and  lying  between
(because selection of 3 collinear point does not make a triangle.)
4000 and 5000 can be formed from the digits 4, 5, 6, 7
and 8. Example – 11

Sol. Clearly, a number between 4000 and 5000 must have 4 at Among 22 cricket players, there are 3 wicketkeepers and


thousand’s place. Since the number is divisible by 5 it must 6 bowlers. In how many ways can a team of 11 players be
have 5 at unit’s place. Now, each of the remaining places chosen  so  as  to  include  exactly  one  wicket  keeper  and
(viz. hundred’s and ten’s) can be filled in 5 ways. atleast 4 bowlers ?
Hence, total number of required numbers = 1×5×5×1=25. Sol. We  have  to  choose  11  players  which  include  exactly  1
Example – 8 wicket keeper and atleast 4 bowlers.
Combinations include 1 wicket keeper – 4 bowlers,
Find the number of ways of selecting 9 balls from 6 red
1 wicket keeper – 5 bowlers and 1 wicket keeper – 6 bowlers
balls,  5  white  balls  and  7  blue  balls  if  each  selection
Total number of combinations.
consists of 3 balls of each colour.
= 3C1 × 6C4 ×  13C6 + 3C1 ×  6C5 × 13C5 +  3C1 × 6C6 ×  13C4
Sol. The number of ways of selecting 9 balls from 6 red balls, 5
= 77220 + 23166 + 2145 = 102531
white balls and 7 blue balls containing 3 balls of each colour
= 6C3 ×  5C3 ×  7C3 Example – 12

6! 5! 7! In how many ways can 5 students be selected out of 11
  
3! (6  3) ! 3!(5  3) ! 3!(7  3) !  = 7000 students if
(i) 2 particular students are included ?
Example – 9 (ii) 2 particular students are not included ?

In how many ways can a team of 3 boys and 2 girls be Sol. There are 11 students, we have to select 5 students.

selected from 6 boys and 5 girls ? (i) 2 particular student are included then reqd no. of ways

Sol. Required number of ways. 9! 9  8  7  6!
 112 C52  9 C3   = 84
3!6! 3  2  6!
= 6C3 ×  5C2
(ii) 2 particular student are not included then reqd no. of ways
6! 5!
 
3!(6  3) ! 2! (5  2) !  = 20 × 10 = 200  11C3 
11! 1110  9  8!
 = 11 × 15 = 165
3!8! 3  2  8!
PERMUTATION AND COMBINATION 147

Example – 13 Example – 17

How many different signals can be made by 5 flags from In how many ways 5 boys and 3 girls can be seated in a
8 flags of different colours ? row so that no two girls are together ?

Sol. The total number of signals is the number of arrangements Sol. The 5 boys can be seated in a row in 5P5 = 5! ways. In each


of 8 flags by taking 5 flags at a time. of these arrangements 6 places are created, shown by the
cross-marks, as given below :
Hence, required number of signals =  8C5 × 5! = 6720
× B × B × B × B × B ×
Example – 14
Since no two girls are to sit together, so we may arrange 3
girls in 6 places. This can be done in  6P3  ways i.e. 3 girls
How  many  different  signals  can  be  given  using  any
can be seated in 6P3 ways.
number of flags from 5 flags of different colours ?
Hence, the total number of seating arrangements
Sol. The signals can be made by using at a time one or two or
= 5P5 × 6P3  = 5! × 6 × 5 × 4 = 14400.
three or four or five flags.
Hence, by the fundamental principle of addition, Example – 18
Total number of signals = 5P1 + 5P2 + 5P3 + 5P4 + 5P5
Find the number of ways in which 5 boys and 5 girls be
= 5 + 20 + 60 + 120 + 120 = 325 seated in a row so that

Example – 15 (i) No two girls may sit together.
(ii) All the girls sit together and all the boys sit together.
How many 4-letter words, with or without meaning, can
(iii) All the girls are never together.
be formed out of the letters of the word, ‘LOGARITHMS’,
if repetition of letters is not allowed ? Sol. (i) 5 boys can be seated in a row in  5P5 = 5! ways. Now, in
the 6 gaps 5 girls can be arranged in  6P5 ways.
Sol. There are 10 letters in the word ‘LOGARITHMS’.
Hence, the number of ways in which no two girls sit together
So, the number of 4 - letter word = 10C4 × 4! = 10P4 = 5040
= 5! × 6P5 = 5! × 6!
Example – 16 (ii) The  two  groups  of  girls  and  boys  can  be  arranged  in  2!
ways. 5 girls can be arranged among themselves in 5! ways.
How many different words can be formed with the letters
Similarly, 5 boys can be arranged among themselves in 5!
of the word EQUATION so that
ways.  Hence,  by  the  fundamental  principle  of  counting,
(i) the words begin with E ? the  total  number  of  requisite  seating  arrangements
(ii) the words begin with E and end with N ? = 2! (5! × 5!) = 2 (5!)2.
(iii) The total number of ways in which all the girls are never
Sol. Clearly, the given word contains 8  letters out of which 5
together
are vowels and 3 consonants.
= Total number of arrangements
(i) Since all words must begin with E. So, we fix E at the first
– Total  number  of  arrangements  in  which  all  the  girls  are
place. So, total number of words =  7P7 = 7 !
always together
(ii) Since all words must begin with E and end with N. So, we
= 10! – 5! × 6!
fix E at the first place and N at the last place.

Hence, the required number of words =  6P6 = 6!
148 PERMUTATION AND COMBINATION

Example – 19 Example – 22

Five  boys  and  five  girls  form  a  line  with  the  boys  and (i) How  many  different  words  can  be  formed  with
girls alternating. Find the number of ways of making the the letters of the word HARYANA ?
line. (ii) How many of these begin with H and end with N ?
Sol. 5 boys can be arranged in a line in 5P5 = 5! ways. Since the (iii) In how many of these H and N are together ?
boys and  girls are alternating. So, corresponding each of
the 5! ways of arrangements of 5 boys we obtain 5 places Sol. (i) There are 7 letters in which 3 are alike
marked by cross as shown below : 7! 7!
So, total number of words     840 .
(i) B1 × B2 × B3 × B4 × B5 × (ii) × B1 × B2 × B3 × B4× B5. 3!1!1!1!1! 3!
(ii) After fixing H in first place and N in last place, we have 5
Clearly, 5 girls can be arranged ir 5 places marked by cross letters out of which three are alike
in (5 ! + 5!) ways. 5!
So, total number of words    20 .
Hence, the total number of ways of making the line 3!
(iii) If H and N together we have  6 letters out of which 3 are
= 5! × (5! + 5!) = 2 (5!)2.
6!
alike.These 6 letters can be arranged in  ways. But H and
Example – 20 3!
N can be arranged amongst themselves in 2! ways.
How  many  five-letter  words  containing  3  vowels  and  2 6!
Hence, the requisite number of words   × 2! = 120 × 2 = 240.
consonants  can  be  formed  using  the  letters  of  the  word 3!
‘EQUATION’ so that the two consonants occur together ? Example – 23

Sol. There  are  5  vowels  and  3  consonants  in  the  word How  many  different  words  can  be  formed  by using  all
‘EQUATION’ 3 vowels out of 5 and 2 consonants out of 3 the letters of the word ‘ALLAHABAD’ ?
can  be  chosen  in  5C3  ×  3C2  ways. As  consonants  occur (i) In  how  many  of  them  vowels  occupy  the  even
positions?
together, Considering 2 consonants as one letter, we have
(ii) In how many of them both L do not come together ?
4  letters  which  can  be  arranged  in  4!  ways.  But  two
consonants  can  be  put  together  in  2!  ways.  Therefore,  5 Sol. There  are  9  letters  in  the  word  ‘ALLAHABAD’  out  of
which 4 are A’s, 2 are L’s and the rest are all distinct.
letters in each group can be arranged in 4! × 2! ways.

The required no. of words = (5C3 × 3C2) × 4!×2!=1440. 9!
So, the requisite number of words    7560 .
4! 2!
Example – 21
(i) Four A’s will occupy four even places in 1 way. Now, we
are left with 5 places and 5 letters, of which two are alike
How  many  words  with  or  without  meaning,  each  2  of
vowels and 3 consonants can be formed from the letters 5!
(2 L’s) and other distinct, can be arranged in  ways.
2!
of the word DAUGHTER ?

Sol. There  are  3  vowels  and  5  consonants  in  the  word 5! 4! 5!


Total no. of reqd. words      60 .
2! 4! 2!
DAUGHTER out of which 2 vowels and 3 consonants can
be chosen in 3C2 × 5C3 ways. These selected five letters can 8!
(ii) The no. of words in which both L come together =  = 1680.
now be arranged in 5! ways. 4!
Hence, required number of words Hence, the no. of words in which both L do not come together
3 5
=  C2 ×  C3 × 5! =  Total no. of words – No. of words in which both L come together
= 3 × 10 × 120 = 3600 = 7560 – 1680 = 5880.
PERMUTATION AND COMBINATION 149

Example – 24 Example – 25

How  many  four–letter  words  can  be  formed  using  the In how many ways 4 letters can be selected from the letters
letters of the word INEFFECTIVE ? of the word INEFFECTIVE ?

Sol. INEFFECTIVE contains 11 letters : Sol. INEFFECTIVE contains 11 letters :


EEE, FF, I I, C, T, N, V.
EEE, FF, II, C, T, N, V
As all letters are not different, we cannot use nPr. The four–
We will make following cases to select 4 letters.
letter  words  will  be  among  any  one  of  the  following
categories. Case–1 : 3 alike and 1 different
1.  3 alike letters, 1 different letter. * 3 alike letters can be selected from 1 set of 3 alike letters
2.  2 alike letters, 2 alike letters. (EEE) in 1 way.
3.  2 alike letters, 2 different letters.
 The number of way to select 3 alike letters = 1 ..(i)
4.  All different letters.
Case–2 : 2 alike and 2 alike
1. 3 alike, 1 different :
3 alike can be selected in one way i.e. EEE. * ‘2 alike and 2 alike’ means we have to select 2 groups of 2

Different letters can be selected from F, I, T, N, V, C in 6C1 alike letters (EE, FF, II) in  3C2 ways.


ways.  The number of ways to select “2 alike and 2 alike” letters
 No. of groups = 1 × 6C1 = 6
=  3C2 = 3.
4!
 No. of words = 6 ×  3!  1!  24 Case–3 : 2 alike and 2 different

* 1 group of 2 alike letters can be selected from 3 sets of 2
2. 2 alike, 2 alike :
alike letters (EE, FF, II) in  3C1 ways.
Two sets of 2 alike can be selected from 3 sets (EE, II, FF)
in  3C2 ways. * 2 different letters can be selected from 6 different (C, T, N,

4! V, remaining 2 sets of two letters alike) in ways.
 No. of words = 3C2 ×  2!  2!  18
The number of ways to select “2 alike and 2 different letters”
3C  ×  6C  = 3 × 15 = 45 ...(ii)
3. 2 alike, 2 different : 1 2

 No. of groups = (3C1) × (6C2) = 45 Case–4 : All different letters

4! * All different letters can be selected from 7 different letters
 No. of words = 45 ×  = 540
2! (I, E, F, N, C, T, V) in  7C4 ways.

4. All different :  The number of ways to select all different letters=  7C4 =


 No. of groups = 7C4 (out of  E, F, I, T, N, V, C) 35 ...(iii)
 No. of words = 7C4 × 4 ! = 840 Combining (i), (ii), (iii), we get
Hence total four–letter words
Total number of ways to select 4 letters from the letters of
= 24 + 18 + 540 + 840 = 1422.
the word ‘INEFFECTIVE”

= 1 + 3 + 45 + 35 = 84.
150 PERMUTATION AND COMBINATION

Example – 26 Example – 28

In how many ways can a cricket team be selected from a In how many ways can 7 departments be divided among 3
group  of  25  players  containing  10  batsmen  8  bowlers, ministers  such  that  every  minister  gets  at  least  one  and
5  all –rounders and  2 wicketkeepers  ? Assume that the atmost 4 departments to control ?
team  of  11  players requires  5  batsmen,  3  all–rounders.
2–bowlers and 1 wicketkeepeer. Sol. The ways  in which  we  can  divide  7  departments among  3
ministers such that each minister gets atleast 1 and atmost 4.
Sol. Divide the selection of team into four operation. S.No. M1 M2 M3
1 4 2 1
I : Selection of batsman can be done (5 from 10) in 10C5 ways.
2 2 2 3
II : Selection of bowlers can be done (2 from 8) in 8C2 ways. 3 3 3 1
III : Selection  of  all–rounders  can  be  done  (3  from  5)  in  5C3 Note : If we have a case (2, 2, 3), then there is no need to
make cases (3, 2, 2) or (2, 3, 2) because we will include
ways. them whem we apply distribution formula to distribute ways
of division among ministers.
IV : Selection of wicketkeeper can be done (1 from 2) in  2C1
Case I : We divide 7 departements among 3 ministers in
ways. number 4, 2, 1 i.e. unequal division. As any minister can
 the team can be selected in = 10C5 × 8C2 × 5C3 × 2C1 ways get 4 departments, any can get 2 any can get 1 department,
we should apply distribution formula.

10!  8  7 10  2 we get :


=   141120.
5! 5! 2! Number  of  ways  to  divide  and  distribute  department  in
number 4, 2, 1

Example – 27  7 
   3!  630 ....(i)
 4 2 1
A  mixed  doubles  tennis  game  is  to  be  arranged  from  5 Case II : It is ‘equal as well as unequal’ division. As any
married couples. In how many ways the game be arranged minister  can  get  any  number  of  departments,  we  use
complete distribution  formula.
if no husband and wife pair is included in the same game ?
we get :
Number of ways to divide departments in number 2, 2, 3,
Sol. To arrange the game we have to do the following operations.
 7 1
(i) Select two men from 5 men in  5C2 ways.   3!  630 ....(ii)
 2 2 3 2 
(ii) Select two women from 3 women excluding the wives of
Case III : It is also ‘equal as well as unequal’ division. As
the men already selected. This can be done in  3C2 ways. any minister  can get any number of departments, we  use
complete distribution of formula.
(iii) Arrange the 4 selected persons in two teams. If the selected
we get :
men are M1 and  M2 and the selected women are W1 and
Number of ways to divide and distribute in number 3, 3, 1
W2, this can be done in 2 ways :
 7 1
M1W1, play against M2W2  2
  3!  420 ....(iii)
  3  1 2 2 
M2W1 play against M1W2
Combining  (i),  (ii)  and  (iii),  we  get  number  of  ways  to
Hence the number of ways to arrange the game divide 7 departments among 3 minister
= 630 + 630 + 420 = 1680 ways.
= 5C2 3C2 (2) = 10  × 3 × 2 = 60
PERMUTATION AND COMBINATION 151

Example – 29 Example – 32

Find the exponent of 2 in 50 ! ?
In how many ways 6 letters can be placed in 6 envelopes
such  that
 50   50   50   50   50 
Sol. E 2 50!      2    3    4    5  (a) No letter is placed in its corresponding envelope.
 2  2  2  2  2 
(b) at least 4 letters are placed in correct envelopes.
= 25 + 12 + 6 + 3 + 1 = 47. (c) at most 3 letters are placed in wrong envelopes.
Example – 30
Sol. (a)  Using dearrangement  theorem.
Find the sum of all five–digit numbers that can be formed
using digits 1, 2, 3, 4, 5 if repetition is not allowed ? Number of ways to place 6 letters in 6 envelopes such that
all are placed in wrong envelopes.
Sol. There are 5! = 120 five digit numbers and there are 5 digits.
Hence by symmetry or otherwise we can see that each digit
 1 1 1 1
will appear in any place  6 1     ....  
 1 2 3 6
5!
(unit’s or ten’s or . . . . . .)   times.
5 = 360 – 120 + 30 – 6 + 1 = 265
 X = sum of digits in any place (b) Number  of  ways  to  place  letters  such  that  at  least  4

5! 5! 5! 5! 5! letters are placed in correct envelopes
 X  5   4   3   2  1
5 5 5 5 5 = 4 letters are placed in correct envelopes and 2 are
in wrong + 5 letters are placed in correct envelopes and 1
5! 5!
 X  5  4  3  2  1  15 
5 5 in wrong + All 6 letters are placed in correct envelopes

 the sum of all numbers = 6C4 × 1 + 0 (not possible to place 1 in wrong envelope) +
= X + 10X + 100X + 1000X + 10000X
65
= X (1 + 10 + 100 + 1000 + 10000) 1    1  16
2
5!
 (15)(1 + 10 + 100 + 1000 + 10000) (c) Number of ways to place 6 letters in 6 envelopes such
5
that  at  most  3  letters  are  placed  in  wrong  envelopes  =  0
= 24 (15) (11111) = 3999960
letter is wrong envelope and 6 in correct + 1 letter in wrong
Example – 31
envelope and 5 in correct 2 letters in wrong envelopes and
Find  the  number  of  ways  in  which  8  different  flowers 4  are  in  correct  +  3  letters  in  wrong  envelopes  and  3  in
can be strung to form a garland so that 4 particular flowers correct = 1 + 0 (not possible to place 1 in wrong envelope)
are never separated.
 1 1 1
Sol. Considering 4 particular flowers as one group of flower, we + 6C4 × 1 +  6C3   3 1    
 1 2 3
have five flowers (one group of flowers and remaining four
4!
flowers) which can be strung to form a garland in   ways.
2 6  5 6  5  4  3  3
=  1     
But 4 particular flowers can be arranged themselves in 4! 2 6 2 3
4!  4!
ways. Thus, the required number of ways =  = 288. = 1 + 15 + 20 × 2 = 56.
2
152 PERMUTATION AND COMBINATION

Example – 33 Example – 34

Find the number of ways in which 30 marks can be alloted In a box there are 10 balls, 4 red, 3 black, 2 white and 1
to 8 questions if each question carries atleast 2 marks. yellow. In how many ways can a child select 4 balls out
of  these  10  balls  ?  (Assume  that  the  balls  of  the  same
colour are identical)
Sol. Let x1, x2, x3, x4,. . . . . . . x8 be marks alloted to 8 questions.
As  total  marks  is  30,  we  can  make  following  integral Sol. Let x1, x2,  x3 and  x4 be  the number of red,  black, white,
equation : yellow balls selected respectively

x1 + x2  + x3 +. . . . . . . x8 = 30. No. of ways to select 4 balls = No. of integral solutions of


the eqn. x1 + x2 + x3 + x4 = 4
It is given that every question should be of atleast  2 marks.
It  means Conditions on x1, x2, x3 and x4
The total number of red, black, white and yellow balls in
2  xi  16    i = 1, 2, 3, . . . , 8.
the box are 4, 3, 2 and 1 respectively.
The number of solution of the integral equation is equal to
So we can take : Max (x1) = 4, Max (x2) = 3, Max (x3) = 2,
number of ways to divide marks. Max (x1) = 1
Number of solutions Number of ways to select 4 balls
= coeff. of x30 in (x2 +  x3 + . . . .+ x16)8 = coeff of x4 in (1 + x + x2 + x3 + x4) × (1 + x + x2 + x3)
= coeff. of x30 in x16  (1 + x + . . . .+ x14)8 (1 + x + x2) × (1 + x)
= coeff of x4 in (1 – x5) (1 – x4) (1 – x3) (1 – x2) (1 – x)–4
8
 1  x 15  = 7C4 –  5C2 –  4C1 –  3C0 = 20
= coeff. of x14 in   1 x


  Thus,  number  of  ways  of  selecting  4  balls  from  the  box
subjected to the given conditions is 20.
= coeff. of x14 in (1 – x)–8 = 21C14 = 116280. Alternate Solution :
Alternate Solution : The 10 balls are RRRR BBB WW Y (where R, B, W, Y
represent red, black, white and yellow balls respectively).
Let, the marks given in each question be ;
The  work  of  selection  of  the  balls  from  the  box  can  be
x1, x2, x3, x4, x5, x6, x7, x8,
divided into following categories.
[where xi’ s 0 (i = 1, 2,. . . .8)] Case–I : All alike
and x1 + x2 + x3 + x4 + x5 + x6 + x7 + x8 = 30 Number of ways of selecting all alike balls = 1C1 = 1

let ; x1 – 2 = y1, x2 – 2 = y2, x3 – 2 = y3, x4 – 2 = y4, x5 Case–II : 3 alike and 1 different


– 2 = y5, x6 – 2 = y6, x7 – 2 = y7, x8 – 2 = y8. Number of ways of selecting 3 alike and 1 different balls
= 2C1 ×  3C1 = 6
 y1 + y2 + y3 + y4 + y5 + y6 + y7 + y8 = 30 – 16
Case–III : 2 alike and 2 alike
 y1 + y2 + y3 + y4 + y5 + y6 + y7 + y8  = 14
Number of ways of selecting 2 alike and 2 alike balls = 3C2 = 3
where yi  0    i = 1, 2, 3, . . . , 8 Case–IV : 2 alike and 2 different
 Number of solutions of the above equation Number of ways of selecting 2 alike and 2 different balls =
3C 3
1 ×  C2 = 9
=  Number  of  ways  to  divide  14  identical  objects
Case–V : All different
among 8 groups such that each group gets 0 or more.
Number of ways of selecting all different balls =  4C4 = 1
 Number of solutions = 14 + 8 – 1C8 – 1 =  21C7.
Total number of ways to select 4 balls = 1 + 6 + 3 + 9 + 1 = 20.
PERMUTATION AND COMBINATION 153

EXERCISE - 1 : BASIC OBJECTIVE QUESTIONS


FPM 10. How many of the 900 three digit numbers have at least one
1. How  many  numbers  lying  between  500  and  600  can  be even digit ?
formed with the help of the digits 1, 2, 3, 4, 5, 6 when the (a) 775 (b) 875
digits are not to be repeated (c) 450 (d) 750
(a) 20 (b) 40 11. The number of six digit numbers that can be formed from the
(c) 60 (d) 80 digits 1, 2, 3, 4, 5, 6 & 7 so that digits do not repeat and the
2. 4 buses runs between Bhopal and Gwalior. If a man goes terminal digits are even is :
from Gwalior to Bhopal by a bus and comes back to Gwalior (a) 144 (b) 72
by another bus, then the total possible ways are (c) 288 (d) 720
(a) 12 (b) 16 12. A new flag is to be designed with six vertical strips using
(c) 4 (d) 8 some or all of the colours yellow, green, blue and red. Then,
3. The number of 3 digit odd numbers, that can be formed by the  number  of  ways  this  can  be  done  such  that  no  two
using the digits 1, 2, 3, 4, 5, 6 when the repetition is allowed, adjacent strips have the same colour is
is (a) 12 × 81 (b) 16 × 192
(a) 60 (b) 108 (c) 20 × 125 (d) 24 × 216
(c) 36 (d) 30 13. The number of 5 digit numbers such that the sum of their
digits is even is :
4. The number of all four digit numbers is equal to
(a) 50000 (b) 45000
(a) 9999 (b) 9000
(c) 60000 (d) none of these
(c) 104 (d) none of these
14. The number 2006 is made up of exactly two zeros and two
5. The number of all four digits numbers with distinct digits is
other digits whose sum is 8. The number of 4 digit numbers
(a) 9×10×10×10 (b) 10P4 with these properties (including 2006) is :
(c) 9 × 9P3 (d) none of these (a) 7 (b) 18
6. There are 4 letter boxes in a post office. In how many ways (c) 21 (d) 24
can a man post 8 distinct letters ?
n!
(a) 4 × 8 (b) 84
15. How many words can be made out from the letters of the
(c) 48 (d) P (8, 4) word  INDEPENDENCE,  in  which  vowels  always  come
7. The number of even numbers that can be formed by using together
the digits 1, 2, 3, 4 and 5 taken all at a time (without repetition) (a) 16800 (b) 16630
is
(c) 1663200 (d) None of these
(a) 120 (b) 48
16. In how many ways can six boys and five girls stand in a row
(c) 1250 (d) none of these
if all the girls are to stand together but the boys cannot all
8. The number of all three digit numbers having no digit as 5 is stand together  ?
(a) 252 (b) 225 (a) 172,800 (b) 432,000
(c) 648 (d) none of these (c) 86,400 (d) None of these
9. All possible three digits even numbers which can be formed 17. If the letters of the word ‘MOTHER’ are written in all possible
with the condition that if 5 is one of the digit, then 7 is the orders and these words are written out as in a dictionary,
next digit is : find the rank of the word ‘MOTHER’.
(a) 5 (b) 325 (a) 307 (b) 308
(c) 345 (d) 365 (c) 309 (d) 120
154 PERMUTATION AND COMBINATION

18. The letters of the word RANDOM are written in all possible (a) 60 (b) 12


orders and  these words are written out as in a  dictionary (c) 27 (d) 8
then the rank of the word RANDOM is
27. 5 Indian and 5 American couples meet at a party and shake
(a) 614 (b) 615 hands. If no wife shakes hands with her own husband and
(c) 613 (d) 616 no Indian wife shakes hands with a male, then the number
n
Cr of hand shakes that takes place in the party is :

19. Number of diagonals of a convex hexagon is (a) 95 (b) 110

(a) 3 (b) 6 (c) 135 (d) 150

(c) 9 (d) 12 28. A rack has 5 different pairs of shoes. The number of ways in


which 4 shoes can be chosen from it so that there will be no
20. A convex polygon has 65 diagonals. The number of its sides
complete pair is
is equal to
(a) 1920 (b) 200
(a) 13 (b) 10
(c) 110 (d) 80
(c) 22 (d) 11
29. A  question  paper  on  mathematics  consists  of  twelve
21. The interior angles of a regular polygon measure 150° each.
questions  divided  into  three  parts. A,  B  and  C,  each
The number of diagonals of the polygon is
containing  four  questions.  In  how  many  ways  can  an
(a) 35 (b) 44 examinee answer five questions, selecting atleast one from
(c) 54 (d) 78 each part.
22. On the  occasion of  Deepawali festival  each student  of a (a) 624 (b) 208
class  sends  greeting  cards  to  the  others.  If  there  are  20 (c) 2304 (d) none of these
students  in  the  class,  then  the  total  number  of  greeting
30. A class has 21 students. The class teacher has been asked
cards exchanged by the students is
to make n groups of r students each and go to zoo taking
(a)  20 C2 (b)  2 .20 C 2 one group at a time. The size of group for which teacher
goes to the maximum number of times is :
(c)  2.20 P2 (d) None of these (a) 9 (b) 10
23. A  father  with  8  children  takes  them  3  at  a  time  to  the (c) 11 (d) 12
Zoological gardens, as often as he can without taking the 31. A seven digit number with distinct digits is in form of abcdefg
same 3 children together more than once. The number of (g,  f,  e,  etc.  are  digits  at  units,  tens,  hundred  place  etc.)
times he will go to the garden is where a < b < c < d > e > f > g. The number of such numbers
(a) 336 (b) 112 are :
(c) 56 (d) None of these (a) 1981 (b) 2009
24. In  how  many ways  can  two balls  of  the  same  colour  be (c) 1560 (d) 1870
selected out of 4 distinct black and 3 distinct white balls 32. Number of rectangles in figure shown which are not squares
(a) 5 (b) 6 is :
(c) 9 (d) 8
25. In a touring cricket team there are 16 players in all including
5 bowlers and 2 wicket-keepers. How many teams of 11
players from these, can be chosen, so as to include three
bowlers and one wicket-keeper
(a) 650 (b) 720
(c) 750 (d) 800
26. Three  couples  (husband  and  wife)  decide  to  form  a
(a) 159 (b) 160
committee  of  three  members.  The  number  of  different
committee that can be formed in which no couple finds a (c) 161 (d) None of these
place is :
PERMUTATION AND COMBINATION 155

33. There are n points on a circle. The number of straight lines 40. There are m points on a straight line AB and n points on the


formed by joining them is equal to line AC none of them being the point A. Triangles are formed
(a) nC2 (b) nP2 with these points as vertices, when :

(c) nC2 – 1 (d) none of these (i) A is excluded.

34. The greatest possible  number of points of intersection of 9 (ii) A is included. The ratio of number of triangles in the two


different straight lines and 9 different circles in a plane is : cases is :

(a) 117 (b) 153 mn2 mn2


(a)  (b) 
(c) 270 (d) none of these mn m  n 1
35. 18 points are indicated on the perimeter of a triangle ABC
(see figure). How many triangles are there with vertices at mn2 m (n  1)
(c)  (d) 
these points ? mn2 (m  1) (n  1)

41. The maximum number of points of intersection of 7 straight
A
lines and 5 circles when 3 straight lines are parallel and 2
circles are concentric, is/are :
(a) 106 (b) 96
(c) 90 (d) None of these
B C n
Cr × r!
(a) 331 (b) 408 42. The number  of ways in which 5  boys and 3 girls can be
(c) 710 (d) 711 seated in a row so that each girl in between two boys

36. Out of 10  points in a plane 6 are in a  straight line. The (a) 2880 (b) 1880


number of triangles formed by joining these points are (c) 3800 (d) 2800
(a) 100 (b) 150 43. The total number of words which can be formed out of the
(c) 120 (d) None of these letters a, b, c, d, e, f taken 3 together, such that each word
contains at least one vowel, is
37. The number of straight lines that can be formed by joining
20 points no three of which are in the same straight line (a) 72 (b) 48
except 4 of them which are in the same line (c) 96 (d) none of these
(a) 183 (b) 186 44. The  number  of  different  seven  digit  numbers  that  can  be
(c) 197 (d) 185 written using only three digits 1, 2 and 3 under the condition
that the digit 2 occurs exactly twice in each number is :
38. There are n distinct points on the circumference of a circle.
The  number  of pentagons that can be formed with these (a) 672 (b) 640
points  as  vertices  is  equal  to  the  number  of  possible (c) 512 (d) none of these
triangles. Then the value of n is 45. Number  of  ways  in  which  4  boys  and  2  girls  (all  are  of
(a) 7 (b) 8 different heights) can be arranged in a line so that boys as
(c) 15 (d) 30 well as girls among themselves are in decreasing order of
height (from left to right), is :
39. There are three coplanar parallel lines. If any n points are
taken on each of the lines, the maximum number of triangles (a) 1 (b) 6!
with vertices at these points will be : (c) 15 (d) None of these
46. Three ladies have each brought a child for admission to a
(a)  3n 2 (n  1) (b)  3n 2 (n  1)  1
school. The head of the school wishes to interview six people
one by one, taking care that no child is interviewed before
(c)  n 2 (4n  3) (d) None of these
his/her  mother.  In  how  many  different  ways  can  the
interviews be arranged ?
(a) 6 (b) 36
(c) 72 (d) 90
156 PERMUTATION AND COMBINATION

Permutation of alike objects 53. The total number of ways of arranging the letters AAAA


47. The number of all possible  different arrangements of the BBB CC D E F in a row such that letters C are separated from
word “BANANA” is one another is
(a) 2772000 (b) 1386000
(a)  6 (b)  6  2  3
(c) 4158000 (d) none of these
6 54. An  old  man  while  dialing  a  7  digit  telephone  number
(c)  2 3 (d) none of these remembers that the first four digits consists of one 1’s, one
2’s and two 3’s. He also remembers that the fifth digit is
48. The number of words that can be formed by using the letters either a 4 or 5 while has no memorising of the sixth digit, he
of the word “MATHEMATICS”, taken all at a time is remembers that the seventh digit is 9 minus the sixth digit.
Maximum number of distinct trials he has to try to make sure
11
(a)  11 (b)  that he dials the correct telephone number, is
2 2 2
(a) 360 (b) 240
11 (c) 216 (d) none of these
(c)  (d) none of these
( 2)3 55. A library has ‘a’ copies of one book, ‘b’ copies of each of
49. The  number  of  ways  in  which  the  letters  of  the  word two  books,  ‘c’ copies of each  of three  books,  and  single
“ARRANGE” can be permuted such that R’s occur together copy each of ‘d’ books. The total number of ways in which
is these books can be arranged in a row is

(a  b  c  d)! (a  2b  3c  d)!
7 (a)  (b)  2 3
(a)  2 2 (b)  6 a!b!c! a! b!  c!

(a  2b  3c  d)!
6 (c)  (d) none of these
(c)  (d) none of these a!b!c!
2
56. If all permutations of the letters  of the word AGAIN  are
50. The  number  of  permutation  of  the  letters  of  the  word arranged as in dictionary, then fiftieth word is
HINDUSTAN such that neither the pattern ‘HIN’ nor ‘DUS’
(a) NAAGI (b) NAGAI
nor ‘TAN’ appears, are :
(c) NAAIG (d) NAIAG
(a) 166674 (b) 169194
57. If all the letters of the word “QUEUE” are arranged in all
(c) 166680 (d) 181434
possible manner as they are in a dictionary, then the rank of
51. Number of ways in which letters of the word ENGINEER can the word QUEUE is :
be arranged : th th
(a) 15 (b) 16
(a) so that no two alike letters are together is 960 th th
(c) 17 (d) 18
(b) so that the word starts with E but does not end with N is
58. The letters of word “RADHIKA” are permuted are arranged
900
in alphabetical order as in English dictionary. The number
(c) so that the word neither starts with E nor ends with N is of words the appear before the word “RADHIKA” is :
2460
(a) 2193 (b) 2195
(d) so that vowels occur in alphabetical order is 840
(c) 2119 (d) 2192
52. The total number of arrangements which can be made out of
Formation of Groups
the letters of the word ‘Algebra’, without altering the relative
position of vowels and consonants is 59. The number of ways in which 52 cards can be divided into 4
sets, three of them having 17 cards each and the fourth one
7! 7! having just one card
(a)  (b) 
2! 2!5! 52 ! 52 !
(a)  (b) 
(17!)3 (17!)3 3!
4!3!
(c) 4 ! 3 ! (d) 
2 51! 51!
(c)  (d) 
(17!)3 (17!)3 3!
PERMUTATION AND COMBINATION 157

60. The  number  of  ways  in  which  12  balls  can  be  divided 67. If a,b,c,d are odd natural numbers such that a + b + c + d = 20,
between two friends, one receiving 8 and the other 4, is then the number of  values of (a, b, c, d) is :
(a) 165 (b) 455
12! 12!2!
(a)  (b)  (c) 310 (d) 255
8!4! 8!4!
68. Number of ways in which 25 identical balls can be distributed
12! among Ram, shyam, Sunder and Ghanshyam such that at
(c)  (d) none of these
8!4!2! least 1, 2, 3, and 4 balls are given to Ram, Shyam, Sunder and
61. In an election three districts are to be canvassed by 2, 3 and Ghanshyam respectively, is :
5 men respectively. If 10 men volunteer, the number of ways (a)  18 C4 (b)  28 C3
they can be alloted to the different districts is :

10! 10! (c)  24 C3 (d)  18 C3


(a)  2! 3! 5! (b)  2! 5! 2
69. The total number of ways in which n  number of identical
balls can be put in n numbered boxed (1, 2, 3, ......... n) such
10! 10! th
that i  box contains at least i number of balls, is :
(c)  2! 2 5! (d)  2! 2 3! 5!
    2 2
(a)  n C n 1 (b)  n 1
Cn 1
62. Number of ways in which 5 different toys can be distributed
among 5 children if exactly one child do not get any toy is : n 2  n 2
(c)  2 C (d) None of these
(a) 1100 (b) 1200 n 1

(c) 1300 (d) 240 Total no. of combinations


63. The  number  of  ways  to  give  16  different  things  to  three 70. The total number of selections of atleast one fruit which can
persons A, B, C so that B gets 1 more than A and C gets 2 be made from 3 bananas, 4 apples and 2 oranges is
more than B, is :
(a) 39 (b) 315
16! (c) 512 (d) none of these
(a)  (b)  4!5!7!
4!5!7! 71. There are five different green dyes, four different blue dyes
and  three  different  red  dyes.  The  total  number  of
16! combinations of dyes that can be chosen taking at least one
(c)   (d)  3!5!8!
3!5!8! green and one blue dye is
12
Distribution of alike objects (a) 3255 (b) 2
64. The number of ordered triplets of positive integers which (c) 3720 (d) none of these
are solutions of the equation x + y + z = 100 is
72. Given 6 different toys of red colour, 5 different toys of blue
(a) 6005 (b) 4851 colour and 4 different toys of green colour. Combination of
(c) 5081 (d) none of these toys that can be chosen taking at least one red and one blue
toys are :
65. The total number of ways of selecting six coins out of 20
one rupee coins, 10 fifty paise coins and 7 twenty five paise (a) 31258 (b) 31248
coins is (c) 31268 (d) None of these
(a) 28 (b) 56 73. The total number of different combinations of one or more
37 letters  which  can  be  made  from  the  letters  of  the  word
(c)  C6 (d) none of these
‘MISSISSIPPI’ is
66. The total number of ways in which 11 identical apples can
be distributed among 6 children is that at least one apple is (a) 150 (b) 148
given to each child (c) 149 (d) None of these
(a) 252 (b) 462
(c) 42 (d) none of these
158 PERMUTATION AND COMBINATION

74. In an examination there are three multiple choice questions Circular Permutations


and each question has 4 choices out of which only one is 77. The number of ways in which seven persons can be arranged
correct. If all the questions are compulsory, then number of at a round table if two particular persons may not sit together
ways in which a student can fail to get all answers correct, is
is
(a) 480 (b) 120
(a) 11 (b) 12
(c) 80 (d) none of these
(c) 27 (d) 63
78. 12  persons  are  to  be  arranged  to  a  round  table.  If  two
75. Every one of the 5 available lamps can be switched on to particular persons among them are not to be side by side,
illuminate certain Hall. The total number of ways in which the total number of arrangements is
the hall can be illuminated, is : (a) 9 (10!) (b) 2 (10!)
(a) 32 (b) 31 (c) 45 (8!) (d) 10 !
(c) 5 (d) 5! 79. In how many ways can 12 gentlemen sit around  a round
76. Sum of all divisors of 5400 whose unit digit is 0, is : table so that three specified gentlemen are always together
(a) 5400 (b) 10800 (a) 9 ! (b) 10 !
(c) 16800 (d) 14400 (c) 3 ! 10! (d) 3 ! 9 !
80. In  how  many  ways  7  men  and  7  women  can  be    seated
around  a  round  table  such  that  no  two  women  can  sit
together

(a)  (7! )2 (b)  7 ! 6!

(c)  (6!)2 (d) 7 !


159 PERMUTATION AND COMBINATION

EXERCISE - 2 : PREVIOUS YEAR JEE MAINS QUESTIONS

1. A  student  is  to  answer  10  out  of  13  questions  in  an 7. If  the  letters  of  the  word  SACHIN  are  arranged  in  all
examination such that he must choose at least 4 from the possible  ways  and  these  words  are  written  out  as  in
first five questions. The number of choices available to dictionary,  then  the  word  SACHIN  appears  at  serial
him  is (2003) number (2005)

(a) 140 (b) 196 (a) 602 (b) 603

(c) 280 (d) 346 (c) 600 (d) 601

2. The number of ways in which 6 men and 5 women can dine 8. At  an  election,  a  voter  may  vote  for  any  number  of

at a round table, if no two women are to sit together, is candidates  not  greater  than  the  number  to  be  elected.

given by (2003) There are 10 candidates and 4 are to be elected. If a voter


votes for at least one candidate, then the number of ways
(a) 6!×5! (b) 30
in which he can vote, is (2006)
(c) 5!×4! (d) 7! × 5!
(a) 6210 (b) 385
3. If  nCr denotes  the  number  of  combinations  of  n  things
(c) 1110 (d) 5040
taken  r at  a  time,  then  the  expression
nC 9. The set S = {1, 2, 3,..., 12} is to be partitioned into
r+1 + nCr _ 1 + 2 × nCr equals (2003)
three sets A, B, C of equal size.  Thus,  A B  C = S,
(a)  n+2Cr (b) n+2Cr+1
A B  = B  C = A  C =  . The number of ways to
(c)  n+1Cr (d) n+1Cr+1
partition S is (2007)
4. The range of the function f(x) = 7–X PX–3 is (2004)
(a) 12!/3!(4!)3 (b) 12!/3!(3!)4
(a) {1, 2, 3} (b) {1, 2, 3, 4, 5, 6}
(c) 12!/(4!)3 (d) 12!/(3!)4
(c) {1,2,3,4} (d) {1,2,3,4,5} 10. In a shop there are five types of ice-creams available. A
5. How  many  ways  are  there  to  arrange  the  letters  in  the child buys six ice-creams.
word  GARDEN with the vowels in alphabetical order ? Statement I :  The  number  of  different  ways  the  child
(2004) can buy the six ice-creams is  10C5.

(a) 120 (b) 240 Statement II : The  number of different ways  the child


can  buy  the  six  ice-creams  is  equal  to  the  number  of
(c) 360 (d) 480
different ways of arranging 6 A’s and 4 B’s in a row.
6. The number of ways of distributing 8 identical balls in 3
(2008)
distinct boxes, so that none of the boxes is empty, is
(a) Statement I is false, Statement II is true
(2004)
(b) Statement I is true, Statement II is true; Statement II
(a) 5 (b) 21 is a correct explanation for Statement I
(c) 38 (d)  8C3 (c) Statement I is true, Statement II is true; Statement II
is not a correct explanation for Statement I

(d) Statement I is true, Statement II is false
PERMUTATION
160 AND COMBINATION 160
PERMUTATION AND COMBINATION

11. How many different words can be formed by jumbling the 16. Let Tn be the number of all possible triangles formed by


letters in the word MISSISSIPPI in which no two S are joining  vertices  of  an  n-sided  regular  polygon.  If
adjacent ? (2008) Tn + 1 – Tn = 10, then the value of n is (2013)

(a) 7.6C4 . 8C4 (b) 8. 6C4 . 7C4 (a) 7 (b) 5

(c) 6.7. 8C4 (d) 6.8. 7C4 (c) 10 (d) 8

12. From  6  different  novels  and  3  different  dictionaries,  4 17. The sum of the digits in the unit’s place of all the 4-digit


numbers formed by using the numbers 3, 4, 5 and 6, with
novels and 1 dictionary are to be selected and arranged
out repretition, is: (2014/Online Set–1)
in a row on the shelf so that the dictionary is always in
(a) 432 (b) 108
the middle. Then, the number of such arrangements is
(c) 36 (d) 18
(2009)
18. An eight digit number divisible by 9 is to be formed using
(a) at least 500 but less than 750 digits from 0 to 9 withou repeating the digits. The number
of ways in which this can be done is:
(b) at least 750 but less than 1000
(2014/Online Set–2)
(c) at least 1000
(a)  72  7  (b) 18  7 
(d) less than 500

13. There are 10 points in a plane, out of these 6 are collinear. (c)  40  7  (d)  36  7 

If N is the number of traingles formed by joining these 19. 8-digit numbers are formed using the digits 1, 1, 2, 2, 2, 3, 4,


points,  then (2011) 4. The number of such numbers in which the odd digits do
not occupy odd places, is : (2014/Online Set–3)
(a) N > 190 (b) N  100
(a) 160 (b) 120
(c) 100 < N  140 (d) 140 < N  190 (c) 60 (d) 48
14. Statement I : The number of ways distributing 10 identical 20. Two  women  and  some  men  participated  in  a  chess
tournament in which every participant played two games
balls in 4 distinct boxes such that no box is empty is 9C3.
with each of the other participants. If the number of games
Statement II : The number of ways of choosing any 3 that  the  men  played  between  themselves  exceeds  the
number of games that the men played with the women by
places from 9 different places is 9C3. (2011)
66,  then  the  number  of  men  who  participated  in  the
(a) Statement I is true, Statement II is true; tournament lies in the interval: (2014/Online Set–4)

Statement II is not a correct explanation for Statement I. (a) [8, 9] (b) [10, 12)


(c) (11, 13] (d) (14, 17)
(b) Statement I is true, Statement II is false.
21. The  number  of  integers  greater than 6,000  that  can  be
(c) Statement I is false, Statement II is true.
formed, using the digits 3, 5, 6, 7 and 8, without repetition,
(d) Statement I is true, Statement II is true. is: (2015)
Statement II is a correct explanation for Statement I. (a) 120 (b) 72
15. Assuming the balls to be identical except for difference in (c) 216 (d) 192
colours, the number of ways in which one or more balls 22. The number of ways of selecting 15 teams from 15 men
can be selected from 10 white, 9 green and 7 black balls is and 15 women, such that each team consists of a man and
a woman, is : (2015/Online Set–1)
(2012)
(a) 15 × 15! (b) 15! × 15!
(a) 880 (b) 629 (c) 15 × 14! (d) 15 × 15
(c) 630 (d) 879
PERMUTATION AND COMBINATION 161

23. If in a regular polygon the number of diagonals is 54, then 28. A man X has 7 friends, 4 of them are ladies and 3 are men.


the number of sides of this polygon is : His wife Y also has 7 friends, 3 of them are ladies and 4 are
(2015/Online Set–2) men. Assume X and Y have no common friends. Then the
total number of ways in which X and Y together can throw
(a) 12 (b) 6
a party inviting 3 ladies and 3 men, so that 3 friends of
(c) 10 (d) 9 each of X and Y are in this party is : (2017)
24. If all the worlds (with or without meaning) having five (a) 485 (b) 468
letters, formed using the letters of the word SMALL and (c) 469 (d) 484
arranged as in a dictionary; then the position of the word 29. If all the words, with or without meaning, are written using
the  letters of the  word  QUEEN  and are  arranged as  in
SMALL is : (2016)
English dictionary, then the position of the word QUEEN
(a) 59th (b) 52nd is : (2017/Online Set–1)

(c) 58th (d) 46th (a) 44th (b) 45th

25. If the four letter words (need not be meaningful ) are to be (c) 46th (d) 47th


formed  using  the  letters  from  the  word 30. The number of ways in which 5 boys and 3 girls can be
“MEDITERRANEAN” such that the first letter is R and seated  on  a  round  table  if  a  particular  boy  B 1  and  a
the  fourth letter is  E,  then  the  total number  of all such particular girl G1 never sit adjacent to each other, is :
words is : (2016/Online Set–1)
(2017/Online Set–2)
11! (a) 5 × 6! (b) 6 × 6!
(a)  3 (b) 110
 2! (c) 7! (d) 5 × 7!
(c) 56 (d) 59 31. From 6  different  novels  and 3  different  dictionaries,  4
novels and 1 dictionary are to be selected and arranged
15 15
2 Cr  in a row on a shelf so that the dictionary is always in the
26. The value of   r  15   is equal to : middle. The number of such arrangements is : (2018)
r 1  Cr 1 
(a) at least 750 but less than 1000
(2016/Online Set–1)
(b) at least 1000
(a) 560 (b) 680
(c) less than 500
(c) 1240 (d) 1085
(d) at least 500 but less than 750
n2
C 32. n-digit numbers are formed using only three digits 2, 5
27. If  n  2 6  11,  then n satisfies the equation :
P2 and 7. The smallest value of n for which 900 such distinct
numbers can be formed, is : (2018/Online Set–1)
(2016/Online Set–2)
(a) 6 (b) 7
(a)  n2  3n  108  0 (b)  n2  5n  84  0
(c) 8 (d) 9
(c)  n2  2n  80  0 (d)  n2  n  110  0 33. The number of four letter words that can be formed using
the letters of the word BARRACK is :
(2018/Online Set–2)
(a) 120 (b) 144
(c) 264 (d) 270
34. The number of numbers between 2,000 and 5,000 that can
be formed with the digits 0, 1, 2, 3, 4 (repetition of digits is
not allowed) and are multiple of 3 is :
(2018/Online Set–3)
(a) 24 (b) 30
(c) 36 (d) 48
162 PERMUTATION AND COMBINATION

EXERCISE - 3 : ADVANCED OBJECTIVE QUESTIONS

1. A committee of 5 is to be chosen from a group of 9 people. 8. Total 5–digit number divisible by 4 can be formed using 0,
Number of ways in which it can be formed if two particular 1, 2, 3, 4, 5, when the repetition of digits is allowed
persons either serve together or not at all and two other (a) 1250 (b) 875
particular persons refuse to serve with each other, is
(c) 1620 (d) 1000
(a) 41 (b) 36
9. If the letters  of the word MOTHER are  arranged in all
(c) 47 (d) 76
possible  orders  and  these  words  are  written  as  in  a
2. Seven  different  coins  are  to  be  divided  amongst  three dictionary, then the rank of the word MOTHER will be
persons. If no two of the persons receive the same number
(a) 240 (b) 261
of coins but each receives atleast one coin and none is left
over, then the number of ways in which the division may (c) 308 (d) 309
be made is : 10. The number of numbers divisible by 3 that can be formed
(a) 420 (b) 630 by four different even digits is :
(c) 710 (d) none of these (a) 18 (b) 36
3. Number of ways in which 7 people can occupy six seats, 3 (c) 20 (d) None of these
seats on each side in a first class railway compartment if
11. The number of possible outcomes in a throw of n ordinary
two  specified  persons  are  to  be  always  included  and dice in which at least one of the dice shows an odd number
occupy adjacent seats on the same side, is (5 !). k then k
is :
has the value equal to :
(a) 6n – 1 (b) 3n – 1
(a) 2 (b) 4
(c) 8 (d) none of these (c) 6n  – 3n (d) None of these

4. Number of different words that can be formed using all the 12. The number of times the digit 5 will be written when listing


letters of the word “DEEPMALA” if two vowels are togther integers from 1 to 1000 is :
and the other two are also together but separated from the (a) 271 (b) 272
first two is : (c) 300 (d) None of these
(a) 960 (b) 1200
(c) 2160 (d) 1440 1 1  1 2 
13. Let E =          ...  upto 50 terms, where [.]
5. In  a  unique  hockey  series  between  India  and  Pakistan, 3 50 3 50
they decide  to  play on  till a  team wins 5  matches. The is greatest integer function. The exponent of 2 in (E)! is :
number of ways in which the series can be won by India, (a) 13 (b) 15
if no match ends in a draw is :
(c) 17 (d) 19
(a) 126 (b) 252
14. The number of 4–digits numbers that can be made with
(c) 225 (d) none of these
the digits 1, 2, 3, 4 and 5 in which at least two digits are
6. The  number  of  all  possible  selections  of  one  or  more identical, is :
questions from 10 given questions, each question having
an alternative is : (a) 45 – 5! (b) 505

(a) 3
10 10
(b) 2  – 1 (c) 600 (d) None of these
10 10
(c) 3  – 1 (d) 2 15. The number of ways in which n different prizes can be
7. An  ice  cream  parlour  has  ice  creams  in  eight  different distributed amongst m (< n) persons if each is entitled to
varieties. Number of ways of choosing 3 ice creams taking receive at most n – 1 prizes, is :
atleast two ice creams of the same variety, is : (a) nm – n (b) mn
(a) 56 (b) 64 (c) mn – m (d) None of these
(c) 100 (d) none of these
PERMUTATION AND COMBINATION 163

16. A seven digit number divisible by 9 is to be formed by 24. The number of ways to select 2 numbers from {0, 1, 2, 3, 4}


using 7 out of numbers {1, 2, 3, 4, 5, 6, 7, 8, 9}. The number such that the sum of the squares of the selected numbers
of ways in which this can be done is is divisible by 5 are (repetition of digits is allowed).
(a) 7! (b) 2. (7)! (a) 13 (b) 11
(c) 3.(7)! (d) 4.7! (c) 17 (d) None of these
17. The  number  of  ways  of  arranging  m  number  out  of 25. The number of ways in which we can choose 2 distinct
1, 2, 3, . . . . ., n so that maximum is (n – 2) and minimum is integers from 1 to 100 such that difference between then
2 (repetitions of numbers is allowed) such that maximum at most 10 is :
and minimum both occur exactly once, (n > 5, m > 3) is (a) 40C2 (b) 70C2
(a) n – 3Cm – 2 (b) mC2 (n – 3)m – 2 (c) 100C2 – 99C2 (d) None of these
(c) m (m – 1) (n – 5)m – 2 (d) nC2 . nCm 26. A teacher takes 3 children from her class to the zoo at a
18. How  many  numbers  greater  than  1000,  but  not  greater time as often as she can, but does not take the same three
than  4000  can  be  formed  with  the  digits  0,  1,  2,  3,  4, children  to  the  zoo  more  than  once.  She  finds  that  she
repetition of digits being allowed : goes to the zoo 84 more than a particular child goes to the
(a) 374 (b) 375 zoo. The number of chidren in her class is :

(c) 376 (d) None of these (a) 12 (b) 10

19. If 33! is divible by 2n, then the maximum value of n is equal (c) 60 (d) None of these


to : 27. The number of ways of selecting 10 balls out of an unlimited
(a) 30 (b) 31 number of white, red, green and blue balls is

(c) 32 (d) 33 (a) 280 (b) 286

20. The total numbers of words that can be made by writing (c) 270 (d) None of these


the letters of the word PARAMETER so that no vowel is 28. Four  couples  (husband  and  wife)  decide  to  form  a
between two consonants is : committee  of  four  members.  The  number  of  different
(a) 1440 (b) 1800 committee that can be formed in which no couple finds a
place is :
(c) 2160 (d) None of these
(a) 10 (b) 12
21. Let p,q {1,2,3, 4}. The number of equations of the form
px2 +qx + 1 = 0 having real roots must be (c) 14 (d) 16

(a) 15 (b) 9 29. A variable name in a certain computer language must be


either an  alphabet  or  a  alphabet  followed  by a  decimal
(c) 7 (d) 8
digit. Total number of different variablef names that can
22. Number of ways in which 3 boys and 3 girls (all are of exist in that language is equal to :
different heights) can be arranged in a line so that boys as
(a) 280 (b) 290
well as girls among themselves are in decreasing order of
heigth (from left to right), is : (c) 286 (d) 296

(a) 1 (b) 6! 30. There are three papers of 100 marks each in an examination.


Then the no. of ways can a student get 150 marks such
(c) 20 (d) None of these
that he gets atleast 60% in two papers
23. There  are  20  questions  in  a  questions  paper.  If  no  two
(a) 3C2 × 32C2 (b) 4C3 × 32C2
students  solve  the  same  combination  of  questions  but
solve equal number of questions then the maximum number (c) 4C3 × 36C2 (d) 4C3 × 32C3
of students who appeared in the examination is :
(a) 20C9 (b) 20C11
(c) 20C10 (d) None of these
164 PERMUTATION AND COMBINATION

31. In an examination the maximum marks for each of three 36. We are required to form different words with the help of


papers is n and that for fourth paper is 2n. If marks obtained the letters of the word INTEGER. Let m1 be the number of
in each paper are whole numbers, then the number of ways words in which I and N are never together and m2 be the
in which a candidate can get 3n marks is number of words which begin with I and end with R, then
m1/m2 is given by :
1
(a)  (n – 1) (5n2 + 10n + 6) (a) 42 (b) 30
6
(c) 6 (d) 1/30
1 37. The number of selections of four letters from the letters of
(b)  (n + 1) (5n2 + 10n + 6)
6 the word ASSASSINATION is :
(a) 72 (b) 71
1
(c)  (n + 1) (5n2 + n + 6) (c) 66 (d) 52
6
38. The letters of the word SURITI are written in all possible
(d) None of these orders and these words are written out as a in a dictionary.
32. There are n concurrent lines and another line parallel to Then the rank of the word SURITI is :
one of them. The number of different triangles that will be (a) 236 (b) 245
formed by the (n + 1) lines, is
(c) 307 (d) 315
n  1n n  1n  2 39. n lines are drawn in a plane such that no two of them are
(a)  (b) 
2 2 parallel and no three of them are concurrent. The number
of different points at which these lines will cut is :
n n  1 n  1n  2
(c)  (d)  n 1
2 2
33. The sides AB, BC and CA of a triangle ABC have a, b and
(a)  k
k 1
(b) n (n – 1)

c interior points on them respectively, then find the number
(c) n2 (d) None of these
of  triangles  that  can  be  construced  using  these  interior
points as vertices. 40. A bag contains 2 Apples, 3 Oranges and 4 Bananas. The
number of ways in which 3 fruits can be selected if atleast
(a) a + b + cC3
one banana is always in the combination (Assume fruit of
(b) a + b + cC3  – (aC3 +  bC3 +  cC3) same species to be alike) is :
(c) a + b + c + 3C3 (a) 6 (b) 10
(d) None of these (c) 29 (d) 7
34. Every one of the 10 available lamps can be switched on to 41. Find number of arangements of 4 letters taken from the
illuminate certain Hall. The total number of ways in which word EXAMINATION.
the hall can be illuminated, is :
(a) 2454 (b) 2500
(a) 55 (b) 1023
(c) 2544 (d) None of these
(c) 210 (d) 10!
42. The number of ways in which the sum of upper faces of
35. There  are  n  different  books  and  p  copies  of  each  in  a four distinct dices can be six.
library. The number of ways in which one or more books
(a) 10 (b) 4
can be selected is :
(c) 6 (d) 7
(a) pn + 1 (b) (p + 1)n – 1
43. Total number of positive integral solution of
(c) (p + 1)n – p (d) pn
15 < x1 + x2 + x3  20 is equal to
(a) 1125 (b) 1150
(c) 1245 (d) None of these
PERMUTATION AND COMBINATION 165

44. The number of subsets of the set A = {a1, a2, . . . . , an} 52. The  minimum  marks  required  for  clearing  a  certain
which contain even number of elements is screening paper is 210 out of 300. The screening paper
(a) 2n – 1 (b) 2n – 1 consists of ‘3’ sections each of Physics, Chemistry, and
Maths. Each section has 100 as maximum marks. Assuming
(c) 2n – 2 (d) 2n
there is no negative marking and marks obtained in each
45. The number of ways of choosing triplets (x, y, z) such that section are integers, the number of ways in which a student
z  max {x, y} and x, y, z  {1, 2, . . . . , n} is can qualify the examinatin is (Assuming no cut–off limit) :
(a) n + 1C3 + n + 2 C3 (b) n (n + 1) (2n + 1) (a) 210C3 – 90C3 (b) 93C3
(c) 11 + 22 + . . . + (n – 1)2 (d) None of these (c) 213C3 (d) (210)3
46. The number of functions from the set A = {0, 1, 2} into the 53. The number of ways in which 10 candidates A1, A2, . . ., A10
set B = {0, 1, 2, 3, 4, 5, 6, 7} such that f (i)  f (j) for i < j and can be ranked so that A1 is always before A2 is :
i, j,  A is
(a) 8C3 (b) 8C  + 2 ( 8 C ) 10!
3 2 (a)  (b) 8! × 10C2
2
(c) 10C3 (d) None of these
(c) 10P2 (d) 10C2
47. The integers from 1 to 1000 are written in order around a
circle. Starting at 1, every fifteenth number is marked (that 54. The  number  of  ways  of  distributing  10  different  books
is 1, 16, 31, etc.) This process is continued until a number among  4  students  (S1  –  S4)  such  that  S1  and  S2  get  2
is reached which has already been marked, then unmarked books each and S3 and S4 get 3 books each is :
numbers are (a) 12600 (b) 25200
(a) 200 (b) 400
10 !
(c) 600 (d) 800 (c) 10C4 (d)  2! 2! 3! 3!
48. The number of ways in which we can choose 3 squares of 55. The number of non–negative integral solutions of
unit area on a chess board such that one of the squares
has its two sides common to other two squares x1 + x2 + x3 + 4x4 = 20 is :

(a) 290 (b) 292 (a) 530 (b) 532

(c) 294 (d) 296 (c) 534 (d) 536

49. The number of times of the digits 3 will be written when 56. If  a,  b,  c  are  three  natural  numbers  in  AP  and
listing the integer from 1 to 1000 is : a + b + c = 21 then the possible number of values of the
ordered triplet (a, b, c) is :
(a) 269 (b) 300
(a) 15 (b) 14
(c) 271 (d) 302
(c) 13 (d) None of these
50. The total number fo six digit numbers x1 x2 x3 x4 x5 x6 having
the property x1 < x2  x3 < x4 < x5  x6, is equal to : 57. If 3n different things can be equally distributed among 3
persons in k ways then the number of ways to divide the
(a) 11C6 (b) 16C2 3n things in 3 equal groups is :
(c) 17C2 (d) 18C2
k
51. The number of integral solutions of x1 + x2 + x3 = 0, with (a) k × 3! (b) 
xi  – 5, is : 3!

(a) 15C2 (b) 16C2 (c) (3!)k (d) None of these


(c) 17C2 (d) 18C2 58. A is a set containing n elements. A subset P of A is chosen.
The set A is reconstructed by replacing the elements of
P.A subset Q of A is again chosen. The number of ways of
choosing  P  and  Q  so  that  P    Q  contains  exactly  two
elements is :
(a) 9 . nC2 (b) 3n – nC2
(c) 2 . nCn (d) None of these
166 PERMUTATION AND COMBINATION

59. The  number  of  different  ways  the  letters  of  the  word 65. The number of odd proper divisors of 3p . 6m . 21n is :
VECTOR can be placed in 8 boxes given below such that (a) (p + 1) (m + 1) (n + 1) – 2
no row remians empty is equal to :
(b) (p + m + n + 1) (n + 1) – 1
(c) (p + 1) (m + 1) (n + 1) – 1
(d) None of these
66. The  number  of  rectangles  excluding  squares  from  a
rectangle of size 9 × 6 is :
(a) 945 (b) 791
(a) 26 (b) 26 × 6!
(c) 154 (d) 364
(c) 6! (d) 2! × 6!
67. In the figure, two 4–digit numbers are to be formed by
60. In the next world cup of cricket there will be 12 teams,
filling the places with digits. The number of different ways
divided equally in two groups. Teams of each group will
in which the places can be filled by digits so that the sum
play a match against each other. From each group 3 top
of the numbers formed is also a 4–digit number and in no
teams will qualify for the next round. In this round each
place the addition is with carrying is :
team will play against others once. Four top teams of this
round will qualify for the semifinal round, when each team
will play against the others once. Two top teams of this
round will go to the final round, where they will play the
best of three matches. The minimum number of matches in
the next world cup will be :
(a) 54 (b) 53
(a) 554 (b) 364
(c) 52 (d) None of these
(c) 454 (d) 36 (553)
61. The lines intersect at O. Points A1, A2 . . ., An are taken on
68. Given 11 points, of which 5 lies on one circle, other than
one of them and B1, B2, . . . , Bn on the other, the number of
these 5, no 4 lie on one circle. Then the maximum number
triangle that can be drawn with the help of these (2n + 1)
of circles that can be drawn so that each contains atleast
points is :
three of the given points is :
(a) n (b) n2
(a) 216 (b) 156
(c) n3 (d) n4
(c) 172 (d) none of these
62. ABCD is a convex quadrilateral 3, 4, 5 and 6 points are marked
69. One hundred management students who read at least one
on the sides, AB, BC, CD and DA respectively. The number
of  the  three  business  magazines  are  surveyed  to  study
of triangles with vertices on different sides is :
the readership pattern. It is found that 80 read Business
(a) 270 (b) 220 India, 50 read Business world, and 30 read Business Today.
(c) 282 (d) 342 Five  students  read  all  the  three  magazines.  How  many
63. There are three coplanar parallel lines. If any p points are read exactly two magazines ?
taken on each of the lines, the maximum number of triangles (a) 50 (b) 10
with vertices at these points is : (c) 95 (d) 25
(a) 3p2(p – 1) + 1 (b) 3p2(p – 1) 70. There are 100 different books in a shelf. Number of ways
(c) p2(4p – 3) (d) None of these in which 3 books can be selected so that no two of which
64. The number of  divisors  of 23 .  33 .  53 .  75 of  the form are neighbours is :
100 97
4n + 1, n  N is : (a)  C3 – 98 (b)  C3
96 98
(a) 46 (b) 47 (c)  C3 (d)  C3
(c) 96 (d) 94
PERMUTATION AND COMBINATION 167

71. Two classrooms A and B having capacity of 25 and 78. There are 12 books on Algebra and Calculus in our library,


(n  –  25)  seats  respectively. An  denotes  the  number  of the  books  of  the  same  subject  being  different.  If  the
possible  seating  arrangments  of  room  ‘A’,  when  ‘n’ number of selections each of which consists of 3 books
students  are  to  be  seated  in  these  rooms,  starting  from on  each  topic  is  greatest  then  the  number  of  books  of
room ‘A’ which is to be filled up full to its capacity. Algebra and Calculus in the library are respectively :
49
If An – An – 1 = 25 ! ( C25) then ‘n’ equals (a) 3 and 9 (b) 4 and 8
(a) 50 (b) 48 (c) 5 and 7 (d) 6 and 6
(c) 49 (d) 51 79. A disarranged number from the set 1 – 9 is an arrangement
72. The number of ways in which we can choose 6 chocolates of all these number so that all numbers take up its unusual
out of 8 different brands available in the market is position. (e.g. 1 is in any place other than the first position,
13 13 2 is in any place other than the second position ......... all
(a)  C6 (b)  C8
the way to 9). Number of ways in which at least six numbers
6
(c) 8 (d) none of these take up their usual positions, is
73. The sum of all numbers greater than 1000 formed by using (a) 84 (b) 168
digits 1, 3, 5, 7 no digit being repeated in any number is :
(c) 205 (d) none of these
(a) 72215 (b) 83911
80. The combinatorial coefficient C(n, r) is equal to
(c) 106656 (d) 114712
(a) number of possible subsets of r members from a set
74. Number  of  positive  integral  solutions  satisfying  the of n distinct members.
equation (x1 + x2 + x3) (y1 + y2) = 77, is
(b) number of possible binary messages of length n with
(a) 150 (b) 270 exactly r 1’s.
(c) 420 (d) 1024 (c) number of non decreasing 2–D paths from the lattice
75. Distinct 3 digit numbers are formed using only the digit point (0, 0) to (r, n)
1, 2, 3 and 4 with each digit used at most once in each number (d) number of ways of selecting r things out of n different
thus formed. The sum of all possible numbers so formed is : things when a particular thing is always included plus
(a) 6660 (b) 3330 the number of ways of selecting ‘r’ things out of n,
when a particular thing is always excluded
(c) 2220 (d) none of these
76. The streets of a city are arranged like the lines of a chess 81. Identify the correct statement(s)
board. There are m streets running North to South and ‘n’ (a) Number of naughts standing at the end of  125  is
streets running East to West. The number of ways in which
30.
a man can travel from NW to SE corner going the shortest
possible distance is : (b) A telegraph has 10 arms and each arm is capable of 9
distinct positions excluding the position of rest. The
(a)  m 2  n 2 (b)  m  12 .n  12 number  of  signals  that  can  be  transmitted  is
10
10  – 1.
m  n  ! m  n  2 ! (c) Number of numbers greater than 4 lacs which can
(c)  m! . n! (d)  be formed by using only the digit 0, 2, 2, 4, 4 and 5
m  1 !. n  1 !
is 90.
77. An  ice  cream  parlour  has  ice  creams  in  eight  different (d) In a table tennis tournament, every player plays with
varieties. Number of ways of choosing 3 ice creams taking every other player. If the number of games played is
atleast two ice creams of the same variety, is : 5050 then the number of players in the tournament is
(a) 56 (b) 64 100.
(c) 100 (d) none of these
(Assume that ice creams of the same variety are identical and
available in unlimited supply)
168 PERMUTATION AND COMBINATION
n–1
82. There  are 10  questions, each question is either True or 85. The combinatorial coefficient  Cp denotes
False. Number of different sequences of incorrect answers (a) the number of ways in which n things of which p are
is also equal to alike and rest different can be arranged in a circle.
(a) Number  of  ways  in  which a  normal  coin  tossed  10 (b) the number of ways in which p different things can be
times would fall in a definite order if both Heads and selected out of n different thing if a particular thing is
Tails are present. always excluded.
(b) Number of ways in which a multiple choice question (c) number of ways in which n alike balls can be distributed
containing 10 alternatives with one or more than one in p different boxes so that no box remains empty and
each box can hold any number of balls.
correct alternatives, can be answered.
(d) the number of ways in which (n – 2) white balls and p
(c) Number of ways in which it is possible to draw a sum
black balls can be arranged in a line if black balls are
of  money  with  10  coins  of  different  denominations
separated, balls are all alike except for the colour.
taken some or all at a time.
86. The maximum number of permutations of 2n letters in which
(d) Number of  different selections  of 10  indistinguishable there are only a’s and b’s, taken all at time is given by :
things takes some or all at a time. 2n
(a)  Cn
83. The continued  product, 2 . 6 . 10 . 14 ........ to n factors is
equal to : 2 6 10 4n  6 4n  2
(b)  . . .... .
2n 1 2 3 n 1 n
(a)  Cn
2n
(b)  Pn n  1 n  2 n  3 n  4 2n  1 2n
(c)  . . . .... .
(c) (n + 1) (n + 2) (n + 3) ....... (n + n) 1 2 3 4 n 1 n
(d) none of these
2 n 1.3.5....(2n  3)(2n  1)
84. The number of ways in which five different books to be (d) 
n!
distributed among 3 persons so that each person gets at
least one books, is equal to the number of ways in which 87. Number of ways in which 3 numbers in A.P. can be selected
from 1, 2, 3, ...... n is :
(a) 5 persons are allotted 3 different residential flats so
that and each person is alloted at most one flat and no 2
 n 1 
two persons are alloted the same flat. (a)     if n is even
 2 
(b) number  of  parallelograms  (some  of  which  may  be
overlapping) formed by one set of 6 parallel lines and n ( n  2)
other set of 5 parallel lines that goes in other direction. (b)   if n is odd
4
(c) 5 different toys are to be distributed among 3 children,
so that each child gets at least one toy. (n  1)2
(c)   if n is odd
4
(d) 3 mathematics professors are assigned five different
lectures to be delivered, so that each professor gets at n ( n  2)
least one lecture. (d)   if n is even
4
88. If P (n, n) denotes the number of permutations of n different
things taken all at a time then P (n, n) is also identical to
(a) r ! P (n, n – r) (b) (n – r) . P (n, r)
(c) n . P (n – 1, n – 1) (d) P (n, n – 1)
where 0 < r < n
PERMUTATION AND COMBINATION 169

89. Which of the following statements are correct ? 92. The  number  of  shortest  routes  which  passes  through
(a) Number of words that can be formed with 6 only of the junction P and R:
letters of the word “CENTRIFUGAL” if each word must (a) 144 (b) 240
contain all the vowels is 3 . 7 ! (c) 216 (d) none of these
(b) There are 15 balls of which some are white and the rest
Match the Column
black. If the number of ways in which the balls can be
arranged in a row, is maximum then the number of white 93. Column–I Column–II
m
balls must be equal to 7 or 8. Assume balls of the same (a) Number of increasing (p) n
colour to be alike. permutations of m symbols
(c) There are 12 things, 4 alike of one kind, 5 alike and of are there from the n set
another kind and the rest are all different. The total
numbers {a1, a2, ... , an} where
number of combinations is 240.
the order among the numbers is
(d) Number of selections that can be made of 6 letters from
the word “COMMITTEE” is 35. given by a1 < a2 < a3 < ... an–1 < an is
m
Passage (Q.No. 90 to 92) (b) There are m men and n monkeys. (q) Cn

Consider  the  network  of  equally  spaced  parallel  lines Number of ways in which every


(6 horizontal and 9 vertical) shown in the figure. All small monkey has a master, if a man can
squares are of the same size. A shortest route from A to C have any number of monkeys
is defines as a route consisting 8 horizontal steps and 5 n
(c) Number of ways in which n red (r) Cm
vertical steps.
balls and (m – 1) green balls can
be arranged in a line, so tha no
two red balls are together, is
(balls of the same colour are alike)
n
(d) Number of ways in which ‘m’ (s) m
different toys can be distributed
in ‘n’ children if every child may
receive any number of toys, is
90. The number of shortest routes through the junction P is:
(a) 240 (b) 216
(c) 560 (d) none of these
91. The number of shortest routes which go following street
PQ must be :
(a) 324 (b) 350
(c) 512 (d) none of these
170 PERMUTATION AND COMBINATION

94. Column–I Column–II 95. 5 balls are to be placed in 3 boxes. Each box can hold all


(a) Four different movies are (p) 11 the  5  balls.  Number  of  ways  in  which  the  balls  can  be
placed so that no box remains empty, if :
running in a town. Ten students
Column–I Column–II
go to watch these four movies.
(a) balls are identical but boxes are (p) 2
The number of ways in which
different
every movie is watched by atleast
(b) balls are different but boxes are (q) 25
one student, is (Assume each way
identical
differs only by number of students
(c) balls as well as boxes are identical (r) 50
watching a movie)
(d) balls as well as boxes are identical
(b) Consider 8 vertices of a regular (q) 36
but boxes are kept in a row (s) 6
octagon and its centre. If T
You may note that two or more entries of column–I can match
denotes the number of triangles
with only entry of column–II
and S denotes the number of
96. 10 identical balls are to be distributed in 5 different boxes
straight lines that can be formed kept in a row and labled A, B, C, D and E. Find the number
with these 9 points then the value of ways in which the balls can be distributed in the boxes
if no two adjacent boxes remain empty.
of (T – S) equals
(c) In an examination, 5 children were (r) 52 97. The  number  of  non  negative  integral  solution  of  the
inequation x + y + z + w < 7 is ..........
found to have their mobiles in
98. How many numbers greater than 1000 can be formed from
their pocket. The Invigilator fired the digits 112340 taken 4 at a time.
them and took their mobiles in 99. Number  of  ways  in  which  12  identical  coins  can  be
his possession. Towards  the end distributed in 6 different purses, if not more than 3 and not
of the test, Invigilator randomly less than 1 coin goes in each purse is ...............
returned their mobiles. The number 100. In  how  many  ways  it  is  possible  to  select  six  letters,
including at least one vowel from the letters of the word
of ways in which at most two
“F L A B E L L I F O R M”. (It is a picnic spot in U. S. A.)
children did not get their own
moblies is
(d) The product of the digits of 3214 (s) 60
is 24. The number of 4 digit natural
numbers such that the product of
their digits is 12, is
(e) The number of ways in which a (t) 84
mixed double tennis game can
be arranged from amongst 5
married couple if no husband
and wife plays in the same game, is
171 PERMUTATION AND COMBINATION

EXERCISE - 4 : PREVIOUS YEAR JEE ADVANCED QUESTIONS


Single Answer 8. The  number  of  arrangements  of the  letters  of  the  word
1. Ten different letters of an alphabet are given. Words with BANANA in which the two N’s do not appear adjacently,
five letters are formed from these given letters. Then, the is (2002)
number of words which have at least one letter repeated, (a) 40 (b) 60
is (1980) (c) 80 (d) 100
(a) 69760 (b) 30240 9. If r, s, t are prime numbers and p, q are the positive integers
(c) 99748 (d) None of these such that LCM of p, q is r2s4t2, then the number of ordered
pairs (p, q ) is (2006)
5
52  j (a) 252 (b) 254
2. The value of the expression 47C4 +  j1
C 3  is equal to
(c) 225 (d) 224
(1980) 10. The letters of the word COCHIN are permuted and all the
(a) 47C5 (b) 52C5 permutations are arranged in an alphabetical order as in
an English dictionary. The number of words that appear
(c)52C4 (d)None of these before the word COCHIN is (2007)
3. Eight chairs are numbered 1 to 8. Two women and three (a) 360 (b) 192
men wish to occupy one chair each. First the women choose
the chairs from amongst the chairs marked 1 to 4, and then (c) 96 (d) 48
the men select the chairs from amongst the remaining. The 11. The number of seven digit integers, with sum of the digits
number of possible arrangements is equal to 10 and formed by using the digits 1,2 and 3 only,
(1982) is (2009)
(a)6C3 × 4C2 (b)4P 4 (a) 55 (b) 66
2 ×  P3
(c) 4C2 + 4P3 (d) None of these (c) 77 (d) 88
4. A five digits number divisible by 3 is to be formed using 12. The total  number of  ways in  which 5  balls  of  different
the numbers 0, 1 , 2, 3 , 4 and 5, without repetition. The colours can be distributed among 3 persons so that each
total number of ways this can be done, is (1989) person gets atleast one ball is (2012)
(a) 216 (b) 240 (a) 75 (b) 150
(c) 600 (d) 3125 (c) 210 (d) 243
5. An  n digit  number  is  a  positive  number  with  exactly  n 13. Let S = {1, 2, 3,…..,9}. For k = 1, 2,…..,5, let Nk be the
digits.  Nine  hundred  distinct  n-digit  numbers  are  to  be number  of  subsets  of  S,  each  containing  five  elements
formed using only the three digits 2,5 and 7. The smallest out  of  which  exactly  k  are  odd.  Then
value of n for which this is possible, is (1998) N1 + N2 + N3 + N4 + N5= (2017)
(a) 6 (b)7 (a) 125 (b) 210
(c) 252 (d) 126
(c) 8 (d) 9
Passage Q. 14 and 15
6. Number of divisors of the form (4n + 2), n  0 of the integer
Let an  denote the number of all n-digit positive integers
240 is (1998)
formed by the digits 0, 1 or both such that no consecutive
(a) 4 (b) 8 digits in them are 0. Let bn = The number of such n-digit
(c) 10 (d) 3 integers ending with digit 1 and cn = The number of such
7. How  many  different  nine  digit  numbers  can  be  formed n-digit integers ending with digit 0. (2012)
from the number 22 33 55 888 by rearranging its digits so 14. Which of the following is correct ?
that the odd digits occupy even positions (2000) (a) a17 = a16 + a15 (b) c17  c16 + c15
(a) 16 (b) 36 (c) b17  b16 + c16 (d) a17 = c17 + b16
(c) 60 (d) 180 15. The value of b6 is
(a) 7 (b) 8
(c) 9 (d) 11
PERMUTATION
172 AND COMBINATION 172
PERMUTATION AND COMBINATION

16. Six cards and six envelopes are numbered 1,2,3,4,5,6 and Column A Column B


cards are to be placed in envelopes so that each envelope
contains  exactly  one  card  and  no  card  is  placed  in  the (A) The value of 1 is (P) 136
envelope bearing the same number and moreover the card (B) The value of  2 is (Q) 189
numbered  1  is  always  placed  in  envelope  numbered  2.
Then the number of ways it can be done is (2014) (C) The value of  3 is (R) 192
(a) 264 (b) 265
(D) The value of  4 is (S) 200
(c) 53 (d) 67
17. A debate club consists of 6 girls and 4 boys. A team of 4 (T) 381
members is  to  be selected  from this club including the (U) 461
selection of a captain (from among these 4 members) for Fill in the Blanks
the team. If the team has to include at most one boy, then
20. In  a  certain  test,  ai  students  gave  wrong  answers  to  at
the number of ways of selecting the team is (2016)
least i question, where i = 1, 2,...., k. No student gave more
(a)  (b)  that k wrong answers. The total number of wrong answer
(c) 2 (d)  given is... . (1982)
Match the Columns 21. Total number of ways in which six ‘+’ and four ‘–’ signs
18. Consider  all  possible  permutations  of  the  letters  of  the can be arranged in a line such that no two ‘–’ signs occur
word ENDEANOEL. (2008) together is... . (1988)

(A) The number of permutations (p) 5! 22. There are four balls of different colours and four boxes of


colours, same as those of the balls. The number of ways in
contianing the word ENDEA, is which the balls, one each in a box, could be placed such
(B) The number of permutations in (q) 2 × 5! that a ball does not go to a box of its own colour is.... .
which the letter E occurs in the (1992)
first and the last positions, is 23. The number of 5 digit numbers which are divisible by 4,
with digits from the set {1, 2, 3, 4, 5} and the repetition of
(C) The number of permutations in (r) 7 × 5! digits is allowed, is _______. (2018)
which none of the letters D, L, N True/False
occurs in the last five positions, is 24. The  product  of  any  r  consecutive  natural  numbers  is
(D) The number of permutations in (s) 21 × 5! always divisible by r !. (1985)
which the letters A, E, O occur Analytical & Descriptive Questions
only in odd positions, is 25. If nCr – 1 = 36, nCr = 84 and nCr+1 = 126, then find the values
19. In a high school, a committee has to be formed from a of n and r. (1979)
group of 6 boys M1, M2, M3, M4, M5, M6 and 5 girls G1, G2, 26. In how many ways can a pack of 52 cards be
G3, G4, G5. (a) divided equally among four players in order
(i)     Let  1 be  the  total  number  of  ways  in  which  the (b) divided into four groups of 13 cards each
committee can be formed such that the committee has 5 (c) divided in 4 sets, three of them having 17 cards each
members, having exactly 3 boys and 2 girls. and the fourth just one card ? (1979)
(ii)   Let   2   be  the  total  number  of  ways  in  which  the 27. Five balls of different colours are  to be placed in three
committee can be formed such that the committee has at boxes of different sizes. Each box can hold all five. In how
least  2  members,  and  having  an  equal  number  of  boys many different ways can we place the balls so that no box
and girls. remains empty ? (1981)
28. mn squares of equal size are arranged to form a rectangle
(iii) Let   3  be  the  total  number  of  ways  in  which the of dimension m by n where m and n are natural numbers.
committee can be formed such that the committee has 5 Two squares will be called ‘neighbours’ if they have exactly
members, at least 2 of them being girls. one  common  side. A  natural  number  is  written  in  each
(iv)  Let   4   be  the  total  number  of  ways  in  which  the square such that the number in written any square is the
arithmetic mean of the numbers written in its neighbouring
committee can be formed such that the committee has 4 squares. Show that this is possible only if all the numbers
members, having at least 2 girls and such that both M1 used are equal. (1982)
and G1 are NOT in the committee together. (2018)
PERMUTATION AND COMBINATION 173

29. m men and n women are to be seated in a row so that no


two  women  sit  together.  If  m > n, then  show  that  the n 2!
35. Using permutation or otherwise, prove that    is an
number of ways in which they can be seated, is n!n
m!(m  1)! integer, where n is a positive integer. (2004)
(m  n  1)! (1983) 36. Two planes P1 and P2 pass through origin. Two lines L1
and L2 also passing through origin are such that L1 lies on
30. 7 relatives of a man comprises 4 ladies and 3 gentlemen, P1 but not on P2 , L2 lies on P2 but not on P1, A, B, C are
his wife has also 7 relatives ; 3 of them are ladies and 4 three  points  other  than  origin,  then  prove  that  the
gentlemen. In how  many ways can they  invite a  dinner permutation [A’ B’ C’ ] of [ABC] exists. Such that
party of 3 ladies and 3 gentlemen so that there are 3 of
(a) A lies on L1, B lies on Pl not on L1, C does not lie on P1.
man’s relative and 3 of the wife’s relatives ? (1985)
(b) A’ lies on L2, B lies on P2 not on L2, C’ does not lies on P2.
31. A box contains two white balls, three black balls and four
red balls. In how many ways can three balls be drawn from (2004)
the box, if at least one black ball is to be included in the 37. Let n be the number of ways in which 5 boys and 5 girls
draw ? (1986) can stand in a queue in such a way that all the girls stand
32. A student  is allowed  to  select  atmost  ‘n’ books from  a consecutively in the queue. Let m be the number of ways
collection of (2n + 1) books. If the total number of ways in in which 5 boys and 5 girls can stand in a queue in such
which he can select at least one books is 63, find the value a way that exactly four girls stand consecutively in the
of ‘n’. (1987) m
queue. Then the value of  is (2015)
33. Eighteen guests have to be seated half on each side of a n
long  table.  Four  particular  guests  desire  to  sit  on  one 38. Words of length 10 are formed using the letters A, B, C, D,
particular side and three other on the other side. Determine E, F, G, H, I, J. Let x be the number of such words where no
the number of ways in which the sitting arrangements can letter is repeated; and let y be the number of such words
be made. (1991) where exactly one letter is repeated twice and no other
34. A committee of 12 is to be formed from 9 women and 8 y
men. In how many ways this can be done if at least five letter is repeated. Then,   (2017)
9x
women have to be included in a committee ? In how many
of these committees
(a) the women are in majority ?
(b) the men are in majority ? (1994)
174 PERMUTATION AND COMBINATION

ANSWER KEY
EXERCISE - 1 : BASIC OBJECTIVE QUESTIONS

1. (a) 2. (a) 3. (b) 4. (b) 5. (c) 6. (c) 7. (b) 8. (c)


9. (d) 10. (a) 11. (d) 12. (a) 13. (b) 14. (c) 15. (a) 16. (b)
17. (c) 18. (a) 19. (c) 20. (a) 21. (c) 22. (b) 23. (c) 24. (c)
25. (b) 26. (d) 27. (c) 28. (d) 29. (a) 30. (b,c) 31. (c) 32. (b)
33. (a) 34. (c) 35. (d) 36. (a) 37. (d) 38. (b) 39. (c) 40. (a)
41. (a) 42. (a) 43. (c) 44. (a) 45. (c) 46. (d) 47. (c) 48. (c)
49. (c) 50. (b) 51. (a,b,d) 52. (d) 53. (b) 54. (b) 55. (b) 56. (c)
57. (c) 58. (a) 59. (b) 60. (b) 61. (a) 62. (b) 63. (a) 64. (b)
65. (a) 66. (a) 67. (a) 68. (d) 69. (c) 70. (d) 71. (c) 72. (b)
73. (c) 74. (d) 75. (b) 76. (c) 77. (a) 78. (a) 79. (d) 80. (b)

EXERCISE - 2 : PREVIOUS YEAR JEE MAINS QUESTIONS

1. (b) 2. (a) 3. (b) 4. (a) 5. (c) 6. (b) 7. (d) 8. (b)


9. (c) 10. (a) 11. (a) 12. (c) 13. (b) 14. (a) 15. (d) 16. (b)
17. (b) 18. (d) 19. (b) 20. (b) 21. (d) 22. (c) 23. (a) 24. (c)
25. (d) 26. (b) 27. (a) 28. (a) 29. (c) 30. (a) 31. (b) 32. (b)
33. (d) 34. (b)

EXERCISE - 3 : ADVANCED OBJECTIVE QUESTIONS

1. (a) 2. (b) 3. (c) 4. (d) 5. (a) 6. (c) 7. (b) 8. (c)


9. (d) 10. (b) 11. (c) 12. (c) 13. (b) 14. (b) 15. (c) 16. (d)
17. (c) 18. (b) 19. (b) 20. (b) 21. (c) 22. (c) 23. (c) 24. (d)
25. (d) 26. (b) 27. (b) 28. (d) 29. (c) 30. (a) 31. (b) 32. (b)
33. (b) 34. (b) 35. (b) 36. (b) 37. (a) 38. (a) 39. (a) 40. (a)
41. (a) 42. (a) 43. (d) 44. (a) 45. (a) 46. (c) 47. (d) 48. (b)
49. (b) 50. (a) 51. (c) 52. (b) 53. (a, b) 54. (b,d) 55. (d) 56. (c)
57. (b) 58. (d) 59. (b) 60. (b) 61. (c) 62. (d) 63. (c) 64. (b)
65. (b) 66. (b) 67. (d) 68. (b) 69. (a) 70. (d) 71. (a) 72. (a)
73. (c) 74. (c) 75. (a) 76. (d) 77. (b) 78. (d) 79. (c) 80. (a, b, d)
81. (b, c) 82. (b, c) 83. (b, c) 84. (b, c, d) 85. (b, d) 86. (a, b, c, d) 87. (c, d) 88. (a, c, d)
89. (a, b, d) 90. (c) 91. (b) 92. (b)
93. (a  r); (b  s); (c  q); (d  p) 94. (a  t); (b  r); (c  p); (d  q); (e  s)
95. (a  s); (b  q); (c  p); (d  s) 96. 771 ways
97. 330 98. 159 99. 0141 100. 296
EXERCISE - 4 : PREVIOUS YEAR JEE ADVANCED QUESTIONS

1. (a) 2. (c) 3. (d) 4. (a) 5. (b) 6. (a) 7. (c) 8. (a)


9. (c) 10. (c) 11. (c) 12. (b) 13. (d) 14. (a) 15. (b) 16. (c)
17. (a) 18. A–p; B–s; C–q; D–q 19. (A–S; B–U; C–T; D–Q) 20. 2n – 1 21. 35 ways 22. 9
 52  !  52  !  52  !
23. (625) 24. True 25. n = 9 and r = 3 26. (a)  4   (b)  4  (c)  3 27. (150)
13! 4! 13! 3! 17 
30. (485) 31. (64) 32. (3) 33. 9P4 × 9P3 × (11)! 34. 6062, (a) 2702 (b) 1008 37. (5)
38.  (5)

You might also like